Sie sind auf Seite 1von 137

1. The only way to travel is on foot 1 12.

Religion and Rationality 64


2. Vicious and Dangerous Sports Should be Banned by Law 3 13. Cryptic Coloring 67
3. Advertisers Perform a Useful Service to the Community 5 14. Pageants 70
4. Preface 7 15. The Second Wave of Feminism 72
5. Meditation in Indonesian Business 9 16. Crime in Computer 74
6. Dropouts for Ph.D.s 11 17. A Strong Stock Market 76
7. Stricter Traffic Law can Prevent Accidents 13 18. The Military Is In 78
8. The Development of Cities 15 19. Creative Process of Works 80
9. Holmes Knowledge 17 20. NCB in Interpol 82
10. Antinuclear Demonstration 19 21. The Result of the Falling US Dollar 84
11. Superconducting Materials 21 22. Pantomime 86
12. We Should All Grow Fat and Be Happy 23 23. Women and Fashions 88
13. The Neutrality of American in the Early World War II 25 24. A Smuggling Syndicate 90
14. Antarctica and Environment 27 25. Exploration of the Titanic 92
15. Contribution of Coeducation 29 26. Sensory Evaluation of Food 94
16. Mules 31 27. Analysis and Interpretation of the News 96
17. On the Presidents Program 33 28. Chemistry and Biology 98
18. Strictly Ban smoking 35 29. Men Are Carrying on a Sex-fight 100
19. TVs Harmfulness 37 30. Fingerprints 102
20. The Law to Keep the Oil Industry under Control 39 31. The Causes of European Separation in 16th Century 104
32. The Young Generation 106
33. Importance of a Computer 108
1. Violence Can Do Nothing to Diminish Race Prejudice 41 34. The Relationship between Brain Process with Mental Experi-
2. The Tourist Trade Contributes Absolutely Nothing to Increasing ence 110
Understanding between Nations 43 35. Exploration on the Origin of Continents 112
3. Pop Stars Earn Much 45
4. Examinations Exert a Pernicious Influence on Education 47
5. Killing in the Name of God - Ugandan Deaths Spotlight Rise of 1. Clinton Is Right 114
Cults 49 2. Europes Gypsies, Are They a Nation? 117
6. Equality of opportunity in the twentieth Century Has Not De- 3. Method of Scientific Inquiry 120
stroyed the Class System 52 4. It Is Bush 122
7. The Most Important of All Human Qualities is a Sense of Humor 5. Womens Positions in the 17th Century 125
 54 6. The Present Is the Most Important 127
8. The Improving Economic Situation In Greece 56 7. Forecasting of Statistics 129
9. The Program-Federal Government Helps Minority Business 58 8. Wakefield Masters Realism 131
10. The Importance of Independent Thinking 60 9. The Continuity of the Religious Struggle in Britain 133
11. The Affect of Electricity on Cancer 62 10. Photography and Art 135

Passage 1: The only way to travel is on foot


The past ages of man have all been carefully labeled by anthropologists. Descriptions like Palaeolithic
Man, Neolithic Man, etc., neatly sum up whole periods. When the time comes for anthropologists to turn their at-
tention to the twentieth century, they will surely choose the label Legless Man. Histories of the time will go some-
thing like this: in the twentieth century, people forgot how to use their legs. Men and women moved about in cars,
buses and trains from a very early age. There were lifts and escalators in all large buildings to prevent people from
walking. This situation was forced upon earth dwellers of that time because of miles each day. But the surprising
thing is that they didnt use their legs even when they went on holiday. They built cable railways, ski-lifts and roads
to the top of every huge mountain. All the beauty spots on earth were marred by the presence of large car parks.
The future history books might also record that we were deprived of the use of our eyes. In our hurry to get
from one place to another, we failed to see anything on the way. Air travel gives you a birds-eye view of the world
or even less if the wing of the aircraft happens to get in your way. When you travel by car or train a blurred
image of the countryside constantly smears the windows. Car drivers, in particular, are forever obsessed with the
urge to go on and on: they never want to stop. Is it the lure of the great motorways, or what? And as for sea travel,
it hardly deserves mention. It is perfectly summed up in the words of the old song: I joined the navy to see the
world, and what did I see? I saw the sea. The typical twentieth-century traveler is the man who always says Ive
been there. You mention the remotest, most evocative place-names in the world like El Dorado, Kabul, Irkutsk and
someone is bound to say Ive been there meaning, I drove through it at 100 miles an hour on the way to some-
where else.
When you travel at high speeds, the present means nothing: you live mainly in the future because you spend
most of your time looking forward to arriving at some other place. But actual arrival, when it is achieved, is mean-
ingless. You want to move on again. By traveling like this, you suspend all experience; the present ceases to be a
reality: you might just as well be dead. The traveler on foot, on the other hand, lives constantly in the present. For
him traveling and arriving are one and the same thing: he arrives somewhere with every step he makes. He experi-
ences the present moment with his eyes, his ears and the whole of his body. At the end of his journey he feels a deli-
cious physical weariness. He knows that sound. Satisfying sleep will be his: the just reward of all true travellers.

1. Anthorpologists label nowadays men Legless because


[A] people forget how to use his legs.
[B] people prefer cars, buses and trains.
[C] lifts and escalators prevent people from walking.
[D] there are a lot of transportation devices.
2. Travelling at high speed means
[A] peoples focus on the future.
[B] a pleasure.
[C] satisfying drivers great thrill.
[D] a necessity of life.
3. Why does the author say we are deprived of the use of our eyes ?
[A] People wont use their eyes.
[B] In traveling at high speed, eyes become useless.
[C] People cant see anything on his way of travel.

1
[D] People want to sleep during travelling.
4. What is the purpose of the author in writing this passage?
[A] Legs become weaker.
[B] Modern means of transportation make the world a small place.
[C] There is no need to use eyes.
[D] The best way to travel is on foot.
5. What does a birds-eye view mean?
[A] See view with birds eyes.
[B] A bird looks at a beautiful view.
[C] It is a general view from a high position looking down.
[D] A scenic place.

VOCABULARY
1. Palaeolithic
2. Neolithic
3. escalator
4. ski-lift
5. mar
6. blur
7. smear
8. evocative
9. El Dorado ()
10. Kabul
11. Irkutsk

ANSWER
1. A 2. A 3. C 4. D 5. C

2
Passage 2: Vicious and Dangerous Sports Should be Banned by Law

When you think of the tremendous technological progress we have made, its amazing how little we have
developed in other respects. We may speak contemptuously of the poor old Romans because they relished the orgies
of slaughter that went on in their arenas. We may despise them because they mistook these goings on for entertain-
ment. We may forgive them condescendingly because they lived 2000 years ago and obviously knew no better. But
are our feelings of superiority really justified? Are we any less blood-thirsty? Why do boxing matches, for instance,
attract such universal interest? Dont the spectators who attend them hope they will see some violence? Human be-
ings remains as bloodthirsty as ever they were. The only difference between ourselves and the Romans is that while
they were honest enough to admit that they enjoyed watching hungey lions tearing people apart and eating them
alive, we find all sorts of sophisticated arguments to defend sports which should have been banned long age; sports
which are quite as barbarous as, say, public hangings or bearbaiting.
It really is incredible that in this day and age we should still allow hunting or bull-fighting, that we should
be prepared to sit back and watch two men batter each other to pulp in a boxing ring, that we should be relatively
unmoved by the sight of one or a number of racing cars crashing and bursting into flames. Let us not deceive our-
selves. Any talk of the sporting spirit is sheer hypocrisy. People take part in violent sports because of the high re-
wards they bring. Spectators are willing to pay vast sums of money to see violence. A world heavyweight champion-
ship match, for instance, is front page news. Millions of people are disappointed if a big fight is over in two rounds
instead of fifteen. They feel disappointment because they have been deprived of the exquisite pleasure of witnessing
prolonged torture and violence.
Why should we ban violent sports if people enjoy them so much? You may well ask. The answer is simple:
they are uncivilized. For centuries man has been trying to improve himself spiritually and emotionally admittedly
with little success. But at least we no longer tolerate the sight madmen cooped up in cages, or public floggings
of any of the countless other barbaric practices which were common in the past. Prisons are no longer the grim
forbidding places they used to be. Social welfare systems are in operation in many parts of the world. Big efforts
are being made to distribute wealth fairly. These changes have come about not because human beings have sud-
denly and unaccountably improved, but because positive steps were taken to change the law. The law is the biggest
instrument of social change that we have and it may exert great civilizing influence. If we banned dangerous and
violent sports, we would be moving one step further to improving mankind. We would recognize that violence is
degrading and unworthy of human beings.

1. It can be inferred from the passage that the authors opinion of nowadays human beings is
[A] not very high.
[B] high.
[C] contemptuous.
[D] critical.
2. The main idea of this passage is
[A] vicious and dangerous sports should be banned by law.
[B] people are willing to pay vast sums money to see violence.
[C] to compare two different attitudes towards dangerous sports.
[D] people are bloodthirsty in sports.
3. That the author mentions the old Romans is
[A] To compare the old Romans with todays people.
[B] to give an example.

3
[C] to show human beings in the past know nothing better.
[D] to indicate human beings are used to bloodthirsty.
4. How many dangerous sports does the author mention in this passage?
[A] Three.
[B] Five.
[C] Six.
[D] Seven.
5. The purpose of the author in writing this passage is
[A] that, by banning the violent sports, we human beings can improve our selves.
[B] that, by banning the dangerous sports, we can improve the law.
[C] that we must take positive steps to improve social welfare system.
[D] to show law is the main instrument of social change.

VOCABULARY
1. relish
2. orgy
3. arena
4. blood-thirsty
5. bear-baiting
6. bull-fight
7. batter /
8. pulp
9. burst into flames /
10. grim
11. coop up

ANSWER
1. A 2. A 3. D 4. B 5. A

4
Passage 3: Advertisers Perform a Useful Service to the Community

Advertisers tend to think big and perhaps this is why theyre always coming in for criticism. Their critics
seem to resent them because they have a flair for self-promotion and because they have so much money to throw
around. Its iniquitous, they say, that this entirely unproductive industry (if we can call it that) should absorb
millions of pounds each year. It only goes to show how much profit the big companies are making. Why dont they
stop advertising and reduce the price of their goods? After all, its the consumer who pays
The poor old consumer! Hed have to pay a great deal more if advertising didnt create mass markets for
products. It is precisely because of the heavy advertising that consumer goods are so cheap. But we get the wrong
idea if we think the only purpose of advertising is to sell goods. Another equally important function is to inform.
A great deal of the knowledge we have about household goods derives largely from the advertisements we read.
Advertisements introduce us to new products or remind us of the existence of ones we already know about. Suppos-
ing you wanted to buy a washing machine, it is more than likely you would obtain details regarding performance,
price, etc., from an advertisement.
Lots of people pretend that they never read advertisements, but this claim may be seriously doubted. It is
hardly possible not to read advertisements these days. And what fun they often are, too! Just think what a railway
station or a newspaper would be like without advertisements. Would you enjoy gazing at a blank wall or reading
railway byelaws while waiting for a train? Would you like to read only closely printed columns of news in your
daily paper? A cheerful, witty advertisement makes such a difference to a drab wall or a newspaper full of the daily
ration of calamities.
We must not forget, either, that advertising makes a positive contribution to our pockets. Newspapers, com-
mercial radio and television companies could not subsist without this source of revenue. The fact that we pay so
little for our daily paper, or can enjoy so many broadcast programmes is due entirely to the money spent by adver-
tisers. Just think what a newspaper would cost if we had to pay its full price!
Another thing we mustnt forget is the small ads. which are in virtually every newspaper and magazine.
What a tremendously useful service they perform for the community! Just about anything can be accomplished
through these columns. For instance, you can find a job, buy or sell a house, announce a birth, marriage or death
in what used to be called the hatch, match and dispatch column but by far the most fascinating section is the per-
sonal or agony column. No other item in a newspaper provides such entertaining reading or offers such a deep
insight into human nature. Its the best advertisement for advertising there is!

1. What is main idea of this passage?


[A] Advertisement.
[B] The benefits of advertisement.
[C] Advertisers perform a useful service to communities.
[D] The costs of advertisement.
2. The attitude of the author toward advertisers is
[A] appreciative.
[B] trustworthy.
[C] critical.
[D] dissatisfactory.
3. Why do the critics criticize advertisers?
[A] Because advertisers often brag.
[B] Because critics think advertisement is a waste of money.

5
[C] Because customers are encouraged to buy more than necessary.
[D] Because customers pay more.
4. Which of the following is Not True?
[A] Advertisement makes contribution to our pockets and we may know everything.
[B] We can buy what we want.
[C] Good quality products dont need to be advertised.
[D] Advertisement makes our life colorful.
5. The passage is
[A] Narration.
[B] Description.
[C] Criticism.
[D] Argumentation.

VOCABULARY
1. come in for ( sth. )
2. flair
3. iniquitous
4. drab
5. subsist
6. hatch
7. match
8. dispatch
9. agony
10. agony column

ANSWER
1. C 2. A 3. A 4. C 5. C

6
Passage 4: Preface

Science is a dominant theme in our culture. Since it touches almost every facet of our life, educated people
need at least some acquaintance with its structure and operation. They should also have an understanding of the
subculture in which scientists live and the kinds of people they are. An understanding of general characteristics of
science as well as specific scientific concepts is easier to attain if one knows something about the things that excite
and frustrate the scientist.
This book is written for the intelligent student or lay person whose acquaintance with science is superficial;
for the person who has been presented with science as a musty storehouse of dried facts; for the person who sees
the chief objective of science as the production of gadgets; and for the person who views the scientists as some sort
of magician. The book can be used to supplement a course in any science, to accompany any course that attempts
to give an understanding of the modern world, or independently of any course simply to provide a better under-
standing of science. We hope this book will lead readers to a broader perspective on scientific attitudes and a more
realistic view of what science is, who scientists are, and what they do. It will give them an awareness and under-
standing of the relationship between science and our culture and an appreciation of the roles science may play in
our culture. In addition, readers may learn to appreciate the relationship between scientific views and some of the
values and philosophies that are pervasive in our culture.
We have tried to present in this book an accurate and up-to-date picture of the scientific community and
the people who populate it. That population has in recent years come to comprise more and more women. This
increasing role of women in the scientific subculture is not an unique incident but, rather, part of the trend evident
in all segments of society as more women enter traditionally male-dominated fields and make significant contribu-
tions. In discussing these changes and contribution, however, we are faced with a language that is implicitly sexist,
one that uses male nouns or pronouns in referring to unspecified individuals. To offset this built-in bias, we have
adopted the policy of using plural nouns and pronouns whenever possible and, when absolutely necessary, alter-
nating he and she. This policy is far form being ideal, but it is at least an acknowledgment of the inadequacy of our
language in treating half of the human race equally.
We have also tried to make the book entertaining as well as informative. Our approach is usually informal.
We feel, as do many other scientists, that we shouldnt take ourselves too seriously. As the reader may observe, we
see science as a delightful pastime rather than as a grim and dreary way to earn a living.

1. According to the passage, scientific subculture means


[A] cultural groups that are formed by scientists.
[B] people whose knowledge of science is very limited.
[C] the scientific community.
[D] people who make good contribution to science.
2. We need to know something about the structure and operation of science because
[A] it is not easy to understand the things that excite and frustrate scientists.
[B] Science affects almost every aspect of our life.
[C] Scientists live in a specific subculture.
[D] It is easier to understand general characteristics of science.
3. The book mentioned in this passage is written for readers who
[A] are intelligent college students and lay person who do not know much about science.
[B] are good at producing various gadgets.
[C] work in a storehouse of dried facts.

7
[D] want to have a superficial understanding of science.
4. According to this passage,
[A] English is a sexist language.
[B] only in the scientific world is the role of women increasing rapidly.
[C] women are making significant contributions to eliminating the inadequacy of our language.
[D] male nouns or pronouns should not be used to refer to scientists.
5. This passage most probably is
[A] a book review.
[B] the preface of a book.
[C] the postscript of a book.
[D] the concluding part of a book.

VOCABULARY
1. subculture
2. superficial
3. lay person
4. musty
5. gadget
6. pervasive
7. populate
8. implicit
9. unspecified
10. offset

ANSWER
1. C 2. B 3. A 4. D 5. B

8
Passage 5: Meditation in Indonesian Business

It looked like a typical business meeting. Six men, neatly dressed in white shirts and ties filed into the
boardroom of a small Jakarta company and sat down at a long table. But instead of consulting files or hearing
reports, they closed their eyes and began to meditate, consulting the spirits of ancient Javanese kings. Mysticism
touches almost every aspect of life in Indonesia and business is no exception. One of the meditators said his weekly
meditation sessions are aimed mainly at bringing the peace of mind that makes for good decision-making. But the
insight gained from mystic communication with spirits of wise kings has also helped boost the profits of his five
companies.
Mysticism and profits have come together since the 13th century introduction of Islam to Indonesia by
Indian Moslem merchants. Those devout traders, called Wali Ullah or those close to God, energetically spread
both trade and religion by adapting their appeals to the native mysticism of Java. Legends attribute magic power of
foreknowledge to the Wali Ullah. These powers were believed to be gained through meditation and fasting.
Businessman Hadisiko said his group fasts and meditates all night every Thursday to become closer to
God and to contact the spirits of the great men of the past. If we want to employ someone at the managerial level,
we meditate together and often the message comes that this man cant hole onto money or he is untrustworthy. Or
maybe the spirits will tell us he should be hired. Hadiziko hastened to add that his companies also hold modern
personnel management systems and that formal qualifications are essential for a candidate even to be considered.
Perspective investments also are considered through mystic meditation. With the mind relaxed and open, it is
easier to be objective in judging the risk of a new venture. Meditation and contact with the wisdom of the old lead-
ers sharpens your own insight and intuition. Then you have to apply that intuition to the information you have and
work hard to be successful. Mystic meditation helped reverse a business slide his companies experienced in the
mid-1980. Operating with normal business procedures, he lost more than $ 3 millions in that year alone. Medi-
tation brought back his peace of mind. Putting the right persons in the right jobs and gaining confidence in his
business decisions were the keys to a turning around that has brought expansion and profitability. The mysticism in
Handspikes boardroom is part of a growing movement in Indonesia called Kebatinan the search for the inner
self.
One of his managers, Yusuf Soemado, who studied business administration at Harvard University, com-
pared the idea of mystic management to western system of positive thinking. Willpower and subconscious mind are
recognized as important factors in business. Such approaches as psycho-cybernetics, Carnegies think and growth
rates, or the power of positive thinking are western attempts to tap the same higher intelligence that we contact
through meditation, he said.

1. What is the most important factor in their doing business?


[A] Mysticism.
[B] Religion.
[C] Meditation.
[D] Investment.
2. Whom do they consult?
[A] The spirits of ancient Javanese kings.
[B] Wali Ullah.
[C] Old Kings.
[D] Carnegie.
3. Why did Hadisike hasten to add his companies also hold modern personnel management systems?

9
[A] He thought Mysticism was not so good as expected.
[B] To show they too focused on qualifications.
[C] To show they hired qualified persons.
[D] To show the possibility of combination of the scientific management with religion.
4. According to the passage, the function of the meditation is
[A] to gain profit from the god.
[B] to gain peace of mind to make decision.
[C] to gain foreknowledge.
[D] to gain objective conclusion.
5. What does operating with normal business procedures refer to?
[A] Adopting the western way of doing business.
[B] Ordinary way of doing business without meditation and fasting.
[C] Contact with God.
[D] Putting right persons in the right jobs.

VOCABULARY
1. file into
2. Jakarta
3. meditate
4. Java
5. Javanese
6. mysticism
7. boost
8. devout
9. appeal (to)
10. legend
11. fasting
12. hold onto
13. personnel management system
14. perspective investment
15. venture
16. sharpen /
17. business slide /
18. turn around
19. subconscious
20. cybernetics
21. Carnegie
22. tap

ANSWER
1. C 2. A 3. B 4. B 5. B

10
Passage 6: Dropouts for Ph.D.s

Educators are seriously concerned about the high rate of dropouts among the doctor of philosophy candi-
dates and the consequent loss of talent to a nation in need of Ph. D. s. Some have placed the dropouts loss as high
as 50 percent. The extent of the loss was, however, largely a matter of expert guessing. Last week a well-rounded
study was published. It was published. It was based on 22,000 questionnaires sent to former graduate students who
were enrolled in 24 universities and it seemed to show many past fears to be groundless.
The dropouts rate was found to be 31 per cent, and in most cases the dropouts, while not completing the
Ph. D. requirement, went on to productive work. They are not only doing well financially, but, according to the
report, are not far below the income levels of those who went on to complete their doctorates.
Discussing the study last week, Dr. Tucker said the project was initiated because of the concern frequently
expressed by graduate faculties and administrators that some of the individuals who dropped out of Ph. D. pro-
grams were capable of competing the requirement for the degree. Attrition at the Ph. D. level is also thought to be
a waste of precious faculty time and a drain on university resources already being used to capacity. Some people
expressed the opinion that the shortage of highly trained specialists and college teachers could be reduced by per-
suading the dropouts to return to graduate schools to complete the Ph. D.
The results of our research Dr. Tucker concluded, did not support these opinions.
1. Lack of motivation was the principal reason for dropping out.
2. Most dropouts went as far in their doctoral program as was consistent with their levels of ability or their
specialities.
3. Most dropouts are now engaged in work consistent with their education and motivation.
Nearly 75 per cent of the dropouts said there was no academic reason for their decision, but those who
mentioned academic reason cited failure to pass the qualifying examination, uncompleted research and failure to
pass language exams. Among the single most important personal reasons identified by dropouts for non-completion
of their Ph. D. program, lack of finances was marked by 19 per cent.
As an indication of how well the dropouts were doing, a chart showed 2% in humanities were receiving
$ 20,000 and more annually while none of the Ph. D. s with that background reached this figure. The Ph. D. s
shone in the $ 7,500 to $ 15,000 bracket with 78% at that level against 50% for the dropouts. This may also be an
indication of the fact that top salaries in the academic fields, where Ph. D. s tend to rise to the highest salaries, are
still lagging behind other fields.
As to the possibility of getting dropouts back on campus, the outlook was glum. The main condition which
would have to prevail for at least 25 % of the dropouts who might consider returning to graduate school would be
to guarantee that they would retain their present level of income and in some cases their present job.

1. The author states that many educators feel that


[A] steps should be taken to get the dropouts back to campus.
[B] the dropouts should return to a lower quality school to continue their study.
[C] the Ph. D. holder is generally a better adjusted person than the dropout.
[D] The high dropouts rate is largely attributable to the lack of stimulation on the part of faculty members.
2. Research has shown that
[A] Dropouts are substantially below Ph. D. s in financial attainment.
[B] the incentive factor is a minor one in regard to pursuing Ph. D. studies.
[C] The Ph. D. candidate is likely to change his field of specialization if he drops out.
[D] about one-third of those who start Ph. D. work do not complete the work to earn the degree.

11
3. Meeting foreign language requirements for the Ph. D.
[A] is the most frequent reason for dropping out.
[B] is more difficult for the science candidate than for the humanities candidate.
[C] is an essential part of many Ph. D. programs.
[D] does not vary in difficulty among universities.
4. After reading the article, one would refrain from concluding that
[A] optimism reigns in regard to getting Ph. D. dropouts to return to their pursuit of the degree.
[B] a Ph. D. dropout, by and large, does not have what it takes to learn the degree.
[C] colleges and universities employ a substantial number of Ph. D. dropouts.
[D] Ph. D. s are not earning what they deserve in nonacademic positions.
5. It can be inferred that the high rate of dropouts lies in
[A] salary for Ph. D. too low.
[B] academic requirement too high.
[C] salary for dropouts too high.
[D] 1000 positions.

VOCABULARY
1. dropout
2. well-rounded
3. attrition /
4. drain
5. bracket
6. lagging behind other fields
7. glum

ANSWER
1. A 2. D 3. C 4. A 5. A

12
Passage 7: Stricter Traffic Law can Prevent Accidents

From the health point of view we are living in a marvelous age. We are immunized from birth against many
of the most dangerous diseases. A large number of once fatal illnesses can now be cured by modern drugs and
surgery. It is almost certain that one day remedies will be found for the most stubborn remaining diseases. The ex-
pectation of life has increased enormously. But though the possibility of living a long and happy life is greater than
ever before, every day we witness the incredible slaughter of men, women and children on the roads. Man versus
the motor-car ! It is a never-ending battle which man is losing. Thousands of people the world over are killed or
horribly killed each year and we are quietly sitting back and letting it happen.
It has been rightly said that when a man is sitting behind a steering wheel, his car becomes the extension of
his personality. There is no doubt that the motor-car often brings out a mans very worst qualities. People who are
normally quiet and pleasant may become unrecognizable when they are behind a steering-wheel. They swear, they
are ill-mannered and aggressive, willful as two-years-olds and utterly selfish. All their hidden frustrations, disap-
pointments and jealousies seem to be brought to the surface by the act of driving.
The surprising thing is that society smiles so benignly on the motorist and seems to condone his behav-
iour. Everything is done for his convenience. Cities are allowed to become almost uninhabitable because of heavy
tragic; towns are made ugly by huge car parks; the countryside is desecrated by road networks; and the mass an-
nual slaughter becomes nothing more than a statistic, to be conveniently forgotten.
It is high time a world code were created to reduce this senseless waste of human life. With regard to driv-
ing, the laws of some countries are notoriously lax and even the strictest are not strict enough. A code which was
universally accepted could only have a dramatically beneficial effect on the accident rate. Here are a few examples
of some the things that might be done. The driving test should be standardized and made far more difficult than it
is; all the drivers should be made to take a test every three years or so; the age at which young people are allowed
to drive any vehicle should be raised to at least 21; all vehicles should be put through stringent annual tests for
safety. Even the smallest amount of alcohol in the blood can impair a persons driving ability. Present drinking and
driving laws (where they exist) should be mad much stricter. Maximum and minimum speed limits should be im-
posed on all roads. Governments should lay down safety specifications for manufacturers, as has been done in the
USA. All advertising stressing power and performance should be banned. These measures may sound inordinately
harsh. But surely nothing should be considered as to severe if tit results in reducing the annual toll of human life.
After all, the world is for human beings, not motor-cars.

1. The main idea of this passage is


[A] Traffic accidents are mainly caused by motorists.
[B] Thousands of people the world over are killed each year.
[C] The laws of some countries about driving are too lax.
[D] Only stricter traffic laws can prevent accidents.
2. What does the author think of society toward motorists?
[A] Society smiles on the motorists.
[B] Huge car parks are built in the cities and towns.
[C] Victims of accidents are nothing.
[D] Society condones their rude driving.
3. Why does the author say: his car becomes the extension of his personality?
[A] Driving can show his real self.
[B] Driving can show the other part of his personality.

13
[C] Driving can bring out his character.
[D] His car embodies his temper.
4. Which of the followings is NOT mentioned as a way against traffic accidents?
[A] Build more highways.
[B] Stricter driving tests.
[C] Test drivers every three years.
[D] raise age limit and lay down safety specifications.
5. The attitude of the author is
[A] ironical
[B] critical
[C] appealing
[D] militant

VOCABULARY
1. immunise
2. expectation of life = life expectancy
3. versus = against
4. mutilate
5. wilful
6. benign
7. condone
8. desecrate
9. code
10. stringent
11. performance

ANSWER
1. D 2. D 3. A 4. A 5. B

14
Passage 8: The Development of Cities

Mass transportation revised the social and economic fabric of the American city in three fundamental
ways. It catalyzed physical expansion, it sorted out people and land uses, and it accelerated the inherent instabil-
ity of urban life. By opening vast areas of unoccupied land for residential expansion, the omnibuses, horse rail-
ways, commuter trains, and electric trolleys pulled settled regions outward two to four times more distant form city
centers than they were in the premodern era. In 1850, for example, the borders of Boston lay scarcely two miles
from the old business district; by the turn of the century the radius extended ten miles. Now those who could afford
it could live far removed from the old city center and still commute there for work, shopping, and entertainment.
The new accessibility of land around the periphery of almost every major city sparked an explosion of real estate
development and fueled what we now know as urban sprawl. Between 1890 and 1920, for example, some 250,000
new residential lots were recorded within the borders of Chicago, most of them located in outlying areas. Over the
same period, another 550,000 were plotted outside the city limits but within the metropolitan area. Anxious to take
advantage of the possibilities of commuting, real estate developers added 800,000 potential building sites to the
Chicago region in just thirty years lots that could have housed five to six million people.
Of course, many were never occupied; there was always a huge surplus of subdivided, but vacant, land
around Chicago and other cities. These excesses underscore a feature of residential expansion related to the
growth of mass transportation: urban sprawl was essentially unplanned. It was carried out by thousands of small
investors who paid little heed to coordinated land use or to future land users. Those who purchased and prepared
land for residential purposes, particularly land near or outside city borders where transit lines and middle-class
inhabitants were anticipated, did so to create demand as much as to respond to it. Chicago is a prime example of
this process. Real estate subdivision there proceeded much faster than population growth.

1. With which of the following subjects is the passage mainly concerned?


[A] Types of mass transportation.
[B] Instability of urban life.
[C] How supply and demand determine land use.
[D] The effect of mass transportation on urban expansion.
2. Why does the author mention both Boston and Chicago?
[A] To demonstrate positive and negative effects of growth.
[B] To exemplify cities with and without mass transportation.
[C] To show mass transportation changed many cities.
[D] To contrast their rate of growth.
3. According to the passage, what was one disadvantage of residential expansion?
[A] It was expensive.
[B] It happened too slowly.
[C] It was unplanned.
[D] It created a demand for public transportation.
4. The author mentions Chicago in the second paragraph as an example of a city,
[A] that is large.
[B] that is used as a model for land development.
[C] where the development of land exceeded population growth.
[D] with an excellent mass transportation system.

15
VOCABULARY
1. revise
2. fabric
3. catalyze
4. sort out
5. omnibus /
6. trolley
7. periphery
8. sprawl
9. lot
10. underscore
11. transit lines
12. subdivision

ANSWER
1. D 2. C 3. C 4. C

16
Passage 9: Holmes Knowledge
His ignorance was as remarkable as his knowledge. Of contemporary literature, philosophy and politics he
appeared to know next to nothing. Upon my quoting Thomas Carlyle, he inquired in the naivest way who he might
be and what he had done. My surprise reached a climax, however, when I found incidentally that he was ignorant
of the Copernican Theory and of the composition of the Solar system.
You appear to be astonished, Holmes said, smiling at my expression. Now that I do know it I shall do
my best to forget it. You see, I consider that a mans brain originally is like a little empty attic, and you have to
stock it with such furniture as you choose: A fool takes in all the lumber of every sort that he comes across, so that
the knowledge which might be useful to him gets crowded out, or at best jumbled up with a lot of other things, so
that he has difficulty in laying his hand upon it. It is a mistake to think that the little room has elastic walls and can
distend to any extent. Depend upon it, there comes a time when for every addition of knowledge you forget some-
thing that you know before. It is of the highest importance, therefore, not to have useless facts elbowing out the
useful ones.
But the Solar System! I protested.
What the deuce is it to me? he interrupted impatiently.
One morning, I picked up a magazine from the table and attempted to while away the time with it, while
my companion munched silently at his toast. One of the articles had a pencil mark at the heading, and I naturally
began to run my eye through it.
Its somewhat ambitious title was The Book of Life, and it attempted to show how much an observant
man might learn by an accurate and systematic examination of all that came in his way. It struck me as being a re-
markable mixture of shrewdness and of absurdity. The reasoning was close and intense, but the deduction appeared
to me to be far-fetched and exaggerated. The writer claimed by a momentary expression, a twitch of a muscle or
a glance of an eye, to fathom a mans inmost thought. Deceit, according to him, was impossibility in the case of
one trained to observation and analysis. His conclusions were as infallible as so many propositions of Euclid. So
startling would his results appear to the uninitiated that until they learned the processes by which he had arrived at
them they might well consider him as a necromancer.
From a drop of water, said the writer, a logician could infer the possibility of an Atlantic. So all life is
a great chain, the nature of which is known whenever we are shown a single link of it. Like all other arts, the sci-
ence of Deduction and Analysis is one which can be acquired by long and patient study, nor is life long enough to
allow any mortal to attain the highest possible perfection in it.
This smartly written piece of theory I could not accept until a succession of evidences justified it.

1. What is the authors attitude toward Holmes?


[A]Praising.
[B]Critical.
[C]Ironical.
[D]Distaste.
2. What way did the author take to stick out Holmes uniqueness?
[A]By deduction.
[B]By explanation.
[C]By contrast.
[D]By analysis.
3. What was the Holmes idea about knowledge-learning?
[A]Learning what every body learned.

17
[B]Learning what was useful to you.
[C]Learning whatever you came across.
[D]Learning what was different to you.
4. What did the article mentioned in the passage talk about?
[A]One may master the way of reasoning through observation.
[B]One may become rather critical through observation and analysis.
[C]One may become rather sharp through observation and analysis.
[D]One may become practical through observation and analysis.

VOCABULARY
1. Thomas Carlyle 1795-1881
2. jumble (up)
3. lay hand on (upon) sth.
4. at best
5. elbow out (off)
6. deuce = devil
what the deuce is it to me?
7. while away the time /
8. shrewdness
9. far-fetched
10. fathom /
11. infallible
12. uninitiated
13. Euclid
14. necromancer

ANSWER
1. A 2. C 3. B 4. C

18
Passage 10: Antinuclear Demonstration

Police fired tear gas and arrested more than 5,000 passively resisting protestors Friday in an attempt to
break up the largest antinuclear demonstration ever staged in the United States. More than 135,000 demonstrators
confronted police on the construction site of a 1,000-megawatt nuclear power plant scheduled to provide power to
most of southern New Hampshire. Organizers of the huge demonstration said, the protest was continuing despite
the police actions. More demonstrators were arriving to keep up the pressure on state authorities to cancel the
project. The demonstrator had charged that the project was unsafe in the densely populated area, would create
thermal pollution in the bay, and had no acceptable means for disposing of its radioactive wasters. The demonstra-
tions would go on until the jails and the courts were so overloaded that the state judicial system would collapse.
Governor Stanforth Thumper insisted that there would be no reconsideration of the power project and no
delay in its construction set for completion in three years. This project will begin on time and the people of this
state will begin to receive its benefits on schedule. Those who break the law in misguided attempts to sabotage the
project will be dealt with according to the law, he said. And police called in reinforcements from all over the state
to handle the disturbances.
The protests began before dawn Friday when several thousand demonstrators broke through police lines
around the cordoned-off construction site. They carried placards that read No Nukes is Good Nukes, Sunpow-
er, Not Nuclear Power, and Stop Private Profits from Public Peril. They defied police order to move from the
area. Tear gas canisters fired by police failed to dislodge the protestors who had come prepared with their own gas
masks or facecloths. Finally gas-masked and helmeted police charged into the crowd to drag off the demonstrators
one by one. The protestors did not resist police, but refused to walk away under their own power. Those arrested
would be charged with unlawful assembly, trespassing, and disturbing the peace.

1. What were the demonstrators protesting about?


[A] Private profits.
[B] Nuclear Power Station.
[C] The project of nuclear power construction.
[D] Public peril.
2. Who had gas-masks?
[A] Everybody.
[B] A part of the protestors.
[C] Policemen.
[D] Both B and C.
3. Which of the following was NOT mentioned as a reason for the demonstration?
[A] Public transportation.
[B] Public peril.
[C] Pollution.
[D] Disposal of wastes.
4. With whom were the jails and courts overloaded?
[A] With prisoners.
[B] With arrested demonstrators.
[C] With criminals.
[D] With protestors.
5. What is the attitude of Governor Stanforth Thumper toward the power project and the demonstration?

19
[A] stubborn.
[B] insistent.
[C] insolvable.
[D] remissible.

VOCABULARY
1. tear gas
2. passively resisting protestor
3. stage
4. break up
5. cordon
6. nuke
7. defy /
8. canister
9. dislodge
10. charge
11. trespass

ANSWER
1. C 2. D 3. A 4. B 5. A

20
Passage 11: Superconducting Materials

The stone age, The Iron Age. Entire epochs have been named for materials. So what to call the decades
ahead? The choice will be tough. Welcome to the age of superstuff. Material science -- once the least sexy technolo-
gy is bursting with new, practical discoveries led by superconducting ceramics that may revolutionize electronics.
But superconductors are just part of the picture: from house and cars to cook pots and artificial teeth, the world
will someday be made of different stuff. Exotic plastics, glass and ceramics will shape the future just as surely as
have genetic engineering and computer science.
The key to the new materials is researchers increasing ability to manipulate substances at the molecular
level. Ceramics, for example, have long been limited by their brittleness. But by minimizing the microscopic imper-
fections that cause it, scientists are making far stronger ceramics that still retain such qualities as hardness and
heat resistance. Ford Motor Co. now uses ceramic tools to cut steel. A firm called Kyocera has created a line of
ceramic scissors and knives that stay sharp for years and never rust or corrode.
A similar transformation has overtaken plastics. High-strength polymers now form bridges, ice-skating
rinks and helicopter rotors. And one new plastic that generates electricity when vibrated or pushed is used in
electric guitars, touch sensors for robot hands and karate jackets that automatically record each punch and chop.
Even plastic litter, which once threatened to permanently blot the landscape, has proved amenable to molecular
tinkering. Several manufacturers now make biodegradable forms; some plastic six-pack rings for example, gradu-
ally decompose when exposed to sunlight. Researchers are developing ways to make plastics as recyclable as metal
or glass. Besides, composites plastic reinforced with fibers of graphite or other compounds made the round-the-
world flight of the voyager possible and have even been proved in combat: a helmet saved an infantrymans life by
deflecting two bullets in the Grenada invasion.
Some advanced materials are old standard with a new twist. The newest fiberoptic cable that carry tele-
phone calls cross-country are made of glass so transparent that a piece of 100 miles thick is clearer than a stan-
dard window pane.
But new materials have no impact until they are made into products. And that transition could prove dif-
ficult, for switching requires lengthy research and investment. It can be said a firmer handle on how to move to
commercialization will determine the success or failure of a country in the near future.

1. How many new materials are mentioned in this passage?


[A] Two
[B] Three
[C] Four
[D] Five
2. Why does the author mention genetic engineering and computer science?
[A] To compare them with the new materials.
[B] To show the significance of the new materials on the future world.
[C] To compare the new materials to them.
[D] To explain his view point.
3. Why is transition difficult?
[A] Because transition requires money and time.
[B] Because many manufacturers are unwilling to change their equipment.
[C] Because research on new materials is very difficult.
[D]Because it takes 10 years.

21
4. Where lies success of a country in the New Age of superstuff?
[A] It lies in research.
[B] It lies in investment.
[C] It lies in innovation.
[D] It lies in application.

VOCABULARY
1. superstuff
2. superconducting ceramic
3. exotic
4. shape
5. brittleness
6. polymer
7. karate jacket
8. touch sensor
9. each punch and chop
10. blot
11. tinker
12. amendable
13. biodegradable
14. six-pack rings
15. decompose
16. recyclable
17. infantryman
18. deflect
19. a new twist

ANSWER
1. B 2. B 3. A 4. D

22
Passage 12: We Should All Grow Fat and Be Happy

Heres a familiar version of the boy-meets-girl situation. A young man has at last plucked up courage to
invite a dazzling young lady out to dinner. She has accepted his invitation and he is overjoyed. He is determined to
take her to the best restaurant in town, even if it means that he will have to live on memories and hopes during the
month to come. When they get to the restaurant, he discovers that this ethereal creature is on a diet. She mustnt
eat this and she mustnt that. Oh, but of course, she doesnt want to spoil his enjoyment. Let him by all means eat
as much fattening food as he wants: its the surest way to an early grave. They spend a truly memorable evening
together and never see each other again.
What a miserable lot dieters are! You can always recognize them from the sour expression on their faces.
They spend most of their time turning their noses up at food. They are forever consulting calorie charts; gazing
at themselves in mirrors; and leaping on to weighing-machines in the bathroom. They spend a lifetime fighting a
losing battle against spreading hips, protruding tummies and double chins. Some wage all-out war on FAT. Mere
dieting is not enough. They exhaust themselves doing exercises, sweating in sauna baths, being pummeled and mas-
saged by weird machines. The really wealthy diet-mongers pay vast sums for health cures. For two weeks they
can enter a nature clinic and be starved to death for a hundred guineas a week. Dont think its only the middle-
aged who go in for these fads either. Many of these bright young things you see are suffering from chronic malnu-
trition: they are living on nothing but air, water and the goodwill of God.
Dieters undertake to starve themselves of their own free will; so why are they so miserable? Well, for one
thing, theyre always hungry. You cant be hungry and happy at the same time. All the horrible concoctions they eat
instead of food leave them permanently dissatisfied. Wonderfood is a complete food, the advertisement says. Just
dissolve a teaspoonful in water. A complete food it may be, but not quite as complete as a juicy steak. And, of
course, theyre always miserable because they feel so guilty. Hunger just proves too much for them and in the end
they lash out and devour five huge guilt-inducing cream cakes at a sitting. And who can blame them? At least three
times a day they are exposed to temptation. What utter torture it is always watching others tucking into piles of
mouth-watering food while you munch a water biscuit and sip unsweetened lemon juice!
Whats all this self-inflicted torture for? Saintly people deprive themselves of food to attain a state of grace.
Unsaintly people do so to attain a state of misery. It will be a great day when all the dieters in the world abandon
their slimming courses; when they hold out their plates and demand second helpings!

1. The best title for this passage is


[A] On Fat.
[B] We Should All Grow Fat and Be Happy.
[C] Many Diseases Are Connected with Fat.
[D] Diet Deprives People of Normal Life.
2. Why do they never see each other again?
[A] Because it is a memorable evening.
[B] Because she lets him eat as much fattening food as he wants.
[C] Because she does not eat this and drink that.
[D] Because eating fattening food is the surest way to an early grave.
3. Which of the following ways is NOT mentioned for diet?
[A] Doing exercises.
[B] Not eating sugar.
[C] Not eating fat.

23
[D] Taking sauna baths.
4. What is the authors attitude toward diet?
[A] Persuasive.
[B] Critical.
[C] Indifferent.
[D] Adversative.

VOCABULARY
1. ethereal
2. sour
3. tummy
4. protrude
5. wage
6. all-out
7. sauna bath
8. pummel = pommel
9. massage
10. weird
11. monger
12. fad
13. concoction
14. wonderfood
15. lash out
16. at a/one sitting
17. munch
18. inflict
19. slim

ANSWER
1. B 2. D 3. B 4. B

24
Passage 13: The Neutrality of American in the Early World War II

The establishment of the Third Reich influenced events in American history by starting a chain of events
which culminated in war between Germany and the United States. The compete destruction of democracy, the per-
secution of Jews, the war on religion, the cruelty and barbarism of the Nazis, and especially the plans of Germany
and her allies, Italy and Japan, for world conquest caused great indignation in this country and brought on fear of
another world war. While speaking out against Hitlers atrocities, the American people generally favored isolation-
ist policies and neutrality. The Neutrality Acts of 1935 and 1936 prohibited trade with any belligerents or loans to
them. In 1937 the President was empowered to declare an arms embargo in wars between nations at his discretion.
American opinion began to change somewhat after President Roosevelts quarantine the aggressor
speech at Chicago (1937) in which he severely criticized Hitlers policies. Germanys seizure of Austria and
the Munich Pact for the partition of Czechoslovakia (1938) also aroused the American people. The conquest of
Czechoslovakia in March, 1939 was another rude awakening to the menace of the Third Reich. In August,1939
came the shock of the Nazi-soviet Pact and in September the attack on Poland and the outbreak of European war.
The United States attempted to maintain neutrality in spite of sympathy for the democracies arrayed against the
Third Reich. The Neutrality Act of 1939 repealed the arms embargo and permitted cash and carry exports
of arms to belligerent nations. A strong national defense program was begun. A draft act was passed (1940) to
strengthen the military services. A Lend Act (1941) authorized the President to sell, exchange, or lend materials to
any country deemed necessary by him for the defense of the United States. Help was given to Britain by exchanging
certain overage destroyers for the right to establish American bases in British territory in the Western Hemisphere.
In August, 1940 President Roosevelt and Prime Minister Churchill met and issued the Atlantic Charter which
proclaimed the kind of a world which should be established after the war. In December, 1941, Japan launched the
unprovoked attack on the United States at Pearl Harbor. Immediately thereafter, Germany declared war on the
United States.

1. One item occurring before 1937 that the author does not mention in his list of actions that alienated the Ameri-
can public was
[A] the burning of the Reichstag.
[B] German plans for conquest.
[C] Nazi barbarism.
[D] the persecution of religious groups.
2. The Lend-Lease Act was designed to
[A] help the British.
[B] strengthen the national defense of the United States.
[C] promote the Atlantic Charter.
[D] avenge Pearl Harbor.
3. American Policy during the years 1935-1936 may be described as being
[A] watchful.
[B] isolationist.
[C] peaceful.
[D] indifferent.
4. The Neutrality Act of 1939
[A] permitted the selling of arms to belligerent nations.
[B] antagonized Japan.

25
[C] permitted the British to trade only with the Allies.
[D] led to Lend-Lease Act.
5. We entered the war against Germany
[A] because Germany declared war.
[B] because Japan was an ally of Germany.
[C] after Germany had signed the Nazi-soviet Pact.
[D] after peaceful efforts had failed.

VOCABULARY
1. Reich
2. atrocity
3. belligerent
4. discretion
5. empower
6. embargo
7. quarantine
8. partition
9. menace
10. repeal
11. overage destroyer
12. unprovoked
13. Neutrality Acts
14. Munich Pact
15. draft act
16. Lend Lease Act
17. Atlantic Charter
18. Pearl Harbor

ANSWER
1. A 2. B 3. B 4. A 5. A

26
Passage 14: Antarctica and Environment

Antarctica has actually become a kind of space station a unique observation post for detecting impor-
tant changes in the worlds environment. Remote from major sources of pollution and the complex geological and
ecological systems that prevail elsewhere, Antarctica makes possible scientific measurements that are often sharper
and easier to interpret than those made in other parts of the world.
Growing numbers of scientists therefore see Antarctica as a distant-early-warning sensor, where potential-
ly dangerous global trends may be spotted before they show up to the north. One promising field of investigation
is glaciology. Scholars from the United States, Switzerland, and France are pursuing seven separate but related
projects that reflect their concern for the health of the West Antarctic Ice Sheet a concern they believe the world
at large should share.
The Transantarctic Mountain, some of them more than 14,000 feet high, divide the continent into two very
different regions. The part of the continent to the east of the mountains is a high plateau covered by an ice sheet
nearly two miles thick. West of the mountain, the half of the continent south of the Americas is also covered by
an ice sheet, but there the ice rests on rock that is mostly well below sea level. If the West Antarctic Ice Sheet disap-
peared, the western part of the continent would be reduced to a sparse cluster of island.
While ice and snow are obviously central to many environmental experiments, others focus on the mysteri-
ous dry valley of Antarctica, valleys that contain little ice or snow even in the depths of winter. Slashed through
the mountains of southern Victoria Land, these valleys once held enormous glaciers that descended 9,000 feet
from the polar plateau to the Ross Sea. Now the glaciers are gone, perhaps a casualty of the global warming trend
during the 10,000 years since the ice age. Even the snow that falls in the dry valleys is blasted out by vicious winds
that roars down from the polar plateau to the sea. Left bare are spectacular gorges, rippled fields of sand dunes,
clusters of boulders sculptured into fantastic shapes by 100-mile-an-hour winds, and an aura of extraterrestrial
desolation.
Despite the unearthly aspect of the dry valleys, some scientists believe they may carry a message of hope
of the verdant parts of the earth. Some scientists believe that in some cases the dry valleys may soak up pollutants
faster than pollutants enter them.

1. What is the best title for this passage?


[A] Antarctica and environmental Problems.
[B] Antarctica: Earths Early-Warning station.
[C] Antarctica: a Unique Observation Post.
[D] Antarctica: a Mysterious Place.
2. What would the result be if the West Antarctic Ice Sheet disappeared?
[A] The western part of the continent would be disappeared.
[B] The western part of the continent would be reduced.
[C] The western part of the continent would become scattered Islands.
[D] The western part of the continent would be reduced to a cluster of Islands.
3. Why are the Dry Valleys left bare?
[A] Vicious wind blasts the snow away.
[B] It rarely snows.
[C] Because of the global warming trend and fierce wind.
[D] Sand dunes.
4. Which of the following is true?

27
[A] The Dry Valleys have nothing left inside.
[B] The Dry Valleys never held glaciers.
[C] The Dry Valleys may carry a message of hope for the verdant.
[D] The Dry Valleys are useless to scientists.

VOCABULARY
1. distant-early-warning sensor
2. plateau
3. slash
4. blast /
5. vicious
6. gorge
7. ripple
8. sand dune
9. verdant
10. extraterrestrial
11. aura

ANSWER
1. A 2. D 3. C 4. C

28
Passage 15: Contribution of Coeducation

Imagining being asked to spend twelve or so years of your life in a society which consisted only of mem-
bers of own sex. How would you react? Unless there was something definitely wrong with you, you wouldnt be too
happy about it, to say the least. It is all the more surprising therefore that so many parents in the world choose to
impose such abnormal conditions on their children conditions which they themselves wouldnt put up with for one
minute!
Any discussion of this topic is bound to question the aims of education. Stuffing childrens heads full of
knowledge is far from being foremost among them. One of the chief aims of educations is to equip future citizens
with all they require to take their place in adult society. Now adult society is made up of men and women, so how
can a segregated school possibly offer the right sort of preparation for it? Anyone entering adult society after years
of segregation can only be in for a shock.
A co-educational school offers children nothing less than a true version of society in miniature. Boys and
girls are given the opportunity to get to know each other, to learn to live together from their earliest years. They
are put in a position where they can compare themselves with each other in terms of academic ability, athletic
achievement and many of the extra-curricular activities which are part of school life. What a practical advantage
it is ( to give just a small example ) to be able to put on a school play in which the male parts will be taken by boys
and the female parts by girls! What nonsense co-education makes of the argument that boys are cleverer than girl
or vice-versa. When segregated, boys and girls are made to feel that they are a race apart. Rivalry between the
sexes is fostered. In a coeducational school, everything falls into its proper place.
But perhaps the greatest contribution of co-education is the healthy attitude to life it encourages. Boys
dont grow up believing that women are mysterious creatures airy goddesses, more like book-illustrations to a
fairy-tale, than human beings. Girls dont grow up imagining that men are romantic heroes. Years of living to-
gether at school dispel illusions of this kind. There are no goddesses with freckles, pigtails, piercing voices and
inky fingers. There are no romantic heroes with knobbly knees, dirty fingernails and unkempt hair. The awkward
stage of adolescence brings into sharp focus some of the physical and emotional problems involved in growing up.
These can better be overcome in a co-educational environment. Segregated schools sometimes provide the right
conditions for sexual deviation. This is hardly possible under a co-educational system. When the time comes for the
pupils to leave school, they are fully prepared to enter society as well-adjusted adults. They have already had years
of experience in coping with many of the problems that face men and women.

1. What is the best title for this passage?


[A] only co-education can be in harmony with society.
[B] people are in great need of co-education.
[C] any form of education other than co-education is simply unthinkable.
[D] co-education has many features.
2. what does co-education offer to children?
[A] A society.
[B] A true small model of society.
[C] A real life.
[D] True version of social condition.
3. According to the passage, what is one of the chief aims of education?
[A] It is for students to acquire knowledge.
[B] It is to equip future citizens with scientific technology.

29
[C] It is to equip future citizens with what is required in getting a position in society.
[D] It is for students to get academic achievements.
4. Why do boys and girls in co-education have no illusion about each other?
[A] They live together and know each other too well.
[B] Years of living together at school dismiss such illusion.
[C] co-education encourage them to have an healthy attitude toward life.
[D] They are familiar with each others problems.

VOCABULARY
1. to be in for = receive
He is in for punishment.
2. miniature
3. freckle
4. pigtail
5. knobbly = knobby
6. unkempt
7. sharp focus = clear view
bring into focus
bring into sharp focus
8. deviation

ANSWER
1. C 2. B 3. C 4. B

30
Passage 16: Mules

Although the top men in smuggling business must work together, most of a syndicates small fry, especially
the mules, know only their immediate contacts. If caught there is little they can give away. A mule probably will not
even know the name of the person who gives him his instructions, nor how to get in touch with him. Usually he even
does not know the person to whom he has to make delivery. He will be told just to sit tight in a certain hotel or bar
until someone contacts him. In this way if he is blown, coming through airport customs he cannot unwittingly lead
agents to the next link in the chain. All the persons at the receiving end do is to hang around the airport among
the waiting crowd, and see that the mule comes through safely. If he does not, he is dimply written off as a loss. To
make identification of mules easier, several syndicates have devised their own club ties so that a mule wearing
one can immediately be picked out.
Mules often receive careful training before embarking on their first journey. One Beirut organization, for
example, uses a room with three airline seats in it. There the trainee mules sit for hours on end wearing weighted
smuggling vests beneath their clothes, so that they become accustomed to standing up after a long flight in a natu-
ral way, and without revealing what they are carrying. An outfit in Brussels maintained a comfortable apartment
where the mules could relax and get a firm grip on themselves on the night before their first journey; they were
helped to dress before setting out for the airport in the morning. More often than not a courier will not know pre-
cisely where he is going or what flight number is until he is actually handed his tickets at the airport. This prevents
the careless boast in some bar or to a girl friend the night before.
Mules occasionally run off with the goods to keep the profit themselves. As insurance against this, a syn-
dicate often sends a high-up on the same plane to keep a wary eye on couriers, particularly new ones. Even then
things can go badly wrong. One international currency smuggler who was having trouble getting money out of
Britain was offered help by a group of men who said they were in a position to fix thing for a fee of course.
Foolishly, the smuggler agreed to accept their help. When he got to Londons Heathrow Airport, he handed over
to one of the men a black suitcase containing nearly $90,000 in cash, destined for Frankfurt. Just to keep an eye
on things, the smuggler went along on the same plane. When they landed at Frankfurt he was handed back his
suitcase. He beat a straight path to the mens toilet, opened the case, and found only old clothes. The courier had
switched suitcase en route, but the smuggler could hardly run to the police and complain that the man who was
smuggling money out of England for me has stolen it.

1. What is a mule?
[A] A person who sends smuggling goods for a syndicate is called mule.
[B] A person in charge of smuggling goods is called mule.
[C] A person who makes delivery for a syndicate is called mule.
[D] A person who receives instructions from a smuggler is called mule.
2. The sentence if he is blown in line (6) is closest in meaning to
[A] if he is arrested.
[B] if he is recognized, but not necessarily arrested.
[C] if he is recognized and arrested.
[D] if he runs away.
3. Why does the author give an example in the last paragraph?
[A] To show how a smuggler is caught.
[B] To show a smuggler is afraid of the police.
[C] To show to keep a wary eye on couriers is useless.

31
[D] To show mules may keep the profit for themselves.
4. How does a mule work?
[A] Jointly.
[B] Independently.
[C] consciously.
[D] Separately.

VOCABULARY
1. fry small fry
2. sit tight
3. blown
4. write off
5. written off as a loss
6. embark (on)
7. grip
8. high up
9. fix thing
10. beat
11. switch

ANSWER
1. C 2. B 3. D 4. D

32
Passage 17: On the Presidents Program

President Arling has put his long awaited economic restructuring program before the Congress. It provides
a coordinated program of investment credits, research grants, education reforms, and tax changes designed to
make American industry more competitive. This is necessary to reverse the economic slide into unemployment, lack
of growth, and trade deficits that have plagued the economy for the past six years.
The most liberal wing of the Presidents party has called for stronger and more direct action. They want an
incomes policy to check inflation while federal financing helps rebuild industry behind a wall of protective tariffs.
The Republicans, however, decry even the modest, graduated tax increases in the Presidents program.
They want tax cuts and more open market. They say if federal money has to be injected into the economy, let it
through defence spending.
Both these alternatives ignore the unique nature of the economic problem before us. It is not simply a
matter of markets or financing. The new technology allows vastly increased production for those able to master it.
But it also threatens those who fail to adopt it with permanent second-class citizenship in the world economy. If an
industry cannot lever itself up to the leading stage of technological advances, then it will not be able to compete
effectively. If it cannot do this, no amount of government protectionism or access to foreign markets can keep it
profitable for long. Without the profits and experience of technological excellence to reinvest, that industry can
only fall still further behind its foreign competitors.
So the crux is the technology and that is where the Presidents program focused. The danger is not that
a plan will not be passed, it is that the ideologues of right and left will distort the bill with amendments that will
blur its focus on technology. The economic restructuring plan should be passed intact. If we fail to restructure our
economy now, we may not get a second chance.

1. The focus of the Presidents program is on


[A] investment.
[B] economy.
[C] technology.
[D] tax.
2. What is the requirement of the most liberal wing of the Democratic-party?
[A] They want a more direct action.
[B] They want an incomes policy to check inflation.
[C] They want to rebuild industry.
[D] They want a wall of protective tariffs.
3. What is the editors attitude?
[A] support.
[B] distaste.
[C] Disapproval.
[D] Compromise.
4. The danger to the plan lies in
[A] the two parties objection.
[B] different idea of the two parties about the plan.
[C] its passage.
[D] distortion.
5. The passage is

33
[A] a review.
[B] a preface.
[C] a advertisement.
[D] an editorial.

VOCABULARY
1. reverse
2. slide
3. plague
4. tariff
5. decry
6. lever
7. crux
8. ideologue
9. intact
10. investment credit
11. research grant

ANSWER
1. C 2. A 3. A 4. D 5. D

34
Passage 18: Strictly Ban smoking

If you smoke and you still dont believe that theres a definite link between smoking and bronchial troubles,
heart disease and lung cancer, then you are certainly deceiving yourself. No one will accuse you of hypocrisy. Let
us just say that you are suffering from a bad case of wishful thinking. This neednt make you too uncomfortable
because you are in good company. Whenever the subject of smoking and health is raised, the governments of most
countries hear no evil, see no evil and smell no evil. Admittedly, a few governments have taken timid measures. In
Britain for instance, cigarette advertising has been banned on television. The conscience of the nation is appeased,
while the population continues to puff its way to smoky, cancerous death.
You dont have to look very far to find out why the official reactions to medical findings have been so luke-
warm. The answer is simply money. Tobacco is a wonderful commodity to tax. Its almost like a tax on our daily
bread. In tax revenue alone, the government of Britain collects enough from smokers to pay for its entire educa-
tional facilities. So while the authorities point out ever so discreetly that smoking may, conceivable, be harmful, it
doesnt do to shout too loudly about it.
This is surely the most short-sighted policy you could imagine. While money is eagerly collected in vast
sums with one hand, it is paid out in increasingly vaster sums with the other. Enormous amounts are spent on can-
cer research and on efforts to cure people suffering from the disease. Countless valuable lives are lost. In the long
run, there is no doubt that everybody would be much better-off if smoking were banned altogether.
Of course, we are not ready for such a drastic action. But if the governments of the world were honestly
concerned about the welfare of their peoples, youd think theyd conduct aggressive anti-smoking campaigns.
Far from it! The tobacco industry is allowed to spend staggering sums on advertising. Its advertising is as insidi-
ous as it is dishonest. We are never shown pictures of real smokers coughing up their lungs early in the morning.
That would never do. The advertisement always depict virile, clean-shaven young men. They suggest it is manly to
smoke, even positively healthy! Smoking is associated with the great open-air life, with beautiful girls, true love and
togetherness. What utter nonsense!
For a start, governments could begin by banning all cigarette and tobacco advertising and should then
conduct anti-smoking advertising campaigns of their own. Smoking should be banned in all public places like
theatres, cinemas and restaurants. Great efforts should be made to inform young people especially of the dire
consequences of taking up the habit. A horrific warning say, a picture of a deaths head should be included in
every packet of cigarettes that is sold. As individuals, we are certainly weak, but if governments acted honestly and
courageously, they could protect us from ourselves.

1. Why do a few governments take timid measures toward smoking?


[A] because they are afraid of people.
[B] Because diseases cost a lot.
[C] Because they are afraid of the cutting down of their revenue.
[D] Because they are afraid of manufacturers.
2. The tone of this passage is
[A] critical.
[B] ironical.
[C] distaste.
[D] amusing.
3. What does the sentence because you are in good company mean?
[A] you are backed by the government.

35
[B] You are not alone.
[C] You have good colleagues.
[D] Governments are blind to evils of smoking too.
4. What is the best title of this passage?
[A] World Governments should conduct serious campaigns against smoking.
[B] World governments take timid measures against smoking.
[C] smoking is the most important source of income to many countries.
[D] tobacco industry spends a large sum of money on medical research.

VOCABULARY
1. a wishful thinking
2. puff
puff its way to
3. lukewarm /
4. insidious
5. virile

ANSWER
1. C 2. B 3. D 4. A

36
Passage 19: TVs Harmfulness

Yes, but what did we use to do before there was television? How often we hear statements like this! Televi-
sion hasnt been with us all that long, but we are already beginning to forget what the world was like without it. Be-
fore we admitted the one-eyed monster into our homes, we never fond it difficult to occupy our spare time. We used
to enjoy civilized pleasures. For instance, we used to have hobbies, we used to entertain our friends and be enter-
tained by them, we used to go outside for our amusements to theatres, cinemas, restaurants and sporting events.
We even used to read books and listen to music and broadcast talks occasionally. All that belongs to the past. Now
all our free time is regulated by the goggle box. We rush home or gulp down our meals to be in time for this or that
programme. We have even given up sitting at table and having a leisurely evening meal, exchanging the news of
the day. A sandwich and a glass of beer will do anything, providing it doesnt interfere with the programme. The
monster demands and obtains absolute silence and attention. If any member of the family dares to open his mouth
during a programme, he is quickly silenced.
Whole generations are growing up addicted to the telly. Food is left uneaten, homework undone and sleep
is lost. The telly is a universal pacifier. It is now standard practice for mother to keep the children quiet by putting
them in the living-room and turning on the set. It doesnt matter that the children will watch rubbishy commercials
or spectacles of sadism and violence so long as they are quiet.
There is a limit to the amount of creative talent available in the world. Every day, television consumes
vast quantities of creative work. That is why most of the programmes are so bad: it is impossible to keep pace with
the demand and maintain high standards as well. When millions watch the same programmes, the whole world
becomes a village, and society is reduced to the conditions which obtain in preliterate communities. We become
utterly dependent on the two most primitive media of communication: pictures and the spoken word.
Television encourages passive enjoyment. We become content with second-hand experiences. It is so easy
to sit in our armchairs watching others working. Little by little, television cuts us off from the real world. We get
so lazy, we choose to spend a fine day in semi-darkness, glued to our sets, rather than go out into the world itself.
Television may be s splendid medium of communication, but it prevents us from communicating with each other.
We only become aware how totally irrelevant television is to real living when we spend a holiday by the sea or in
the mountains, far away from civilization. In quiet, natural surroundings, we quickly discover how little we miss the
hypnotic tyranny of King Telly.

1. What is the biggest harm of TV?


[A] It deprives people of communication with the real world.
[B] People become lazy.
[C] People become dependent on second-hand experience.
[D] TV consumes a large part of ones life.
2. In what way can people forget TV?
[A] Far away from civilization.
[B] To a mountain.
[C] By the sea.
[D] In quiet natural surroundings.
3. What does a mother usually do to keep her children quiet?
[A] Let them watch the set.
[B] Put them in the living room.
[C] Let them watch the rubbish.

37
[D] Let them alone.
4. What does the first sentence in the first paragraph mean?
[A] We found it difficult to occupy our spare time.
[B] We become addicted to TV.
[C] What we used to do is different from now.
[D] We used to enjoy civilized pleasures.

VOCABULARY
1. goggle
goggle box
2. gulp
3. telly
4. pacifier
5. rubbishy
6. sadism
7. glue
glue to the sets
8. hypnotic

ANSWER
1. A 2. D 3. A 4. B

38
Passage 20: The Law to Keep the Oil Industry under Control

The Norwegian Government is doing its best to keep the oil industry under control. A new law limits explo-
ration to an area south of the southern end of the long coastline; production limits have been laid down (though
these have already been raised); and oil companies have not been allowed to employ more than a limited number
of foreign workers. But the oil industry has a way of getting over such problems, and few people believe that the
Government will be able to hold things back for long. As on Norwegian politician said last week: We will soon be
changed beyond all recognition.
Ever since the war, the Government has been carrying out a programme of development in the area north
of the Arctic Circle. During the past few years this programme has had a great deal of success: Tromso has been
built up into a local capital with a university, a large hospital and a healthy industry. But the oil industry has al-
ready started to draw people south, and within a few years the whole northern policy could be in ruins.
The effects of the oil industry would not be limited to the north, however. With nearly 100 percent employ-
ment, everyone can see a situation developing in which the service industries and the tourist industry will lose
more of their workers to the oil industry. Some smaller industries might even disappear altogether when it becomes
cheaper to buy goods from abroad.
The real argument over oil is its threat to the Norwegian way of life. Farmers and fishermen do not make
up most of the population, but they are an important part of it, because Norwegians see in them many of the quali-
ties that they regard with pride as essentially Norwegian. And it is the farmers and the fishermen who are most
critical of the oil industry because of the damage that it might cause to the countryside and to the sea.

1. The Norwegian Government would prefer the oil industry to


[A] provide more jobs for foreign workers.
[B] slow down the rate of its development.
[C] sell the oil it is producing abroad.
[D] develop more quickly than at present.
2. The Norwegian Government has tried to
[A] encourage the oil companies to discover new oil sources.
[B] prevent oil companies employing people from northern Norway.
[C] help the oil companies solve many of their problems.
[D] keep the oil industry to something near its present size.
3. According to the passage, the oil industry might lead northern Norway to
[A] the development of industry.
[B] a growth in population.
[C] the failure of the development programme.
[D] the development of new towns.
4. In the south, one effect to the development of the oil industry might be
[A] a large reduction on unemployment.
[B] a growth in the tourist industry.
[C] a reduction in the number of existing industries.
[D] the development of a number of service industries.
5. Norwegian farmers and fishermen have an important influence because
[A] they form such a large part of Norwegian ideal.
[B] their lives and values represent the Norwegian ideal.

39
[C] their work is so useful to the rest of Norwegian society.
[D] they regard oil as a threat to the Norwegian way of life.

VOCABULARY
1. Norwegian
2. coastline
3. recognition
4. countryside

ANSWER
1. B 2. D 3. C 4. C 5. B

40
:

Passage 1: Violence Can Do Nothing to Diminish Race Prejudice

In some countries where racial prejudice is acute, violence has so come to be taken for granted as a means
of solving differences, that it is not even questioned. There are countries where the white man imposes his rule by
brute force; there are countries where the black man protests by setting fire to cities and by looting and pillaging.
Important people on both sides, who would in other respects appear to be reasonable men, get up and calmly argue
in favor of violence as if it were a legitimate solution, like any other. What is really frightening, what really fills
you with despair, is the realization that when it comes to the crunch, we have made no actual progress at all. We
may wear collars and ties instead of war-paint, but our instincts remain basically unchanged. The whole of the
recorded history of the human race, that tedious documentation of violence, has taught us absolutely nothing. We
have still not learnt that violence never solves a problem but makes it more acute. The sheer horror, the bloodshed,
the suffering mean nothing. No solution ever comes to light the morning after when we dismally contemplate the
smoking ruins and wonder what hit us.
The truly reasonable men who know where the solutions lie are finding it harder and herder to get a hear-
ing. They are despised, mistrusted and even persecuted by their own kind because they advocate such apparently
outrageous things as law enforcement. If half the energy that goes into violent acts were put to good use, if our
efforts were directed at cleaning up the slums and ghettos, at improving living-standards and providing education
and employment for all, we would have gone a long way to arriving at a solution. Our strength is sapped by having
to mop up the mess that violence leaves in its wake. In a well-directed effort, it would not be impossible to fulfill the
ideals of a stable social programme. The benefits that can be derived from constructive solutions are everywhere
apparent in the world around us. Genuine and lasting solutions are always possible, providing we work within the
framework of the law.
Before we can even begin to contemplate peaceful co-existence between the races, we must appreciate each
others problems. And to do this, we must learn about them: it is a simple exercise in communication, in exchang-
ing information. Talk, talk, talk, the advocates of violence say, all you ever do is talk, and we are none the wiser.
Its rather like the story of the famous barrister who painstakingly explained his case to the judge. After listening
to a lengthy argument the judge complained that after all this talk, he was none the wiser. Possible, my lord, the
barrister replied, none the wiser, but surely far better informed. Knowledge is the necessary prerequisite to wis-
dom: the knowledge that violence creates the evils it pretends to solve.

1. What is the best title for this passage?


[A] Advocating Violence.
[B] Violence Can Do Nothing to Diminish Race Prejudice.
[C] Important People on Both Sides See Violence As a Legitimate Solution.
[D] The Instincts of Human Race Are Thirsty for Violence.
2. Recorded history has taught us
[A] violence never solves anything.
[B] nothing.
[C] the bloodshed means nothing.
[D] everything.
3. It can be inferred that truly reasonable men
[A] cant get a hearing.
41
[B] are looked down upon.
[C] are persecuted.
[D] Have difficulty in advocating law enforcement.
4. He was none the wiser means
[A] he was not at all wise in listening.
[B] He was not at all wiser than nothing before.
[C] He gains nothing after listening.
[D] He makes no sense of the argument.
5. According the author the best way to solve race prejudice is
[A] law enforcement.
[B] knowledge.
[C] nonviolence.
[D] Mopping up the violent mess.

VOCABULARY
1. acute
2. loot v.n.
3. pillage v.
4. crunch v.n.
when it comes to the crunch = if/when the decisive moment comes.
5. war-paint
6. come to light = become known
7. sap
I was sapped by months of hospital treatment.
8. mop up
9. wake
in the wake of sth. = come after

ANSWER
1. B 2. B 3. D 4. C 5. A

42
Passage 2: The Tourist Trade Contributes Absolutely Nothing to Increasing Understanding be-
tween Nations

The tourist trade is booming. With all this coming and going, youd expect greater understanding to devel-
op between the nations of the world. Not a bit of it! Superb systems of communication by air, sea and land make it
possible for us to visit each others countries at a moderate cost. What was once the grand tour, reserved for only
the very rich, is now within everybodys grasp? The package tour and chartered flights are not to be sneered at.
Modern travelers enjoy a level of comfort which the lords and ladies on grand tours in the old days couldnt have
dreamed of. But whats the sense of this mass exchange of populations if the nations of the world remain basically
ignorant of each other?
Many tourist organizations are directly responsible for this state of affairs. They deliberately set out to pro-
tect their clients from too much contact with the local population. The modern tourist leads a cosseted, sheltered
life. He lives at international hotels, where he eats his international food and sips his international drink while he
gazes at the natives from a distance. Conducted tours to places of interest are carefully censored. The tourist is
allowed to see only what the organizers want him to see and no more. A strict schedule makes it impossible for the
tourist to wander off on his own; and anyway, language is always a barrier, so he is only too happy to be protected
in this way. At its very worst, this leads to a new and hideous kind of colonization. The summer quarters of the
inhabitants of the cite universitaire: are temporarily reestablished on the island of Corfu. Blackpool is recreated at
Torremolinos where the traveler goes not to eat paella, but fish and chips.
The sad thing about this situation is that it leads to the persistence of national stereotypes. We dont see the
people of other nations as they really are, but as we have been brought up to believe they are. You can test this for
yourself. Take five nationalities, say, French, German, English, American and Italian. Now in your mind, match
them with these five adjectives: musical, amorous, cold, pedantic, native. Far from providing us with any insight
into the national characteristics of the peoples just mentioned, these adjectives actually act as barriers. So when
you set out on your travels, the only characteristics you notice are those which confirm your preconceptions. You
come away with the highly unoriginal and inaccurate impression that, say, Anglo-Saxons are hypocrites of that
Latin peoples shout a lot. You only have to make a few foreign friends to understand how absurd and harmful
national stereotypes are. But how can you make foreign friends when the tourist trade does its best to prevent you?
Carried to an extreme, stereotypes can be positively dangerous. Wild generalizations stir up racial hatred
and blind us to the basic facthow trite it sounds! That all people are human. We are all similar to each other
and at the same time all unique.

1. The best title for this passage is


[A] tourism contributes nothing to increasing understanding between nations.
[B] Tourism is tiresome.
[C] Conducted tour is dull.
[D] tourism really does something to ones country.
2. What is the authors attitude toward tourism?
[A] apprehensive.
[B] negative.
[C] critical.
[D] appreciative.
3. Which word in the following is the best to summarize Latin people shout a lot?
[A] silent.

43
[B] noisy.
[C] lively.
[D] active.
4. The purpose of the authors criticism is to point out
[A] conducted tour is disappointing.
[B] the way of touring should be changed.
[C] when traveling, you notice characteristics which confirm preconception.
[D] national stereotypes should be changed.
5. What is grand tour now?
[A] moderate cost.
[B] local sight-seeing is investigated by the tourist organization.
[C] people enjoy the first-rate comforts.
[D] everybody can enjoy the grand tour.

VOCABULARY
1. superb
2. moderate
3. grand tour
4. package tour package holiday
5. chartered flight
6. set out to do sth. = begin a job with a particular aim /
7. cosset
8. conducted tour = guided tour /
9. censor
10. wander off /
11. quarters
12. paella
13. chip
14. amorous
15. pedantic
16. generalization
17. stir up
18. trite

ANSWER
1. A 2. C 3. B 4. B 5. D

44
Passage 3: Pop Stars Earn Much

Pop stars today enjoy a style of living which was once the prerogative only of Royalty. Wherever they go,
people turn out in their thousands to greet them. The crowds go wild trying to catch a brief glimpse of their smiling,
colorfully dressed idols. The stars are transported in their chauffeur driven Rolls-Royces, private helicopters or
executive aeroplanes. They are surrounded by a permanent entourage of managers, press agents and bodyguards.
Photographs of them appear regularly in the press and all their comings and goings are reported, for, like Royalty,
pop stars are news. If they enjoy many of the privileges of Royalty, they certainly share many of the inconveniences
as well. It is dangerous for them to make unscheduled appearances in public. They must be constantly shielded
from the adoring crowds which idolize them. They are no longer private individuals, but public property. The finan-
cial rewards they receive for this sacrifice cannot be calculated, for their rates of pay are astronomical.
And why not? Society has always rewarded its top entertainers lavishly. The great days of Hollywood have
become legendary: famous stars enjoyed fame, wealth and adulation on an unprecedented scale. By todays stan-
dards, the excesses of Hollywood do not seem quite so spectacular. A single gramophone record nowadays may
earn much more in royalties than the films of the past ever did. The competition for the title Top of the Pops is
fierce, but the rewards are truly colossal.
It is only right that the stars should be paid in this way. Dont the top men in industry earn enormous sala-
ries for the services they perform to their companies and their countries? Pop stars earn vast sums in foreign cur-
rency often more than large industrial concerns and the taxman can only be grateful fro their massive annual
contributions to the exchequer. So who would begrudge them their rewards?
Its all very well for people in humdrum jobs to moan about the successes and rewards of others. People
who make envious remarks should remember that the most famous stars represent only the tip of the iceberg. For
every famous star, there are hundreds of others struggling to earn a living. A man working in a steady job and
looking forward to a pension at the end of it has no right to expect very high rewards. He has chosen security and
peace of mind, so there will always be a limit to what he can earn. But a man who attempts to become a star is
taking enormous risks. He knows at the outset that only a handful of competitors ever get to the very top. He knows
that years of concentrated effort may be rewarded with complete failure. But he knows, too, that the rewards for
success are very high indeed: they are the recompense for the huge risks involved and if he achieves them, he has
certainly earned them. Thats the essence of private enterprise.

1. The sentence Pop stars style of living was once the prerogative only of Royalty means
[A] their life was as luxurious as that of royalty.
[B] They enjoy what once only belonged to the royalty.
[C] They are rather rich.
[D] Their way of living was the same as that of the royalty.
2. What is the authors attitude toward top stars high income?
[A] Approval.
[B] Disapproval.
[C] Ironical.
[D] Critical.
3. It can be inferred from the passage
[A] there exists fierce competition in climbing to the top.
[B] People are blind in idolizing stars.
[C] Successful Pop stars give great entertainment.

45
[D] The tax they have paid are great.
4. What can we learn from the passage?
[A] Successful man should get high-income repayment.
[B] Pop stars made great contribution to a country.
[C] Pop stars can enjoy the life of royalty.
[D] Successful men represent the tip of the iceberg.
5. Which paragraph covers the main idea?
[A] The first.
[B] The second.
[C] The third.
[D] The fourth.

VOCABULARY
1. prerogative
2. chauffeur
3. entourage
4. astronomical
5. adulation
6. gramophone
7. colossal
8. exchequer
Exchequer Bond
9. begrudge /
10. humdrum
11. moan
moan about

ANSWER
1. B 2. A 3. A 4. D 5. D

46
Passage 4: Examinations Exert a Pernicious Influence on Education

We might marvel at the progress made in every field of study, but the methods of testing a persons knowl-
edge and ability remain as primitive as ever they were. It really is extraordinary that after all these years, educa-
tionists have still failed to device anything more efficient and reliable than examinations. For all the pious claim
that examinations text what you know, it is common knowledge that they more often do the exact opposite. They
may be a good means of testing memory, or the knack of working rapidly under extreme pressure, but they can tell
you nothing about a persons true ability and aptitude.
As anxiety-makers, examinations are second to none. That is because so much depends on them. They are
the mark of success of failure in our society. Your whole future may be decided in one fateful day. It doesnt mat-
ter that you werent feeling very well, or that your mother died. Little things like that dont count: the exam goes
on. No one can give of his best when he is in mortal terror, or after a sleepless night, yet this is precisely what the
examination system expects him to do. The moment a child begins school, he enters a world of vicious competition
where success and failure are clearly defined and measured. Can we wonder at the increasing number of drop-
outs: young people who are written off as utter failures before they have even embarked on a career? Can we be
surprised at the suicide rate among students?
A good education should, among other things, train you to think for yourself. The examination system does
anything but that. What has to be learnt is rigidly laid down by a syllabus, so the student is encouraged to memo-
rize. Examinations do not motivate a student to read widely, but to restrict his reading; they do not enable him to
seek more and more knowledge, but induce cramming. They lower the standards of teaching, for they deprive the
teacher of all freedoms. Teachers themselves are often judged by examination results and instead of teaching their
subjects, they are reduced to training their students in exam techniques which they despise. The most successful
candidates are not always the best educated; they are the best trained in the technique of working under duress.
The results on which so much depends are often nothing more than a subjective assessment by some
anonymous examiner. Examiners are only human. They get tired and hungry; they make mistakes. Yet they have to
mark stacks of hastily scrawled scripts in a limited amount of time. They work under the same sort of pressure as
the candidates. And their word carries weight. After a judges decision you have the right of appeal, but not after
an examiners. There must surely be many simpler and more effective ways of assessing a persons true abili-
ties. Is it cynical to suggest that examinations are merely a profitable business for the institutions that run them?
This is what it boils down to in the last analysis. The best comment on the system is this illiterate message recently
scrawled on a wall: I were a teenage drop-out and now I are a teenage millionaire.

1. The main idea of this passage is


[A] examinations exert a pernicious influence on education.
[B] examinations are ineffective.
[C] examinations are profitable for institutions.
[D] examinations are a burden on students.
2. The authors attitude toward examinations is
[A] detest.
[B] approval.
[C] critical.
[D] indifferent.
3. The fate of students is decided by
[A] education.

47
[B] institutions.
[C] examinations.
[D] students themselves.
4. According to the author, the most important of a good education is
[A] to encourage students to read widely.
[B] to train students to think on their own.
[C] to teach students how to tackle exams.
[D] to master his fate.
5. Why does the author mention court?
[A] Give an example.
[B] For comparison.
[C] It shows that teachers evolutions depend on the results of examinations.
[D] It shows the results of court is more effectise.

VOCABULARY
1. pernicious
2. knack
3. embark
4. write off
5. syllabus
6. cram
7. duress
8. stack
9. scrawl /
10. script
11. cynical
12. boil down

ANSWER
1. A 2. C 3. C 4. B 5. B

48
Passage 5: Killing in the Name of God - Ugandan Deaths Spotlight Rise of Cults

How could faith beget such evil? After hundreds of members of a Ugandan cult, the Movement for the Res-
toration of the Ten Commandments of God, died in what first appeared to be a suicidal fire in the village of Kanun-
gu two weeks age, police found 153 bodies buried in a compound used by the cult in Buhunga, 25 miles away.
When investigators searched the house of a cult leader in yet another village, they discovered 155 bodies, many
buried under the concrete floor of the house. Then scores more were dug up at a cult members home. Some had
been poisoned; others, often-young children, strangled. By weeks end, Ugandan police had counted 924 victims
including at least 530 who burned to death inside the sealed church exceeding the 1978 Jonestown mass suicide
and killings by followers of American cult leader Jim Jones that claimed 913 lives.
Authorities believe two of the cults leaders, Joseph Kibwetere, a 68-year-old former Roman Catholic
catechism teacher who started the cult in 1987, and his prophetess, Credonia Mwerinde, by some accounts a
former prostitute who claimed to speak for the Virgin Mary, may still be alive and on the run. The pair had pre-
dicted the world would end on Dec. 31, 1999. When that didnt happen, followers who demanded the return of their
possessions, which they had to surrender on joining the cult, may have been systematically killed.
The Ugandan carnage focuses attention on the proliferation of religious cults in East Africas impoverished
rural areas and city slums. According to the institute for the study of American religion, which researches cults
and sects, there are now more than 5,000 indigenous churches in Africa, some with apocalyptic or revolutionary
leanings. One such group is the Jerusalem Church of Christ in Nairobis Kawangwara slums, led by Mary Snaida-
Akatsa, or mommy as she is known to her thousands of followers. She prophesies about the end of the world and
accuses some members of being witches. One day the brought a special visitor to church, an Indian Sikh man
she claimed was Jesus, and told her followers to repent or pay the consequences.
Most experts say Africas hardships push people to seek hope in religious cults. These groups thrive
because of poverty, says Charles Onyango Obbo, editor of the Monitor, an independent newspaper in Uganda,
and a close observer of cults. People have no support, and theyre susceptible to anyone who is able to tap into
their insecurity. Additionally, they say, AIDS, which has ravaged East Africa, may also breed a fatalism that helps
apocalyptic notions take root.
Some Africans turn to cults after rejecting mainstream Christian churches as Western or non-African.
Agnes Masitsa, 30, who used to attend a Catholic church before she joined the Jerusalem Church of Christ, says of
Catholicism: Its dull.
Catholic icons. Yet, the Ugandan doomsday cult, like many of the sects, drew on features of Roman Ca-
tholicism, a strong force in the region. Catholic icons were prominent in its buildings, and some of its leaders were
defrocked priests, such as Dominic Kataribabo, 32, who reportedly studied theology in the Los Angeles area in
the mid-1980s. He had told neighbors he was digging a pit in his house to install a refrigerator; police have now
recovered 81 bodies from under the floor and 74 from a field nearby. Police are unsure whether Kataribabo died in
the church fire.
Still, there is the question: How could so many killings have been carried out without drawing attention?
Villagers were aware of Kibweteres sect, whose followers communicated mainly through sign language and ap-
parently were apprehensive about violating any of the cults commandments. There were suspicions. Ugandan
president Yoweri Mseveni told the BBC that intelligence reports about the dangerous nature of the group had been
suppressed by some government officials. On Thursday, police arrested an assistant district commissioner, the Rev.
Amooti Mutazindwa, for allegedly holding back a report suggesting the cult posed a security threat.
Now, there are calls for African governments to monitor cults more closely. Says Gilbert Ogutu, a profes-
sor of religious studies at the University of Nairobi: When cult leaders lose support, they become dangerous.

49
1. Why did so may Ugandans die in faith?
[A] Many of them were killed for asking for the return of their possessions.
[B] They found the cults leaders had cheated them.
[C] They lost faith in cults.
[D] They are willing to die.
2. The main reason of peoples joining the cults is
[A] poverty.
[B] insecurity.
[C] AIDS.
[D] fatalism.
3. What does Mary Snaide Akatsa prophesy?
[A] She prophesies the world will be flooded.
[B] She prophesies the world will be in fire.
[C] She prophesies about the end of the world.
[D] She prophesies he followers should die in faith.
4. Why do some Africans reject Christian Churches?
[A] They feel Christianity is dull.
[B] They reject Christian Churches as Western or non-African.
[C] They are susceptible.
[D] They are dangerous persons.
5. How could so many killing have been carried out without drawing attention?
[A] The cult acted secretly.
[B] The government officials did not see through its dangerous nature.
[C] There were no preventive measures.
[D] People were frightened.

VOCABULARY
1. beget
2. cult
3. compound
4. strangle
5. carnage
6. catechism
7. prophet
8. on the run
9. proliferation
10. indigenous
11. apocalypse
12. be susceptible
13. ravage
14. fatalism
15. apocalyptic
16. icon
17. doomsday
18. Jerusalem
19. Kenya

50
20. Nairobi
21. mommy=mother
22. repent
23. Sikh
24. defrock
25. sect
26. pose

ANSWER
1. A 2. A 3. C 4. B 5. A

51
Passage 6: Equality of opportunity in the twentieth Century Has Not Destroyed the Class System

These days we hear a lot of nonsense about the great classless society. The idea that the twentieth century
is the age of the common man has become one of the great clichs of our time. The same old arguments are put
forward in evidence. Here are some of them: monarchy as a system of government has been completely discredited.
The monarchies that survive have been deprived of all political power. Inherited wealth has been savagely reduced
by taxation and, in time, the great fortunes will disappear altogether. In a number of countries the victory has been
complete. The people rule; the great millennium has become a political reality. But has it? Close examination
doesnt bear out the claim.
It is a fallacy to suppose that all men are equal and that society will be leveled out if you provide everybody
with the same educational opportunities. (It is debatable whether you can ever provide everyone with the same
educational opportunities, but that is another question.) The fact is that nature dispenses brains and ability with a
total disregard for the principle of equality. The old rules of the jungle, survival of the fittest, and might is right
are still with us. The spread of education has destroyed the old class system and created a new one. Rewards are
based on merit. For aristocracy read meritocracy; in other respects, society remains unaltered: the class system
is rigidly maintained.
Genuine ability, animal cunning, skill, the knack of seizing opportunities, all bring material rewards. And
what is the first thing people do when they become rich? They use their wealth to secure the best possible opportu-
nities for their children, to give them a good start in life. For all the lip service we pay to the idea of equality, we
do not consider this wrong in the western world. Private schools which offer unfair advantages over state schools
are not banned because one of the principles in a democracy is that people should be free to choose how they will
educate their children. In this way, the new meritocracy can perpetuate itself to a certain extent: an able child from
a wealthy home can succeed far more rapidly than his poorer counterpart. Wealth is also used indiscriminately to
further political ends. It would be almost impossible to become the leader of a democracy without massive financial
backing. Money is as powerful a weapon as ever it was.
In societies wholly dedicated to the principle of social equality, privileged private education is forbidden.
But even here people are rewarded according to their abilities. In fact, so great is the need for skilled workers that
the least able may be neglected. Bright children are carefully and expensively trained to become future rulers. In
the end, all political ideologies boil down to the same thing: class divisions persist whether you are ruled by a feu-
dal king or an educated peasant.

1. What is the main idea of this passage?


[A] Equality of opportunity in the twentieth century has not destroyed the class system.
[B] Equality means money.
[C] There is no such society as classless society.
[D] Nature cant give you a classless society.
2. According to the author, the same educational opportunities cant get rid of inequality because
[A] the principle survival of the fittest exists.
[B] Nature ignores equality in dispensing brains and ability.
[C] Material rewards are for genuine ability.
[D] People have the freedom how to educate their children.
3. Who can obtain more rapid success
[A] those with wealth.
[B] Those with the best brains.

52
[C] Those with the best opportunities.
[D] Those who have the ability to catch at opportunities.
4. Why does the author say the new meritocracy can perpetuate itself to a certain extent? Because
[A] money decides everything.
[B] Private schools offer advantages over state schools.
[C] People are free to choose the way of educating their children.
[D] Wealth is used for political ends.
5. According to the author, class divisions refers to
[A] the rich and the poor.
[B] Different opportunities for people.
[C] Oppressor and the oppressed.
[D] Genius and stupidity.

VOCABULARY
1. discredit
2. monarch
3. millennium
the millennium
4. bear out
5. level out
6. meritocracy
7. knack
8. perpetuate
9. indiscriminate
10. boil down

ANSWER
1. A 2. B 3. A 4. A 5. A

53
Passage 7: The Most Important of All Human Qualities is a Sense of Humor

Biologically, there is only one quality which distinguishes us from animals: the ability to laugh. In a uni-
verse which appears to be utterly devoid of humor, we enjoy this supreme luxury. And it is a luxury, for unlike any
other bodily process, laughter does not seem to serve a biologically useful purpose. In a divide world, laughter is
a unifying force. Human beings oppose each other on a great many issues. Nations may disagree about systems of
government and human relations may be plagued by ideological factions and political camps, but we all share the
ability to laugh. And laughter, in turn, depends on that most complex and subtle of all human qualities: a sense of
humor Certain comic stereotypes have a universal appeal. This can best be seen from the world-wide popularity
of Charlie Chaplins early films. The little man at odds with society never fails to amuse no matter which country
we come from. As that great commentator on human affairs, Dr. Samuel Johnson, once remarked, Men have been
wise in very different modes; but they have always laughed in the same way.
A sense of humor may take various forms and laughter may be anything from a refined tingle to an earth
quaking roar, but the effect is always the same. Humor helps us to maintain a correct sense of values. It is the one
quality which political fanatics appear to lack. If we can see the funny side, we never make the mistake of taking
ourselves too seriously. We are always reminded that tragedy is not really far removed from comedy, so we never
get a lop sided view of things.
This is one of the chief functions of satire and irony. Human pain and suffering are so grim; we hover so of-
ten on the brink of war; political realities are usually enough to plunge us into total despair. In such circumstances,
cartoons and satirical accounts of somber political events redress the balance. They take the wind out of pompous
and arrogant politicians who have lost their sense of proportion. They enable us to see that many of our most pro-
found actions are merely comic or absurd. We laugh when a great satirist like Swift writes about war in Gullivers
Travels. The Lilliputians and their neighbors attack each other because they cant agree which end to break an
egg. We laugh because we meant to laugh; but we are meant to weep too. It is too powerful a weapon to be allowed
to flourish.
The sense of humor must be singled out as mans most important quality because it is associated with
laughter. And laughter, in turn, is associated with happiness. Courage, determination, initiative these are quali-
ties we share with other forms of life. But the sense of humor is uniquely human. If happiness is one of the great
goals of life, then it is the sense of humor that provides the key.

1. The most important of all human qualities is


[A] a sense of humor.
[B] A sense of satire.
[C] A sense of laughter.
[D] A sense of history.
2. The author mentions about Charlie Chaplins early films because
[A] they can amuse people.
[B] Human beings are different from animals.
[C] They show that certain comic stereotypes have a universal appeal.
[D] They show that people have the same ability to laugh.
3. One of the chief functions of irony and satire is
[A] to show absurdity of actions.
[B] to redress balance.
[C] to take the wind out of politicians.

54
[D] to show too much grimness in the world.
4. What do we learn from the sentence it is too powerful a weapon to be allowed to flourish in totalitarian re-
gimes?
[A] It can reveal the truth of political events with satire.
[B] It can arouse people to riot.
[C] It shows tragedy and comedy are related.
[D] It can make people laugh.
5. Who is Swift?
[A] A novelist.
[B] A poet.
[C] A dramatist.
[D] A essayist.

VOCABULARY
1. devoid
2. plague n.v.
3. faction
4. comic stereotype /
5. commentator
6. tinkle n.v.
7. fanatic
8. lop-sided
9. hover
10. somber
11. redress
12. pompous
13. arrogant
14. proportion
15. Lilliputian
16. totalitarian
17. regime

ANSWER
1. A 2. C 3. B 4. A 5. A

55
Passage 8: The Improving Economic Situation In Greece

Greece, economically, is in the black. With very little to export other than such farm products as tobacco,
cotton and fruit, the country earns enough from invisible earnings to pay for its needed, growing imports. From
the sending out of things the Greeks, earn only $285 million; from tourism, shipping and the remittances of Greeks
abroad, the country takes in an additional #375 million and this washes out the almost $400 million by which im-
ports exceed exports.
It has a balanced budget. Although more than one drachma out of four goes for defense, the government
ended a recent year with a slight surplus -- $66 million. Greece has a decent reserve of almost a third of a billion
dollars in gold and foreign exchange. It has a government not dependent on coalescing incompatible parties to
obtain parliamentary majorities.
In thus summarizing a few happy highlights, I dont mean to minimize the vast extent of Greeces problems.
It is the poorest country by a wide margin in Free Europe, and poverty is widespread. At best an annual income of
$60 to $70 is the lot of many a peasant, and substantial unemployment plagues the countryside, cities, and towns of
Greece. There are few natural resources on which to build any substantial industrial base. Some years ago I wrote
here:
Greek statesmanship will have to create an atmosphere in which home and foreign savings will willingly
seek investment opportunities in the back ward economy of Greece. So far, most American and other foreign at-
tempt have bogged down in the Greek governments red tape and shrewdness about small points.
Great strides have been made. As far back as 1956, expanding tourism seemed a logical way to bring need-
ed foreign currencies and additional jobs to Greece. At that time I talked with the Hilton Hotel people, who had
been examining hotel possibilities, and to the Greek government division responsible for this area of the economy.
They were hopelessly deadlocked in almost total differences of opinion and outlook.
Today most of the incredibly varied, beautiful, historical sights of Greece have new, if in many cases mod-
est, tourist facilities. Tourism itself has jumped from approximately $31 million to over $90 million. There is both a
magnificent new Hilton Hotel in Athens and a completely modernized, greatly expanded Grande Bretagne, as well
as other first-rate new hotels. And the advent of jets has made Athens as accessible as Paris or Rome without the
sky-high prices of traffic-choked streets of either.

1. The title below that best expresses the ideas of this passage is
[A] Greek income and expenditures.
[B] The improving economic situation in Greece.
[C] The value of tourism.
[D] Military expenditures.
2. Many peasants earn less than
[A] $60 a week.
[B] $2 a week.
[C] $1 a day.
[D] $10 a month.
3. The Greek Government spends
[A] more than 25%of its budget on military terms.
[B] More than its collects.
[C] A third of a billion dollars in gold.
[D] Less than 25% of its budget on military terms.

56
4. According to the passage, Greece has
[A] a dictatorship.
[B] a monarchy.
[C] a single majority party.
[D] too much red tape.
5. Greece imports annually goods and materials
[A] totaling almost $700 million.
[B] that balance exports.
[C] that are paid by tourists.
[D] costing $66 million.

VOCABULARY
1. remittance
2. wash out
3. drachma
4. lot
5. incompatible
6. coalesce
7. highlight
8. margin
9. bog down
10. red tape
11. shrewdness
12. deadlock

ANSWER
1. B 2. B 3. A 4. C 5. A

57
Passage 9: The Program-Federal Government Helps Minority Business

Federal efforts to aid minority businesses began in the 1960s when the Small Business Administration
(SBA) began making federally guaranteed loans and government-sponsored management and technical assistance
available to minority business enterprises. While this program enabled many minority entrepreneurs to form new
businesses, the results were disappointing, since managerial inexperience, unfavorable locations, and capital
shortages led to high failure rates. Even 15 years after the program was implemented, minority business receipts
were not quite two percent of the national economys total receipts.
Recently federal policymakers have adopted an approach intended to accelerate development of the minor-
ity business sector by moving away from directly aiding small minority enterprises and toward supporting large,
growth-oriented minority firms through intermediary companies. In this approach, large corporations participate
in the development of successful and stable minority businesses by making use of government-sponsored venture
capital. The capital is used by a participating company to establish a Minority Enterprise Small Businesses that
have potential to become future suppliers of customers of the sponsoring company.
MESBICs are the result of the belief that providing established firms with easier access to relevant man-
agement techniques and more job-specific experience, as well as substantial amounts of capital, gives those firms
a greater opportunity to develop sound business foundations than does simply making general management experi-
ence and small amounts of capital available. Further, since potential markets for the minority businesses already
exist through the sponsoring companies, the minority businesses face considerably less risk in terms of location and
market fluctuation. Following early financial and operating problems, sponsoring corporations began to capitalize
MESBICs far above the legal minimum of $500,000 in order to generate sufficient income and to sustain the qual-
ity of management needed. MESBICs are now emerging as increasingly important financing sources for minority
enterprises.
Ironically, MESBIC staffs, which usually consist of Hispanic and Black professionals, tend to approach
investments in minority firms more pragmatically than do many MESBIC directors, who are usually senior manag-
ers from sponsoring corporations. The latter often still think mainly in terms of the social responsibility approach
and thus seem to prefer deals that are riskier and less attractive than normal investment criteria would warrant.
Such differences in viewpoint have produced uneasiness among many minority staff members, who feel that minor-
ity entrepreneurs and businesses should be judged by established business considerations. These staff members
believe their point of view is closer to the original philosophy of MESBICs and they are concerned that, unless
a more prudent course if followed, MESBIC directors may revert to policies likely to re-create the disappointing
results of the original SBA approach.

1. Which of the following best states the central idea of the passage?
[A] The use of MESBICs for aiding minority entrepreneurs seems to have greater potential for success than
does the original SBA approach.
[B] There is a crucial difference in point of view between the staff and directors of some MESBICs.
[C] After initial problems with management and marketing, minority businesses have begun to expand at a
steady rate.
[D] Minority entrepreneurs wishing to form new businesses now have several equally successful federal pro-
grams on which to rely.
2. According to the passage, the MESBIC approach differ s from the SBA approach in that MESBICs
[A] seek federal contracts to provide market for minority businesses.
[B] Encourage minority businesses to provide markets for other minority businesses.

58
[C] Attempt to maintain a specified rate of growth in the minority business sector.
[D] Rely on the participation of large corporations to finance minority businesses.
3. Which of the following statements about the SBA program can be inferred from the passage?
[A] The maximum term for loans made to recipient businesses was 15 years.
[B] Business loans were considered to be more useful to recipient businesses than was management and tech-
nical assistance.
[C] The anticipated failure rate for recipient businesses was significantly lower than the rate that actually
resulted.
[D] Recipient businesses were encouraged to relocate to areas more favorable for business development.
4. The author refers to the financial and operating problems encountered by MESBICs primarily in order to
[A] broaden the scope of the discussion to include the legal considerations of funding MESBICs through
sponsoring companies.
[B] call attention to the fact that MESBICs must receive adequate funding in order to function effectively.
[C] show that sponsoring companies were willing to invest only $500,000 of government-sponsored venture
capital in the original MESBICs.
[D] Compare SBA and MESBIC limits on minimum funding.
5. It can be inferred from the passage that the attitude of some MESBIC staff member toward the investments pre-
ferred by some MESBIC directors can be best described as
[A] disappointing.
[B] Indifferent.
[C] Shocked.
[D] Defensive.

VOCABULARY
1. implement
2. growth-oriented
3. intermediary company
4. venture capital
5. entrepreneur
6. fluctuation
7. pragmatically

ANSWER
1. A 2. D 3. C 4. B 5. A

59
Passage 10: The Importance of Independent Thinking

No one can be a great thinker who does not realize that as a thinker it is her first duty to follow her intellect
to whatever conclusions it may lead. Truth gains more even by the errors of one who with due study and prepara-
tion, thinks for himself, than by the true opinions of those who only hold them because they do not suffer themselves
to think. No that it is solely, of chiefly, to form great thinkers that freedom of thinking is required. One the contrary,
it is as much or even more indispensable to enable average human beings to attain the mental stature which they
are capable of. There have been and many again be great individual thinkers in a general atmosphere of mental
slavery. But there never has been, nor ever will be, in that atmosphere an intellectually active people. Where any
of heterodox speculation was for a time suspended, where there is a tacit convention that principles are not to be
disputed: where the discussion of the greatest questions which can occupy humanity is considered to be closed, we
cannot hope to find that generally high scale of mental activity which has made some periods of history so remark-
able. Never when controversy avoided the subjects which are large and important enough to kindle enthusiasm was
the mind of a people stirred up fro9m its foundation and the impulse given which raised even persons of the most
ordinary intellect to something of the dignity of thinking beings.
She who knows only her own side of the case knows little of that. Her reasons may be food, and no one
may have been able to refute them. But if she s equally unable to refute the reasons of the opposite side; if she does
not so much as know what they are, she has no ground for preferring either opinion. The rational position for her
would be suspension of judgment, and unless she contents herself with that, she is either led by authority, or adopts,
like the generality of the world the side to which she feels the most inclination. Nor is it enough that she should
heat the arguments of adversaries from her own teachers, presented as they state them, and accompanied by what
they offer as refutations, That is not the way to do justice to the arguments, or bring them into real contact with her
own mind. She must be able to hear them form persons who actually believe them; who defend them in earnest, and
do their very utmost for them. She must know them in their most plausible and persuasive form; she must feel the
whole force of the difficulty which the true view of the subject has to encounter and dispose of; else she will never
really possess herself of the portion of truth which meets and removes that difficulty. Ninety-nine in a hundred of
what are called educated persons are in this condition; even of those who can argue fluently for their opinions.
Their conclusion may be true, but it might be false for anything they know; they have never thrown themselves into
the mental position of those who think differently form them and considered what such persons may have to say;
and consequently they do not, in any proper sense of the word, know the doctrines which they themselves profess.

1. The best title for this passage is


[A] The Age of Reason
[B] The need for Independent Thinking
[C] The Value of Reason
[D] Stirring Peoples Minds
2. According to the author, it is always advisable to
[A] have opinions which cannot be refuted.
[B] adopt the point of view to which one feels the most inclination.
[C] be acquainted with the arguments favoring the point of view with which one disagrees,
[D] suspend heterodox speculation in favor of doctrinaire approaches.
3. According to the author, in a great period such as the Renaissance we may expect to find
[A] acceptance of truth
[B] controversy over principles

60
[C] inordinate enthusiasm
[D] a dread of heterodox speculation
4. According to the author, the person who holds orthodox beliefs without examination may be described in all of
the following ways EXCEPT as
[A] enslaved by tradition
[B] less than fully rational
[C] determinded on controversy
[D] having a closed mind
5. It can be inferred from the passage that the author would be most likely to agree with which of the following
statements
[A] A truly great thinker makes no mistakes.
[B] Periods of intellectual achievement are periods of unorthodox reflection,
[C] The refutation of accepted ideas can best be provided by ones own teachers.
[D] excessive controversy prevents clear thinking,

VOCABULARY
1. stature
2. heterodox
3. tacit
4. refute
5. adversary
6. plausible //
7. doctrine
8. profess

ANSWER
1. B 2. C 3. B 4. C 5. B

61
Passage 11: The Affect of Electricity on Cancer

Can electricity cause cancer? In a society that literally runs on electric power, the very idea seems prepos-
terous. But for more than a decade, a growing band of scientists and journalists has pointed to studies that seem to
link exposure to electromagnetic fields with increased risk of leukemia and other malignancies. The implications
are unsettling, to say the least, since everyone comes into contact with such fields, which are generated by every-
thing electrical, from power lines and antennas to personal computers and micro-wave ovens. Because evidence
on the subject is inconclusive and often contradictory, it has been hard to decide whether concern about the health
effects of electricity is legitimateor the worst kind of paranoia.
Now the alarmists have gained some qualified support from the U.S. Environmental Protection Agency. In
the executive summary of a new scientific review, released in draft form late last week, the EPA has put forward
what amounts to the most serious government warning to date. The agency tentatively concludes that scientific
evidence suggests a casual link between extremely low-frequency electromagnetic fieldsthose having very
longwave-lengthsand leukemia, lymphoma and brain cancer, While the report falls short of classifying ELF fields
as probable carcinogens, it does identify the common 60-hertz magnetic field as a possible, but not proven,
cause of cancer in humans.
The report is no reason to panicor even to lost sleep. If there is a cancer risk, it is a small one. The evi-
dence is still so controversial that the draft stirred a great deal of debate within the Bush Administration, and the
EPA released it over strong objections from the Pentagon and the Whit House. But now no one can deny that the
issue must be taken seriously and that much more research is needed.
At the heart of the debate is a simple and well-understood physical phenomenon: When an electric current
passes through a wire, tit generates an electromagnetic field that exerts forces on surrounding objects, For many
years, scientists dismissed any suggestion that such forces might be harmful, primarily because they are so extraor-
dinarily weak. The ELF magnetic field generated by a video terminal measures only a few milligauss, or about
one-hundredth the strength of the earths own magnetic field, The electric fields surrounding a power line can be
as high as 10 kilovolts per meter, but the corresponding field induced in human cells will be only about 1 millivolt
per meter. This is far less than the electric fields that the cells themselves generate.
How could such minuscule forces pose a health danger? The consensus used to be that they could not, and
for decades scientists concentrated on more powerful kinds of radiation, like X-rays, that pack sufficient wallop to
knock electrons out of the molecules that make up the human body. Such ionizing radiations have been clearly
linked to increased cancer risks and there are regulations to control emissions.
But epidemiological studies, which find statistical associations between sets of data, do not prove cause
and effect. Though there is a body of laboratory work showing that exposure to ELF fields can have biological ef-
fects on animal tissues, a mechanism by which those effects could lead to cancerous growths has never been found.
The Pentagon is for from persuaded. In a blistering 33-page critique of the EPA report, Air Force scientists
charge its authors with having biased the entire document toward proving a link. Our reviewers are convinced
that there is no suggestion that (electromagnetic fields) present in the environment induce or promote cancer, the
Air Force concludes. It is astonishing that the EPA would lend its imprimatur on this report. Then Pentagons
concern is understandable. There is hardly a unit of the modern military that does not depend on the heavy use of
some kind of electronic equipment, from huge ground-based radar towers to the defense systems built into every
warship and plane.

1. The main idea of this passage is


[A] studies on the cause of cancer

62
[B] controversial view-points in the cause of cancer
[C] the relationship between electricity and cancer.
[D] different ideas about the effect of electricity on caner.
2. The view-point of the EPA is
[A] there is casual link between electricity and cancer.
[B] electricity really affects cancer.
[C] controversial.
[D]low frequency electromagnetic field is a possible cause of cancer
3. Why did the Pentagon and Whit House object to the release of the report? Because
[A] it may stir a great deal of debate among the Bush Administration.
[B] every unit of the modern military has depended on the heavy use of some kind of electronic equipment.
[C] the Pentagons concern was understandable.
[D] they had different arguments.
4. It can be inferred from physical phenomenon
[A] the force of the electromagnetic field is too weak to be harmful.
[B] the force of the electromagnetic field is weaker than the electric field that the cells generate.
[C] electromagnetic field may affect health.
[D] only more powerful radiation can knock electron out of human body.
5. What do you think ordinary citizens may do after reading the different arguments?
[A]They are indifferent.
[B] They are worried very much.
[C] The may exercise prudent avoidance.
[D] They are shocked.

VOCABULARY
1. preposterous
2. leukemia
3. malignancy
4. legitimate
5. paranoia
6. lymphoma
7. carcinogen
8. minuscule
9. consensus
10. wallop
11. epidemiological
12. blistering
13. critique
14. imprimatur

ANSWER
1. D 2. A 3. B 4. A 5. C

63
Passage 12: Religion and Rationality

Yet the difference in tome and language must strike us, so soon as it is philosophy that speaks: that change
should remind us that even if the function of religion and that of reason coincide, this function is performed in the
two cases by very different organs. Religions are many, reason one. Religion consists of conscious ideas, hopes,
enthusiasms, and objects of worship; it operates by grace and flourishes by prayer. Reason, on the other hand, is
a mere principle or potential order, on which indeed we may come to reflect but which exists in us ideally only,
without variation or stress of any kind. We conform or do not conform to it; it does not urge or chide us, not call
for any emotions on our part other than those naturally aroused by the various objects which it unfolds in their true
nature and proportion. Religion brings some order into life by weighting it with new materials. Reason adds to the
natural materials only the perfect order which it introduces into them. Rationality is nothing but a form, an ideal
constitution which experience may more or less embody. Religion is a part of experience itself, a mass of sentiments
and ideas. The one is an inviolate principle, the other a changing and struggling force. And yet this struggling and
changing force of religion seems to direct man toward something eternal. It seems to make for an ultimate harmony
within the soul and for an ultimate harmony between the soul and all that the soul depends upon. Religion, in its in-
tent, is a more conscious and direct pursuit of the Life of Reason than is society, science, or art, for these approach
and fill out the ideal life tentatively and piecemeal, hardly regarding the foal or caring for the ultimate justification
of the instinctive aims. Religion also has an instinctive and blind side and bubbles up in all manner of chance prac-
tices and intuitions; soon, however, it feels its way toward the heart of things, and from whatever quarter it may
come, veers in the direction of the ultimate.
Nevertheless, we must confess that this religious pursuit of the Life of Reason has been singularly abortive.
Those within the pale of each religion may prevail upon themselves, to express satisfaction with its results, thanks
to a fond partiality in reading the past and generous draughts of hope for the future; but any one regarding the
various religions at once and comparing their achievements with what reason requires, must feel how terrible is
the disappointment which they have one and all prepared for mankind. Their chief anxiety has been to offer imagi-
nary remedies for mortal ills, some of which are incurable essentially, while others might have been really cured by
well-directed effort. The Greed oracles, for instance, pretended to heal out natural ignorance, which has its appro-
priate though difficult cure, while the Christian vision of heaven pretended to be an antidote to our natural death
- the inevitable correlate of birth and of a changing and conditioned existence. By methods of this sort little can be
done for the real betterment of life. To confuse intelligence and dislocate sentiment by gratuitous fictions is a short-
sighted way of pursuing happiness. Nature is soon avenged. An unhealthy exaltation and a one-sided morality have
to be followed by regrettable reactions. When these come. The real rewards of life may seem vain to a relaxed vital-
ity, and the very name of virtue may irritate young spirits untrained in and natural excellence. Thus religion too
often debauches the morality it comes to sanction and impedes the science it ought to fulfill.
What is the secret of this ineptitude? Why does religion, so near to rationality in its purpose, fall so short of
it in its results? The answer is easy; religion pursues rationality through the imagination. When it explains events
or assigns causes, it is an imaginative substitute for science. When it gives precepts, insinuates ideals, or remoulds
aspiration, it is an imaginative substitute for wisdomI mean for the deliberate and impartial pursuit of all food.
The condition and the aims of life are both represented in religion poetically, but this poetry tends to arrogate to
itself literal truth and moral authority, neither of which it possesses. Hence the depth and importance of religion
becomes intelligible no less than its contradictions and practical disasters. Its object is the same as that of reason,
but its method is to proceed by intuition and by unchecked poetical conceits.

1. As used in the passage, the author would define wisdom as

64
[A] the pursuit of rationality through imagination.
[B] an unemotional search for the truth.
[C] a purposeful and unbiased quest for what is best.
[D] a short-sighted way of pursuing happiness
2. Which of the following statements is NOT TRUE?
[A] Religion seeks the truth through imagination, reason, in its search, utilizes the emotions.
[B] Religion has proved an ineffective tool in solving mans problems.
[C] Science seeks a piece meal solution to mans questions.
[D] The functions of philosophy and reason are the same.
3. According to the author, science differs from religion in that
[A] it is unaware of ultimate goals.
[B] it is unimaginative.
[C] its findings are exact and final.
[D] it resembles society and art.
4. The author states that religion differs from rationality in that
[A] it relies on intuition rather than reasoning .
[B] it is not concerned with the ultimate justification of its instinctive aims.
[C] it has disappointed mankind.
[D] it has inspired mankind.
5. According to the author, the pursuit of religion has proved to be
[A] imaginative.
[B] a provider of hope for the future.
[C] a highly intellectual activity
[D] ineffectual.

VOCABULARY
1. grace
2. chide
3. sentiment
4. inviolate
5. intent
6. piecemeal
7. bubble up
8. veer
9. abortive
10. pale
11. draught
12. oracle
13. antidote
14. correlate
15. dislocate
16. gratuitous
17. debauch
18. sanction
19. impede
20. ineptitude
21. insinuate

65
22. remould
23. aspiration
24. arrogate to
25. literal
26. intelligible
27. conceit

ANSWER
1. C 2. A 3. A 4. D 5. D

66
Passage 13: Cryptic Coloring

Cryptic coloring is by far the commonest use of color in the struggle for existence. It is employed for the
purpose of attack (aggressive resemblance or anticryptic coloring ) as well as of defense (protective resemblance
or procryptic coloring ). The fact that the same method concealment, may be used both for attack and defense has
been well explained by T.Belt who suggests as an illustration the rapidity of movement which is also made use of by
both pursuer and pursued, which is similarly raised to a maximum in both by the gradual dying out of the slowest
through a series of generations. Cryptic coloring is commonly associated with other aids in the struggle for life.
Thus well-concealed mammals and birds, when discovered, will generally endeavor to escape by speed and will of-
ten attempt to defend themselves actively. On the other hand, small animals which have no means of active defense,
such as large, numbers of insects, frequently depend upon concealment alone. Protective resemblance is far com-
moner among animals than aggressive resemblance, in correspondence with the fact that predaceous forms are as
a rule much larger and much less numerous than their prey. In the case of insectivorous Vertebrata and their prey
such differences exist in an exaggerated form. Cryptic coloring, whether used for defense of attack, may be either
general or special. In general resemblance the animal, in consequence of its coloring, produces the same effect
as its environment, but the conditions do not require any special adaptation of shape and outline. General resem-
blance is especially common among the animal inhabiting some uniformly colored expanse of the earths surface,
such as an ocean or a desert. In the former, animals of all shapes are frequently protected by their transparent blue
color, on the latter, equally diverse forms are defended by their sandy appearance. The effect of a uniform appear-
ance may be produced by a combination of tints in startling contrast. Thus the black and white stripes of the zebra
blend together at a little distance, and their proportion is such as exactly to match the pale tint which arid ground
possesses when seen by moonlight. Special resemblance is far commoner than general and is the form which is
usually met with on the diversified surface of the earth, on the shores, and in shallow water, as well as on the float-
ing masses of algae on the surface of the ocean, such as the Sargasso Sea. In these environments the cryptic color-
ing of animals is usually aided by special modifications of shape, and by the instinct which leads them to assume
particular attitudes. Complete stillness and the assumption of a certain attitude play an essential part in general
resemblance on land; but in special resemblance the attitude is often highly specialized, and perhaps more impor-
tant than any other element in the complex method by which concealment is effected. In special resemblance the
combination of coloring, shape, and attitude is such as to produce a more or less exact resemblance to some one of
the objects in the environment, such as a leaf of twig, a patch of lichen, a flake of bark. In all cases the resemblance
is to some object which is of no interest to the enemy or prey respectively. The animal is not hidden from view by
becoming indistinguishable from its background as in the case of general resemblance, but it is mistaken for some
well-know object.
In seeking the interpretation of these most interesting and elaborate adaptations, attempts have been made
along two lines. The first seeks to explain the effect as a result of the direct influence of the environment upon the
individual (G.L.L.Buffon), or by the inherited effects of efforts and the use and disuse of parts (J.B.P.Lamarck). The
second believes that natural selection produced the result and afterwards maintained it by the survival of the best
concealed in each generation. The former suggestion breaks down when the complex nature of numerous special
resemblances is appreciated. Thus the arrangement of colors of many kinds into an appropriate pattern requires
the cooperation of a suitable shape and the rigidly exact adoption of a certain elaborate attitude. The latter is
instinctive and thus depends on the central nervous system. The cryptic effect is due to the exact cooperation of all
these factors; and in the present state of science, the only possible hole of an interpretation lies in the theory of
natural selection, which can accumulate any and every variation which tends toward survival. A few of the chief
types of methods by which concealment is effected may be briefly described. The colors of large numbers of verte-

67
brate animals are darkest on the back and become gradually lighter on the sides, passing into white on the belly.
Abbot H. Thayer has suggested that this gradation obliterates the appearance of solidity, which is due to shadow.
The color harmony, which is also essential to concealment, is produced because the back is of the same tint as the
environment (e. g. earth), bathed in the cold blue-white of the sky, while the belly, being cold blue-white and bathed
in shadow and yellow earth reflections produces the same effects. This method of neutralizing shadow for the
purpose of concealment by increased lightness of tint was first suggested by E.B.Poulton in the case of a larva and
a pupa, but he did not appreciate the great importance of the principle. In an analogous method an animal in front
of a background of dark shadow may have part of its body obliterated by the existence of a dark tint, the remainder
resembling, e.g., a part of a leaf. This method of rendering invisible any part which would interfere with the resem-
blance is well know in mimicry.

1. The black and white stripes of the zebra are most useful form
[A] hunters.
[B] nocturnal predators
[C] lions and tigers.
[D] insectivorous Vertrbrata
2. Aggressive resemblance occurs when
[A] a predaceous attitude is assumed.
[B] special resemblance is utilized.
[C] an animal relies on speed.
[D] an animal blends in with its background.
3. Special resemblance differs from general resemblance in that the animal relies on
[A] its ability to frighten its adversary.
[B] speed.
[C] its ability to assume an attitude.
[D] mistaken identify
4. The title below that best expresses the ides of this passage is
[A] Cryptic coloration for Protection.
[B] How Animals Survive.
[C] The uses of Mimicry in Nature.
[D] Resemblances of Animals.
5. Of the following which is the least common?
[A] protective resemblance.
[B] General resemblance.
[C] Aggressive resemblance.
[D] Special resemblance.

VOCABULARY
1. cryptic
cryptic coloring
2. predaceous
3. vertebrate
4. tint
5. zebra
6. Sargasso

68
7. twig
8. lichen
9. flake
10. gradation
11. obliterate
12. larva
13. pupa
14. mimicry
15. nocturnal
16. insectivorous
17. procryptic

ANSWER
1. B 2. A 3. D 4. C 5. C

69
Passage 14: Pageants

Pageants are usually conceived on a fairly large scale, often under the auspices of some local or civic
authority or at any rate in connection with local groups of some kind. This sometimes means that there is an al-
location of funds available for the purpose of mounting the production, though unfortunately this will usually be
found to be on the meager side and much ingenuity will have to be used to stretch it so that all performers can be
adequately clothed.
Most pageants have a historical flavour as they usually come about through the celebration of the anniver-
sary of some event of historic importance, or the life or death of some local worthy. Research among archives and
books in the public library will probably prove very useful and produce some workable ideas which will give the
production an especially local flavour. From the first economy will have to be practiced because there are usually
a great number of people to dress. Leading characters can be considered individually in the same way as when
designing for a play; but the main body of the performers will need to be planned in groups and the massed effect
must be always borne in mind.
Many pageants take place in daylight in the open air. This is an entirely different problem from designing
costumes which are going to be looked at under artificial lighting; for one thing, scenes viewed in the daylight are
subject to many more distractions. No longer is everything around cut out by the surrounding darkness, but instead
it is very easy to be aware of disturbing movement in the audience of behind the performers. Very theatrically
conceived clothes do not always look their best when seen in a daylight setting of trees, verdant lawns and old ivy-
covered walls; the same goes for costumes being worn in front of the mellow colors of stately homes. The location
needs to be studied and then a decision can be made as to what kinds of colors and textures will harmonize best
with the surroundings and conditions and then to carry this out as far as possible on the funds available.
If money is available to dress the performers without recourse to their own help in the provision of items,
it is best to arrange for all the cutting and pinning together of the costumes to be done by one or two experienced
people than to be given out to the groups and individuals for completion. When there is little or no money at all, the
garments need to be reduced to the basic necessities. Cloaks and shawls become invaluable, sheets and large bath
towels and bath sheets are admirable for draping. Unwanted curtains and bed spreads can be cut to make tunics,
robes and skirts. These are particularly valuable if they are of heavy fabrics such as velvet or chenille.
Colors should be massed together so that there are contrasting groups of dark and light, this will be found
to help the visual result substantially. Crowds of people gathered together in a jumble of colors will be ground to
look quite purposeless and will lack dramatic impact.
The use of numbers of identical head-dresses, however simply made, are always effective when working
with groups. If these are made of cardboard and painted boldly the cost can be almost negligible. Helmets, hats
and plumes will all make quite a show even if the costumes are only blandest or sheets cleverly draped. The same
can be said of the use of banners, shields and poles with stiff pennants and garlandsanything which will help to
have a unifying effect. Any kind of eye-catching device will always go with a flourish and add excitement to the
scenes.

1. The main idea of this passage is


[A] Pageants.
[B] Costumes on the stage.
[C] Costumes for pageants.
[D] How to arrange a pageant.
2. It can be inferred that the most important factor in costume design is

70
[A] money.
[B] color.
[C] harmony
[D] texture
3. Why will much ingenuity have to be required in costume design?
[A] Because pageants take place in daylight in the open air.
[B] Because different characters require different costumes.
[C] Because the colors and textures must be in harmony with the setting.
[D] Because an allocation of the funds available is usually rather small.
4. Why do most pageants have a historical flavour?
[A] Because most pageants take place for celebration.
[B] Many pageants take place for amusement.
[C] A lot of pageants take place for religion.
[D] Because pageants usually take place for competition.

VOCABULARY
1. conceive
to be conceived
2. under the auspices of
3. meager
4. mount
5. flavour
6. archive(s)
7. distraction
8. ivy
9. mellow
10. recourse (to)
11. drape
12. invaluable
13. tunic
14. chenille
15. jumble
16. pennant
17. garland

ANSWER
1. C 2. C 3. D 4. A

71
Passage 15: The Second Wave of Feminism

The statistics Ive cited and the living examples are all too familiar to you. But what may not be so familiar
will be the increasing number of women who are looking actively for advancement of for a new job in your offices.
This woman may be equipped with professional skills and perhaps valuable experience, She will not be content to
be Executive Assistant to Mr. Seldom Seen of the Assistant Vice Presidents Girl Friday, who is the only one who
comes in on Saturday.
She is the symbol of what I call the Second Wave of Feminism. She is the modern woman who is determined
to be.
Her forerunner was the radical feminist who interpreted her trapped position as a female as oppression by
the master class of men. Men, she believed, had created a domestic, servile role for women in order that men could
have the career and the opportunity to participate in making the great decisions of society. Thus the radical femi-
nist held that women through history had been oppressed and dehumanized, mainly because man chose to exploit
his wife and the mother of his children. Sometimes it was deliberate exploitation and sometimes it was the inno-
cence of never looking beneath the pretensions of life.
The radical feminists found strength in banding together. Coming to recognize each other for the first time,
they could explore their own identities, realize their own power, and view the male and his system as the com-
mon enemy. The first phases of feminism in the last five years often took on this militant, class-warfare tone. Betty
Friedan, Gloria Steinem, Germaine Greer, and many others hammered home their ideas with a persistence that
aroused and intrigued many of the brightest and most able women in the country. Consciousness-raising groups
allowed women to explore both their identities and their dreamsand the two were often found in direct conflict.
What is the stereotyped role of American women? Marriage. A son. Two daughters. Breakfast. Ironing.
Lunch. Bowling, maybe a garden club of for the very daring, non-credit courses in ceramics. Perhaps an occasion-
al cocktail party. Dinner. Football or baseball on TV. Each day the same. Never any growth in expectationsun-
less it is growth because the husband has succeeded. The inevitable question: Is that all there is to life?
The rapid growth of many feminist organizations attests to the fact that these radical feminists had touched
some vital nerves. The magazine Ms. was born in the year of the death of the magazine Life. But too often the
consciousness-raising sessions became ends in themselves. Too often sexism reversed itself and man-hating was
encouraged. Many had been with the male chauvinist.
It is not difficult, therefore, to detect a trend toward moderation. Consciousness-raising increasingly is
regarded as a means to independence and fulfillment, rather than a ceremony of fulfillment itself. Genuine indepen-
dence can be realized through competence, through finding a career, through the use of education. Remember that
for many decades the education of women was not supposed to be useful.

1. What was the main idea of this passage?


[A] The Second Wave of Feminist.
[B] Womens Independent Spirits.
[C] The Unity of Women.
[D] The Action of Union.
2. What was the authors attitude toward the radical?
[A] He supported it wholeheartedly.
[B] He opposed it strongly.
[C] He disapproved to some extent.
[D] He ignored it completely.

72
3. What does the word militant mean?
[A] Aggressive.
[B] Ambitions.
[C] Progressive.
[D] Independent.
4. What was the radical feminists view point about the male?
[A] Women were exploited by the male.
[B] Women were independent of the male.
[C] Womens lives were deprived by the male.
[D] The male were their common enemy.

VOCABULARY
1. trapped
2. servile
3. dehumanized /
4. pretension
5. look beneath
6. band together
7. explore
8. phase
9. class-warfare
10. hammer home
hammer home an idea into sb.
11. intrigue
12. stereotype
13. attest
14. chauvinist
15. moderation
16. fulfillment
17. sexism
18. be independent of /

ANSWER
1. A 2. C 3. A 4. D

73
Passage 16: Crime in Computer

New and bizarre crimes have come into being with the advent of computer technology. Organized crime
to has been directly involved; the new technology offers it unlimited opportunities, such as data crimes, theft of
services, property-related crimes, industrial sabotage, politically related sabotage, vandalism, crimes against the
individual and financially related crimes
Theft of data, or data crime, has attracted the interest of organized criminal syndicates. This is usually the
theft or copying of valuable computer grogram. An international market already exists for computerized data, and
specialized fences are said to be playing a key role in this rapidly expanding criminal market. Buyers for stolen
programs may range from a firms competitors to foreign nations.
A competitor sabotages a companys computer system to destroy or cripple the firms operational ability,
thus neutralizing its competitive capability either in the private or the government sector. This computer sabotage
may also be tied to an attempt by affluent investors to acquire the victim firm. With the growing reliance by firms
on computers for their recordkeeping and daily operations, sabotage of their computers can result in internal hav-
oc, after which the group interested in acquiring the firm can easily buy it at a substantially lower price. Criminal
groups could also resort to sabotage if the company is a competitor of a business owned or controlled by organized
crime.
Politically motivated sabotage is on the increase; political extremist groups have sprouted on every conti-
nent. Sophisticated computer technology arms these groups with awesome powers and opens technologically ad-
vanced nations to their attack. Several attempts have already been made to destroy computer facility at an air force
base. A university computer facility involved in national defence work suffered more than $2 million in damages as
a result of a bombing.
Computer vulnerability has been amply documented. One congressional study concluded that neither gov-
ernment nor private computer systems are adequately protected against sabotage. Organized criminal syndicates
have shown their willingness to work with politically motivated groups. Investigators have uncovered evidence of
cooperation between criminal groups and foreign governments in narcotics. Criminal groups have taken attempts
in assassinating political leaders. Computers are used in hospital life-support system, in laboratories, and in
major surgery. Criminals could easily turn these computers into tools of devastation. By sabotaging the computer
of a life-support system, criminals could kill an individual as easily as they had used a gun. By manipulating a
computer, they could guide awesome tools of terror against large urban centers. Cities and nations could become
hostages. Homicide could take a now form. The computer may become the hit man of the twentieth century.
The computer opens vast areas of crime to organized criminal groups, both national and international.
It calls on them to pool their resources and increase their cooperative efforts, because many of these crimes are
too complex for one group to handle, especially those requiting a vast network of fences. Although criminals have
adapted to computer technology, law enforcement has not. Many still think in terms of traditional criminology.

1. How many kinds of crimes are mentioned in the passage?


[A] 7
[B] 8
[C] 9
[D] 10
2. What is the purpose of a competitor to sabotage a companys computer?
[A] His purpose is to destroy or weaken the firms operational ability.
[B] His purpose is to weaken firms competitive capability and get it.

74
[C] His purpose is to buy the rivals company at a relatively low price.
[D] His purpose is to steal important data.
3. Which of the following can be labeled as a politically motivated sabotage of a computer system?
[A] Sabotage of a university computer.
[B] Sabotage of a hospital computer.
[C] Sabotage of computer at a secret training base.
[D] Sabotage of a factory computer.
4. What does the author mean by Homicide could take a new form?
[A] There is no need to use a gun in killing a person.
[B] Criminals can kill whoever they want by a computer.
[C] The computer can replace any weapons.
[D] The function of a computer is just like a gun.

VOCABULARY
1. bizarre
2. vandalism
3. cripple
4. fence
5. neutralize
6. affluent
7. recordkeeping
8. havoc
9. resort to
10. motivate
11. extremist
12. sprout
13. awesome
14. vulnerability
15. devastation
16. hospital life-support system
17. hit man
18. pool
19. criminology

ANSWER
1. B 2. B 3. C 4. B

75
Passage 17: A Strong Stock Market

The increase in the margin rate from 50% to 70% was not an attempt to stem any rampant speculation on
the part of the public, actually the market seemed technically quite strong, with public participation essentially dig-
nified, but rather an attempt by the Federal Reserve Board to preserve the sound underpinnings that existed in the
market. Naturally, such a move had a momentarily chilling effect upon prices but if the FRB had been preoccupied
with undue speculation, the increase might have been to the 80% or even 90% level. Such an increases in the mar-
gin rate is a confirmation of a strong stock market and since 19,such increases have resulted in interim market
highs over twelve months later. Obviously, there could be no guarantee that this would once again be the case, but
if history is any guideline, and if business and corporate earnings were to continue on the same course, continued
optimism over the outlook for the stock market would seem more prudent than pessimism.
The margin increase underscored the good rise that stocks had enjoyed for the previous year, and the fact
that a 50% rate was maintained as long as it was pointed up the fact that the rise was mainly conservative in that
it was concentrated in the blue chips for the most part. In past Investment Letters we have voiced the thought that
speciality stocks could outperform the general market from this point. We continue to believe that this could be the
case. For example, steel stocks tend to sell at certain fixed price/earnings ratios. Below a certain ratio they are
considered good value, above a certain ratio, overpriced. If a company produces a unique product it is far more
difficult for market analysis to place a numerical ratio upon the companys earnings. We have also contended in
the past Letters that the stock market reflects mass psychology as well as the business outlook. When investors,
both the public and the institutions, are nervous and pessimistic they definitely hesitate to buy stocks: they seek
low price/earnings multiples and high yields. These same investors, when they are in an optimistic frame of mind,
become for less preoccupied with yields and more wiling to pay a premium(high p/e multiples) for accelerated
growth. If the publics attitude towards the auto industry is any measure, then this period seems to have been one of
optimism.

1. The title that best expresses the ideas of this passage is


[A] A Time to Sell Stock.
[B] A Strong Stock Market
[C] Raising the Margin Rate
[D] Price/earnings Ratio in Steel
2. When investors are pessimistic what do they do?
[A] They look to the FRB for help.
[B] They buy steel
[C] They buy automobile stocks.
[D] They look for high yields.
3. Why does the writer believe that speciality stocks could outperform the general market?
[A] Because analysis have difficulty in deciding upon a fixed price/earnings ratio.
[B] Because the activity had been limited to blue chips.
[C] Because the rise was conservative.
[D] Because of the FRB action.
4. When investors are optimistic, what do they do?
[A] They look for accelerated growth.
[B] They buy speciality stocks.
[C] They look for high yields.

76
[D] They are more prudent.

VOCABULARY
1. margin rate
2. rampant
3. stem
4. stem from
5. underpin
6. underpinning
7. preoccupy
8. undue
9. interim
10. guideline
11. underscore
12. point up
13. bluechip
14. blue-chip
15. outperform
16. overprice
17. numerical ratio
18. earnings
19. contend
20. premium

ANSWER
1. B 2. D 3. A 4. A

77
Passage 18: The Military Is In

Things have really changed. Not only is the military standing tall again, it is staging a remarkable come-
back in the quantity and quality of the recruits it is attracting. Recruiters, once denounced by antiwar students as
baby killers and barred from campuses, are welcomed ever at elite universities. ROTC (Reserve Officers Train-
ing Corps) programs, that faltered during the Viet Nam era, when protesters were fire bombing their headquarters,
are flourishing again. The military academies are enjoying a steady increase in applications.
Certainly, the depressed economy has increased the allure of the jobs, technical training and generous
student loans offered by the military. Students know that if they go in and become, say, nuclear weapons special-
ists, they can come out and demand a salary of $60,000 a year. Military salaries, while not always competitive
with those paid for comparable jobs in the private sector, are more than respectable, especially considering the
wide array of benefits that are available: free medical service, room and board, and PX (Post Exchange) privi-
leges. Monthly pay for a recruit is $574; for a sergeant with four years services it is $906; for a major with ten
years service it is $2,305. The services slick $175 million-a-year advertising campaign promising adventure and
fulfillment has helped win over the TV generation. Kids are walking down the school hallways chanting Army,
Navy, Air Force, Marines, just like in the commercials. And many military officials feel that the key difference is
the enhanced patriotism among the nations youth. There is a return to the view that the military is an honorable
profession. The days of a judge telling a miscreant to join the Army or go to jail are over. Recruiting for all four
services combined is running at 101%of authorized goals. And the retention rate is now so high, that the services
are refusing some re-enlistment applications and reducing annual recruiting target.
The military academics are also enjoying halcyon years, attracting more and better-qualified students.
Compared to private colleges, where tuition and expenses have been climbing sharply, the service schools are a
real bargain: not only is tuition free, but recruits get allowances of up to $500 a mouth. It is reported 12,300 appli-
cants are for the 1,450 positions in this years freshman class. Military academies are now just as selective as any
of the best universities in the country.
Nationwide, ROTC enrollment exceeds 105,000,a 64% increase over the 1974 figure. In the mid 70s, the
ROTC students refused to wear their uniforms on campus because they suffered all sorts of ridicule, if they did.
Now if they wear them to class no one looks at them twice. To them, Viet Nam is ancient history, something the old
folks talk about.

1. What is the main idea of this passage?


[A] The Military is in
[B] The Military is up
[C] The Military is down
[D] The Military is on
2. What was the attitude of the students in 1970s towards the military?
[A] Approval.
[B] Indifferent.
[C] Distaste.
[D] Scolding.
3. The phrase come out is closest in meaning to
[A] become visible.
[B] begin to grow.
[C] be made public.

78
[D] gain a certain position.
4. Which one of the following is NOT mentioned as a reason to attract students.
[A] Free tuition.
[B] Spacious room.
[C] Considerate allowance.
[D] Technical training.

VOCABULARY
1. stage a comeback
2. standing tall
3. babykiller
4. denounce
5. elite
6. ROTC=Reserve Offices Training Cope ()
7. falter
8. flourish
9. allure
10. come out
11. the wide array
12. PX=Post Exchange
13. sergeant
14. major
15. slick
16. hallway
17. chant
18. miscreant
19. retention rate
20. real bargain

ANSWER
1. A 2. C 3. D 4. B

79
Passage 19: Creative Process of Works

The great question that this paper will, but feebly, attempt to answer is , what is the creative process?
Though much theory has accumulated, little is really known about the power that lies at the bottom of
poetic creation. It is true that great poets and artists produce beauty by employing all the powers of personality and
by fusing emotions, reason, and intuitions. But what is the magical synthesis that joins and arranges these complex
parts into poetic unity?
John L.Lowes, in his justly famous The Road to Xanadu, developed one of the earliest and still generally
acceptable answers to this tantalizing question. Imaginative creation, he concludes, is a complex process in which
the conscious and unconscious minds jointly operate. There isthe deep well with its chaos of fortuitously blend-
ing images; but there is likewise the Vision which sees shining in and through the chaos of the potential lines of
Form, and with the Vision, the controlling Will. Which gives to that potential beauty actuality.
The Deep Well is the unconscious mind that is peopled with the facts, ideas, feelings of the conscious
activity. The imaginative vision, an unconscious activity, shines through the land of chaos, of lights and shadows,
silently seeking pattern and form. Finally, the conscious mind again, through Will, captures and embodies the idea
in the final work of art. In this way is unity born out of chaos.
Though there can be no absolute certainty, there is general agreement that the periods in the development
of a creative work parallel, to some extent, Lowes theory of Well, Vision, Form, and Will. There are at least three
stages in the creative process: preparation, inspiration, work.
In a sense, the period of preparation is all of the writers life. It is the Deep Well. It is the Deep Well. It is
especially a period of concentration which gives the unconscious mind an opportunity to communicate with the
conscious mind. When remembrance of things past reach the conscious level of the writers mind, he is ready to
go on with the process. Part of this preparation involves learning a mediumlearning a language, learning how
to write, learning literary forms. It is important to mot here that form cannot be imposed upon the idea. Evidence,
though sparse, shows that the idea gives birth to the form that can best convey it. It is the Vision, according to
Lowes, which sees shining in and through the chaws of the potential lines of from

1. When remembrance of things past reach the conscious level, the post has reached the stage called
[A] Well.
[B] Vision.
[C] Form.
[D] Will.
2. Which of the following statements is TRUE?
[A] The form determines the subject matter.
[B] The idea determines the form.
[C] Vision makes beauty an actuality.
[D] A writer is unconscious when he prepares his work.
3. The word fortuitously in the third paragraph means
[A] accidentally.
[B] luckily.
[C] thoroughly.
[D] potentially.
4. The remembrance of things past is carried on in the
[A] Deep Well.

80
[B] Vision.
[C] Chaotic lights and shadows.
[D] Conscious mind.

VOCABULARY
1. fuse
2. intuition
3. Xanadu
Sanul TColeridge
Kubla Khan
4. John L. Lowes 18761945
5. tantalizing
6. fortuitous
7. parallel
8. sparse
9. synthesis

ANSWER
1. D 2. B 3. A 4. A

81
Passage 20: NCB in Interpol

The organization known to the world as Interpol has sometimes been described as an outfit of chisel-jawed
gimlet-eyed crime fighters who put their lives in jeopardy every working hour. Less flatteringly, Interpol has also
been described as a huge filing cabinet, stuffed with clerks choking on their own statistics.
As with most generalities, there is some truth in both statements. There are, certainly, some grim battlers of
crime to be found working with Interpol. There are, just as surely, those drones shuffling mountains of paper whose
cheeks are sallow from indoor life. Consider the charisma of the name alone: INTERPOL, the international police
force. Continents leaped in a single bound, oceans crossed in the space of a breath, villains watched by eyes that
never sleep. Surprisingly, a lot of it happens almost that way.
Four groups coordinate and direct the activities of Interpol. One of them is the National Central Bureaus,
or NCBs, bodies designated by the member nations to serve as their link with Interpol. These are the front-line
troops, the action people. IN the United States, the Treasury Department is the National Central Bureau. In the
United Kingdom, it is Scotland Yard; the Questura in Italy and the Melbourne City Police in Australia. Because po-
lice organization varies from country, the NCBs were established to act as the one special group to handle Interpol
chores and unsure maximum cooperation between nations. Each NCB is usually an official government body with
police powers if a country has only one central police authority, that body becomes the National Centre Bureau. Of
course, any service appointed as an NCB is bound to its nations law and authority and retains its national title.
Each NCB is connected by radio to the regional station for its geographic zone. The regional stations are
connected to the Central Station in France. The radio network is versatile. Network stations can monitor the Cen-
tral station or any regional station. Because of this messages can be broadcast to more than one station at a time.
A coding system determines the urgency of each message so that those with high priority can be given precedence.
Besides, other communication tools, such as radio-teleprinters and phototelegraphy equipment. Permit rapid
transfers of fingerprints and photographs. Sometimes ever more advanced technology is employed. When the police
all over the world were looking for a Canadian named George Leray, they turned to the Early Bird Satellite. Leray
had led his gang on a daring holdup of a Montreal bank and gotten away with $4 million. Scotland Yard broadcast
Lerays photo to the world by satellite. An American who saw the picture in Florida recognized Leray as a man
who was living on a yacht in Fort Lauderdale under an assumed name. The police were alerted and arrested Leray.

1. What is the best title for this passage?


[A] The Function of the Interpol.
[B] The Quality of the Interpol.
[C] The Organization of the Interpol.
[D] The Rapid Development of the Interpol.
2. The organization of this passage is
[A] general to specific.
[B] cause and effect.
[C] comparison and contrast.
[D] development.
3. The sentence stuffed with clerks choking on their own statistics in the first paragraph is closest in meaning to
[A] a lot of employees busying in their work.
[B] many office workers busying with various documents.
[C] crowded with office workers busying with their own collected data.
[D] workers busying in their own information.

82
4. Which is the easiest tool to communicate?
[A] Satellite.
[B] Radio.
[C] Teleprinter.
[D] Phototelegraphy.

VOCABULARY
1. Interpol
2. outfit
3. chisel-jawed
4. gimlet-eyed
5. jeopardy
6. flatteringly
7. choke
choking on their own statistics
8. as with most generalities
9. grim
10. drone
11. shuffle
12. charisma
13. National Central Bureaus
14. Treasury Department
15. Questura
16. Melbourne
17. chore
18. to be bound to
19. geographic zone
20. versatile
21. radio-teleprinter
22. phototelegraph
23. daring
24. holdup
25. alert

ANSWER
1. C 2. A 3. C 4. B

83
Passage 21: The Result of the Falling US Dollar

Like a ticking time bomb, the falling dollar has grabbed the attention of Japan and West Germany, forcing
them to consider adopting economic polices the United States advocates. The U.S. government wants the dollar to
fall because as the dollar declines in value against the yen and Deutsche mark, U.S. good becomes cheaper. U.S.
companies then sell more at home and abroad, and U.S. trade deficit declines. Cries for trade protection abate, and
the global free-trade system is preserved.
Then, the cheaper dollar makes it cheaper for many foreign investors to snap up U.S. stocks. That prompts
heavy buying from abroadespecially from Japan. Also, if the trade picture is improving, that means U.S. compa-
nies eventually will be more competitive. Consequently, many investors are buying shares of export-oriented U.S.
companies in anticipation of better profits in the next year or so. But that is a rather faddish notion right now; if
corporate earnings are disappointing in interest rates, the stock market rally could stall.
Improving U.S. competitiveness means a decline in anothers competitiveness.
Japan and West Germany are verging on recession. Their export-oriented economies are facing major
problems. Japan is worried about the damage the strong yen will do to Japanese trade. West Germany is also wor-
ried. Share prices in Frankfurt plummeted this past week. Bonn is thought to be considering a cut in interest rates
to boost its economy.
Could the falling dollar get out of hand? If the dollar falls too far, investors might lose confidence in U.S.
investmentsespecially the government bond market. The money to finance the federal budget and trade deficits
could migrate elsewhere. Inflation could flare up, too, since Japanese and German manufacturers will eventually
pass along price hikesand U.S. companies might follow suit to increase their profit margins. The U.S. federal
Reserve then might need to step in and stabilize the dollar by raising interest rates. And higher interest rates could
cause the U.S. economy to slow down and end the Wall Street Rally.
Worried about these side effects, Federal Reserve chairman Paul Volcher has said the dollar has fallen far
enough. What is the equilibrium level? Probably near where it is or slightly lower. It all depends on when the U.S.
trade deficit turns around or if investors defect from U.S. Treasury Bonds. It requires a good deal of political pa-
tience on the part of the U.S. Congress, says Dr. Cline, And there must be an expectation of patience on the part
of private investors. The chance are relatively good that we will avoid an investor break or panic.

1. What is the main idea of this passage?


[A] The impression of the falling U.S. dollar.
[B] The result of the U.S. falling dollar.
[C] The side effect of U.S. falling dollar.
[D] Japan and West Germany are worried about U.S. falling dollar.
2. What does the word rally mean.
[A] prosperity.
[B] decline.
[C] richness.
[D] import.
3. Why are Japan and West Germany worried about the falling dollar?
[A] Because the falling dollar may cause inflation in their countries.
[B] Because it may force them to sell a lot of U.S, stocks.
[C] Because it may do damage to their trade.
[D] Because it may make Japanese company less competitive.

84
4. If dollar-falling got out of hand, and the U.S. Federal Reserve might step in , what would happen?
[A] The prosperity of the U.S. economy would disappear.
[B] The U.S. economy might face serious problems.
[C] Investors might lose confidence in U.S. investments.
[D] Inflation could flare up.

VOCABULARY
1. ticking
2. grab
3. abate
4. snap up
5. heavy buying
6. export-oriented
7. in anticipation of
8. faddish
9. spree
10. buying spree
11. plummet
12. stall
13. verge
14. verging on recession
15. boost
16. bond market
17. flare up
18. hike
19. follow suit
20. profit margin
21. step in
22. rally
23. equilibrium
24. defect
25. break or panic

ANSWER
1. B 2. A 3. C 4. A

85
Passage 22: Pantomime

Pantomimes like pageants, need to be very well planned, and it is essential that initial organization should
begin many weeks in advance of the production date. Pantomimes are nearly always divided into separate scenes,
very often taking place in different countries or even in different centuries. It is therefore necessary that there
should be an overall unity of design culminating in the grand finale. This latter is really just an excuse for visual
effects, and for once the performers become merely cloths hangers on which to put elaborate garments.
The scene should be discussed at some length with the producer so that the background, which is decided
upon, does not present impossibilities for the provision of costumes within the budget or insuperable making
problems for the wardrobe. Because of the large numbers of costumes needed full use must be made of the cheapest
materials available, such as tarlatan (thin, stiff, open-weave muslin) nets and inexpensive cottons and taffetas. Very
often it is possible to pick up goods that have been substantially reduced in price as cheap lines either in the big
stores or on stalls in street markets.
Costumes for pantomimes need to be imaginative, gay and fairly bold in conceptionthis does not mean
that they need to be garish. Usually in one scene there needs to be the flavor of what is newest at the moment in
clothes. It is always a good idea to make use of a modern gimmick and to point it in some way if this can be conve-
niently fitted into the scheme. The audience comes to pantomime to have the eye feasted as much as for any other
purpose, thus making a great chance for the designer to excel. Because of the very varied audience to be catered
for there must be costumes to please patrons of all ages and delight the eyes of toddlers, teenagers, parents and
grandparents.
Usually there is the chance for some country scene involving merry-making peasants in ginghams, stripes
of chintzes. There may be a military or naval routine or some number emphasizing precision and calling for trim
slick costumes. There is certain to be a ballet which is to look fairy-like or romantic and pretty and which may
well need either classical or romantic tutus. The finale, which must be the most spectacular of all, is often set in a
ballroom or palace where all the characters come together to make their final bows; and it is for this scene that the
glitter of sequins and jewels, the sparkle of tinsel, the gold and silver materials and the waving plumes should be
saved.
It may be helpful to examine the different characters and the various scenes in which they are likely to ap-
pear. They remain much the same in all pantomimes; the flavor varying according to the settingso that a s\dash
of the Orient, or the particular feeling of a historical epoch is added to the standard costume.

1. Which word can best describe the final scene in pantomime?


[A] Spectacular.
[B] Beautiful.
[C] Romantic.
[D] Sparkling.
2. What is the best title for this passage?
[A] How to Gain Success of a Pantomime.
[B] The Most Important Factors in a Pantomime.
[C] Pantomime needs to have a very-well.
[D] How to Make Preparation for a Pantomime.
3. What does paragraph two imply?
[A] It implies actor and actress outnumber costume.
[B] It implies costume costs too much.

86
[C] It implies the funds available is on meager side.
[D] It implies a good plan is necessary for a pantomime.
4. The success of a pantomime lie in
[A] well-planned preparation.
[B] excellent actors.
[C] brilliant costume.
[D] harmony.

VOCABULARY
1. culminate
2. insuperable
3. tarlatan
4. taffeta
5. gimmick
6. garish
7. gingham
8. stripe
9. chintz
10. slick
11. tutu
12. sequin
13. tinsel

ANSWER
1. A 2. D 3. C 4. A

87
Passage 23: Women and Fashions

Whenever you see an old film, even one made as little as ten years ago, you cannot help being struck by the
appearance of the women taking part. Their hair-styles and make-up look dated; their skirts look either too long
or too short; their general appearance is, in fact, slightly ludicrous. The men taking part in the film, on the other
hand, are clearly recognizable. There is nothing about their appearance to suggest that they belong to an entirely
different age.
This illusion is created by changing fashions. Over the year, the great majority of men have successfully
resisted all attempts to make them change their style of dress. The same cannot be said for women. Each year a few
so- called top designers in Paris or London lay down the law and women the whole world over rush to obey. The
decrees of the designers are unpredictable and dictatorial. This year, they decide in their arbitrary fashion, skirts
will be short and waists will be high; zips are in and buttons are out. Next year the law is reversed and far from
taking exception, no one is even mildly surprised.
If women are mercilessly exploited year after year, they have only themselves to blame. Because they shud-
der at the thought of being seen in public in clothes that are out of fashion, they are annually black-mailed by the
designers and the big stores. Clothes, which have been worn, only a few times have to be discarded because of
the dictates of fashion. When you come to think of it, only a women is capable of standing in front of a wardrobe
packed full of clothes and announcing sadly that she has nothing to wear.
Changing fashions are nothing more than the deliberate creation of waste. Many women squander vast
sums of money each year to replace clothes that have hardly been worn. Women, who cannot afford to discard
clothing in this way, waste hours of their time altering the dresses they have. Hem-limes are taken up or let down;
waist-lines are taken in or let out; neck-lines are lowered or raised, and so on.
No one can claim that the fashion industry contributes anything really important to society. Fashion
designers are rarely concerned with vital things like warmth, comfort and durability. They are only interested in
outward appearance and they take advantage of the fact that women will put up with any amount of discomfort,
providing they look right. There can hardly be a man who hasnt at some time in his life smiled at the sight of a
woman shivering in a flimsy dress on a wintry day, or delicately picking her way through deep snow in dainty
shoes.
When comparing men and women in the matter of fashion, the conclusions to be drawn are obvious. Do the
constantly changing fashions of womens clothes, one wonders, reflect basic qualities of fickleness and instability?
Men are too sensible to let themselves be bullied by fashion designers. Do their unchanging styles of dress reflect
basic qualities of stability and reliability? That is for you to decide.

1. The main idea of this passage is


[A] New fashions in clothes reflect the qualities of women.
[B] New fashions in clothing are created solely for commercial exploitation of women.
[C] The top designers seem to have the right to creating new fashion.
[D] Men have the basic quality of reliability.
2. Why do the general appearance of actresses look ludicrous?
[A] Because they want their appearance in the fashion.
[B] Because the top designers want them to follow the fashion.
[C] Because the top designers want them to make fashion.
[D] Because the top designers want them to lead the fashion.
3. Why are women mercilessly exploited by the fashion designers?

88
[A] They love new fashion.
[B] They love new clothes.
[C] They want to look beautiful.
[D] They are too vain.
4. What are fashion designers interested in?
[A] Outward appearance.
[B] Comfort.
[C] Beauty.
[D] Durability.

VOCABULARY
1. dated
2. decree
3. dictatorial
4. arbitrary
5. take exception (at)
6. take exception to (against)
7. dictates (pl)
8. squander
9. hem
10. flimsy
11. dainty
12. fickle
13. bully
14. lay down the law

ANSWER
1. B 2. D 3. D 4. A

89
Passage 24: A Smuggling Syndicate

The smuggler in many ways is just another international businessman and his turnover would do credit to
many international corporations. His business happens to be illegal and risky, but look at the stakes involved: $5
billion worth of heroin smuggled into the United States each year, and $1.5 billion in gold passing annually along
smuggling pipelines to India and Indonesia, to France and Morocco, to Brazil and Turkey. Perhaps half of all the
watches made in Switzerland reach their eventual wearers by some back door. Most of this illicit trade is carried
on with all the efficiency of any multinational company. Entirely legitimate businesses, such as a travel bureau or
an import-export agency, are also often fronts for smuggling organizations. One of the worlds largest gold smug-
glers also owned and operated the franchise for a leading make of British cars in a small Middle Eastern country.
He made a good profit from both activities.
A smuggling syndicate operates much like any other business. The boss is really a chief executive. He
makes all the plans, establishes international contacts, and thinks up the smuggling routes and method but remains
aloof from actual operations. He is aided by a handful of managers looking after such specialties as financing,
travel (one reason why many smuggling syndicates find it handy to have their own travel agency), the bribing of
airline or customs officials, and recruitment of couriers, or mules as they are called. There may also be someone in
charge of local arrangements in the countries to which the smuggled goods is going.
Another similarity between legitimate business and its illegal counterpart is price fluctuation. Just as the
prices of products traded legally vary with quality and market conditions such as supply and demand, so do the
prices of goods go up and down in the smuggling trade. Consider the price of drugs. Heroin and cannabis, in what-
ever form or by whatever name, cone in several grades, each with a going price. The wholesale price at which big
dealers sell to big dealers is less than the street price. When the authorities are successful in reducing the supply
buy seizures, the price of all grades rises.

1. The main idea for this passage is


[A] The Comparison between Legitimate Business and Its Illegal Counterpart.
[B] The similarities between Legitimate Business and Smuggling.
[C] Smugglers May Make Great Profit from Both Activities.
[D] The Boss in Smuggling Syndicate is a Chief Executive.
2. When is the price going down?
[A] The quality of the foods and market condition are not very well.
[B] The quality of goods and market condition vary.
[C] Unbalance between supply and demand.
[D] The price of other goods fluctuates.
3. It can be inferred that a smuggler
[A] may make plan and establish international contacts.
[B] is a real boss.
[C] may make money in different ways .
[D] may sell other goods.
4. One of the best ways smugglers usually take is
[A] to set up multinational companies.
[B] to engage in illegal businesses only.
[C] to make legitimate businesses as fronts for smuggling organizations.
[D] to make good profits from both activities.

90
VOCABULARY
1. turnover
2. do credit to /
3. stake
4. smuggling pipeline
5. speciality
6. recruit

ANSWER
1. B 2. A 3. D 4. C

91
Passage 25: Exploration of the Titanic

After resting on the ocean floor, split asunder and rusting, for nearly three-quarters of a century, a great
ship seemed to cone alive again. The saga of the White Star liner Titanic, which struck an iceberg and sank on its
maiden voyage in 1912, carrying more than 1,500 passengers to their death, has been celebrated in print and on
film, in poetry and song. But last week what had been legendary suddenly became real. As they viewed videotapes
and photographs of the sunken leviathan, millions of people around the world could sense her mass, her eerie quiet
and the ruined splendor of a lost age.
Watching on television, they vicariously joined the undersea craft Alvin and Jason Jr. (J.J.) as they toured
the wreckage of the luxury liner, wandering across the decks past corroded bollards, peering into the officers
quarters and through rust-curtained portholes. Views of the railings where doomed passengers and crewmembers
stood evoked images of the moonless night 74years ago when the great ship slipped beneath the waves.
The two-minute videotape and nine photographs, all in color and shot 12,500ft.under the North Atlantic,
were a tiny sample of 60 hours of video and 60,000 stills garnered during the twelve-day exploration. They are
released at a Washington press conference conducted by Marine Geologist Robert Ballard, 44, who led the teams
from the Wood Hole Oceanographic Institution that found the Titanic last September and revisited it this July.
Recounting the highlights of what has already become the most celebrated feat of underwater exploration,
Ballard revealed some startling new information. His deep-diving craft failed to find the 300-ft. gash that, accord-
ing to legend, was torn in the Titanics hull when the ship plowed into the iceberg. Instead, he suggested, the col-
lision had buckled the ships plates, allowing water to pour in. He also brought back evidence that the ship broke
apart not when she hit bottom, as he had thought when viewing the first Titanic images last September, but as she
sank: the stern, which settled on the bottom almost 1,800ft. from the bow, had swiveled 180 on its way down.

1. What is the best title for this passage?


[A] New Information about the Sunken Ship.
[B] Exploration of the Titanic.
[C] To watch the Videotape.
[D] To explore the Titanic with High Tech.
2. How did the viewers feel when watching the videotape?
[A] They felt rather sad, and felt they themselves took part in the exploration.
[B] They felt keenly for it.
[C] They felt rather bad about it.
[D] They felt out of spirits.
3. When did the great ship sink?
[A] In 1912.
[B] She sank in its maiden voyage in 1912.
[C] She sank in its second voyage in 1912.
[D] She sank in its first voyage in 1912.
4. What did Robert reveal at a press conference?
[A] He revealed some startling information.
[B] He said nothing.
[C] He complained the exploration was very hard.
[D] He revealed the success of their work.

92
VOCABULARY
1. split asunder
2. saga
3. maiden voyage
4. celebrate
5. leviathan
6. eerie
7. splendor
8. vicariously
9. luxury liner
10. bollard
11. porthole
12. railing
13. evoke
14. still
15. garner
16. highlight
17. feat
18. gash
19. hull
20. plow into= plough into
21. buckle
22. stern
23. bow
24. swivel

ANSWER
1. D 2. A 3. B 4. A

93
Passage 26: Sensory Evaluation of Food

A Polish proverb claims that fish, to taste right, should three timesin water, in butter and in wine. The
early efforts of the basic scientists in the food industry were directed at improving the preparation, preservation,
and distribution of safe and nutritious food. Our memories of certain foodstuffs eaten during the World War II sug-
gest that, although these might have been safe and nutritious, they certainly did not taste right nor were they par-
ticularly appetizing in appearance or smell. This neglect of the sensory appeal of foods is happily becoming a thing
of the past. Bow, in the book Principles of Sensory Evaluation of Food, the authors hope that it will be useful to
food technologists in industry and also to others engaged in research into problem of sensory evaluation of foods.
An attempt has clearly been make to collect every possible piece of information, which might be useful, more than
one thousand five hundred references being quoted. As a result, the book seems at first sight to be an exhaustive
and critically useful review of the literature. This it certainly is, but this is by no means is its only achievement, for
there are many suggestions for further lines of research, and the discursive passages are crisply provocative of new
ideas and new ways of looking at established findings.
Of particular interest is the weight given to the psychological aspects of perception, both objectively and
subjectively. The relation between stimuli and perception is well covered, and includes a valuable discussion of the
uses and disadvantages of the Weber fraction of differences. It is interesting to find that in spite of many attempts
to separate and define the modalities of taste, nothing better has been achieved than the familiar classification
into sweet, sour salty and bitter. Nor is there as yet any clear-cut evidence of the physiological nature of the taste
stimulus. With regard to smell, systems of classification are of little value because of the extraordinary sensitivity of
the nose and because the response to the stimulus is so subjective. The authors suggest that a classification based
on the size, shape and electronic status of the molecule involved merits further investigation, as does the theoreti-
cal proposition that weak physical binding of the stimulant molecule to he receptor site is a necessary part of the
mechanism of stimulation.
Apart from taste and smell, there are many other components of perception of the sensations from food in
the mouth. The basic modalities of pain, cold, warmth and touch, together with vibration sense, discrimination and
localization may all play a part, as, of course, does auditory reception of bone-conducted vibratory stimuli from the
teeth when eating crisp or crunchy foods. In this connection the authors rightly point out that this type of stimulus
requires much more investigation, suggesting that a start might be made by using subjects afflicted with various
forms of deafness. It is well-known that extraneous noise may alter discrimination, and the attention of the authors
is directed to the work of Prof. H. J. Eysenck on the stimulus hunger of extroverts and the stimulus avoidance
of introverts.

1. The reviewer uses a Polish proverb at the beginning of the article in order to
[A] introduce, in an interesting manner, the discussion of food.
[B] show the connection between food and nationality of food.
[C] indicate that there are various ways to prepare food.
[D] impress upon the reader the food value of fish.
2. The reviewers appraisal of Principles of Sensory Evaluation of Food is one of
[A] mixed feelings.
[B] indifference
[C] high praise.
[D] faint praise.
3. The writer of the article does not express the view, either directly or by implication, that

94
[A] sharply defined classifications of taste are needed.
[B] more research should be done regarding the molecular constituency of food.
[C] food values are objectively determined by an expert smeller.
[D] temperature is an important factor in the value of food.
4. The authors of the book suggest the use of deaf subject because
[A] deaf people are generally introversive.
[B] the auditory sense is an important factor in food evaluation.
[C] they are more fastidious in their choice of foods.
[D] All types of subjects should be used.

VOCABULARY
1. preservation
2. sensory appeal
3. be provocative of
4. exhaustive
5. discursive
6. be provocative of /
7. crisp
8. perception
9. modality
modality of taste
10. discrimination
11. localization
12. merit
13. crunchy
14. extraneous
15. extrovert
16. introvert

ANSWER
1. A 2. C 3. C 4. B

95
Passage 27: Analysis and Interpretation of the News

The newspaper must provide for the reader the facts, unalloyed, unslanted, objectively selected facts. But in
these days of complex news it must provide more; it must supply interpretation, the meaning of the facts. This is the
most important assignment confronting American journalismto make clear to the reader the problems of the day,
to make international news as understandable as community news, to recognize that there is no longer any such
thing (with the possible exception of such scribbling as society and club news) as local news, because any event
in the international area has a local reaction in manpower draft, in economic strain, in terms, indeed, of our very
way of life.
There is in journalism a widespread view that when you embark on interpretation, you are entering choppy
and dangerous waters, the swirling tides of opinion. This is nonsense.
The opponents of interpretation insist that the writer and the editor shall confine himself to the facts.
This insistence raises two questions: What are the facts? And: Are the bare facts enough?
As to the first query. Consider how a so-called factual story cones about. The reporter collects, say, fifty
facts; out of these fifty, his space allotment being necessarily restricted, he selects the ten, which he considers most
important. This is Judgment Number One. Then he or his editor decides which of these ten facts shall constitute
the lead of the piece (This is important decision because many readers do not proceed beyond the first paragraph.)
This is Judgment Number Two. Then the night editor determines whether the article shall be presented on page
one, where it has a large impact, or on page twenty-four, where it has little. Judgment Number Three.
Thus, in the presentation of a so-called factual or objective story, at least three judgments are in-
volved. And they are judgments not at all unlike those involved in interpretation, in which reporter and editor,
calling upon their general background, and their news neutralism, arrive at a conclusion as to the significance
of the news.
The two areas of judgment, presentation of the news and its interpretation, are both objective rather then
subjective processesas objective, that is, as any human being can be. (Note in passing: even though complete
objectivity can never be achieved, nevertheless the ideal must always be the beacon on the murky news channels.)
Of an editor is intent on slanting the news, he can do it in other ways and more effectively than by interpretation.
He can do it by the selection of those facts that prop up his particular plea. Or he can do it by the pay he gives a
storypromoting it to page one or demoting it to page thirty.

1. The title that best expresses the ideas of this passage is


[A] Interpreting the News.
[B] Choosing Facts.
[C] Subjective versus Objective Processes.
[D] Everything Counts.
2. Why does the writer of an article select ten out of 50 available facts?
[A] Space is limited.
[B] His editor is prejudiced.
[C] The subject is not important.
[D] He is entering choppy and dangerous.
3. What is the least effective way of slanting news/
[A] Interpretation.
[B] His editor is prejudiced.
[C] Placement.

96
[D] Concentration.
4. Why should the lead sentence present the most important fact?
[A] It will influence the reader to continue.
[B] It will be the best way to write.
[C] Some readers do not read beyond the first paragraph.
[D] It will gratify the editor.

VOCABULARY
1. unalloyed
2. unslanted
3. scribble
4. manpower draft
5. economic strain
6. embark on
7. choppy
8. query
9. come about
10. allotment
11. beacon
12. murky
13. prop up
14. demote promote
15. news neutralism
16. lead

ANSWER
1. A 2. A 3. A 4. C

97
Passage 28: Chemistry and Biology

About a century ago, the Swedish physical scientist Arrhenius proposed a low of classical chemistry that
relates chemical reaction rate to temperature. According to his equation, chemical reactions are increasingly
unlikely to occur as temperature approaches absolute zero, and at absolute zero, reactions stop. However, recent
experiment evidence reveals that although the Arrhenius equation is generally accurate in describing the kind of
chemical reaction that occurs at relatively high temperature, at temperatures closer to zero a quantum-mechanical
effect known as tunneling comes into play; this effect accounts for chemical reactions that are forbidden by the
principles of classical chemistry. Specifically, entire molecules can tunnel through the barriers of repulsive forces
from other molecules and chemically react even though these molecules do not have sufficient energy, according to
classical chemistry, to overcome the repulsive barrier.
The rate of any chemical reaction, regardless of the temperature at which it takes place, usually depends on
a very important characteristic known as its activation energy. Any molecule can be imagined to reside at the bot-
tom of a so-called potential well of energy. S chemical reaction corresponds to the transition of a molecule from the
bottom of one potential well to the bottom of another. In classical chemistry, such a transition can be accomplished
only by going over the potential barrier between the well, the height of which remain constant and is called the
activation energy of the reaction. In tunneling, the reacting molecules tunnel from the bottom of one to the bottom
of another well without having to rise over the barrier between the two wells. Recently researchers have developed
the concept of tunneling temperature: the temperature below which tunneling transitions greatly outnumber Arrhe-
nius transitions, and classical mechanics gives way to its quantum counterpart.
This tunneling phenomenon at very low temperatures suggested my hypothesis about a cold prehistory of
life: formation of rather complex organic molecules in the deep cold of outer space, where temperatures usually
reach only a few degrees Kelvin. Cosmic rays might trigger the synthesis of simple molecules, such as interstellar
formaldehyde, in dark clouds of interstellar dust. Afterward complex organic molecules would be formed, slowly
but surely, by means of tunneling. After I offered my hupothesis, Hoyle and Wickramashinghe argued that mol-
ecules of interstellar formaldehyde have indeed evolved into stable polysaccharides such as cellulose and starch.
Their conclusions, although strongly disputed, have generated excitement among investigators such as myself who
are proposing that the galactic clouds are the places where the prebiological evolution of compounds necessary to
life occurred.

1. The author is mainly concerned with


[A] describing how the principles of classical chemistry were developed.
[B] initiating a debate about the kinds of chemical reaction required for the development of life.
[C] explaining how current research in chemistry may be related to broader biological concerns.
[D] clarifying inherent ambiguities in the laws of classical chemistry.
2. In which of the following ways are the mentioned chemical reactions and tunneling reactions alike?
[A] In both, reacting molecules have to rise over the barrier between the two wells.
[B] In both types of reactions, a transition is made from the bottom of one potential well to the bottom of
another.
[C] In both types of reactions, reacting molecules are able to go through the barrier between the two wells.
[D] In neither type of reaction does the rate of a chemical reaction depend on its activation energy.
3. The authors attitude toward the theory of a cold prehistory of life can best be described as
[A] neutral.
[B] skeptical.

98
[C] mildly positive.
[D] very supportive.
4. Which of the following best describes the hypothesis of Hoyle and Wickramasinghe?
[A] Molecules of interstellar formaldehyde can evolve into complex organic molecules.
[B] Interstellar formaldehyde can be synthesized by tunneling.
[C] Cosmic rays can directly synthesize complex organic molecules.
[D] The galactic clouds are the places where prebilogical evolution of compounds necessary to life occurred.

VOCABULARY
1. Arrhenius equation
2. Arrhennius (Svante August) 18591927 1903
3. quantum-mechanical effect
4. quantum mechanic
5. tunnel
6. tunnel through, onto
7. repulsive
8. activation energy
9. formaldehyde
10. polysaccharide
11. cellulose
12. starch
13. galactic
14. come into play /

ANSWER
1. C 2. B 3. C 4. A

99
Passage 29: Men Are Carrying on a Sex-fight

This is supposed to be an enlightened age, but you wouldnt think of if you could heat what the average
man thinks of the average woman/ Women won their independence years ago. After a long, bitter struggle, they
now enjoy the same educational opportunities as men in most parts of the world. They have proved repeatedly that
they are equal and often superior to men in almost every field. The hard-fought battle for recognition has been
won, but it is by no means over. It is men, not women who still carry on the sex war because their attitude remains
basically hostile. Even in the most progressive societies, women continue to be regarded as second-rate citizens. To
hear some men talk, youd think that women belonged to a different species!
On the surface, the comments made by men about womens abilities seem light-hearted. The same tired
jokes about women drivers are repeated day in, day out. This apparent light-heartedness dose not conceal the real
contempt that men feel for women. However much men sneer at women, their claims to superiority are not borne
out by statistics. Lets consider the matter of driving, for instance. We all know that women cause far fewer ac-
cidents than men. They are too conscientious and responsible to drive like maniacs. But this is a minor quibble.
Women have succeeded in any job you care to name. As politicians, soldiers, doctors, factory-hands, university
professors, farmers, company directors, lawyers, bus-conductors, scientists and presidents of countries they have
often put men to shame. And we must remember that they frequently succeed brilliantly in all these fields in addi-
tion to bearing and rearing children.
Yet men go on maintaining the fiction that there are many jobs women cant don Top-level political nego-
tiation between countries, business and banking are almost entirely controlled by men, who jealously guard their
so-called rights. Even in otherwise enlightened places like Switzerland women havent even been given the cote.
This situation is preposterous! The arguments that men put forward to exclude women from these fields are all too
familiar. Women, they say, are unreliable and irrational. They depend too little on cool reasoning and too much
on intuition and instinct to arrive at decisions. They are not even capable of thinking clearly. Yet when women
prove their abilities, men refuse to acknowledge them and give them their due. So much for a mans ability to think
clearly!
The truth is that men cling to their supremacy because of their basic inferiority complex. They shun real
competition. They know in their hearts that women are superior and they are afraid of being beaten at their own
game. One of the most important tasks in the world is to achieve peace between the nations. You can be sure that if
women were allowed to sit round the conference table, they would succeed brilliantly, as they always do, there men
have failed for centuries. Some things are too important to be left to men!

1. What does the first sentence imply?


[A] It is not really an enlightened age.
[B] It is different from an enlightened age.
[C] It is the same as an enlightened age.
[D] It is like an enlightened age.
2. Why do men carry on the sex war against women?
[A] Because of their inferiority.
[B] Because they shun real competition.
[C] Because of their claim to supremacy.
[D] Because they still look down upon women.
3. The fiction is closest in meaning to
[A] Novel.

100
[B] Man-made idea.
[C] False idea.
[D] Story.
4. What is the main argument men have raised against women?
[A] Women are lack of cold reasoning.
[B] They depend on intuition too much.
[C] They are unreliable and irrational.
[D] They are too still look down upon women.

VOCABULARY
1. conscientious
2. maniac
3. preposterous
4. quibble

ANSWER
1. A 2. C 3. C 4. C

101
Passage 30: Fingerprints

The search for latent prints is done in a systematic and intelligent manner. Investigators develop techniques
to locate traces of fingerprints at a crime scene. The basic premise in searching for latent prints is to examine more
carefully those areas, which would most likely be touched by persons who have been on the scene. The natural
manner in which a person would use and place his hands in making an entrance or exit from a building or in han-
dling any object is the key to the discovery of latent prints.
Where a forced entrance has been made, latent prints are likely to be found on any surface adjacent to or
at that point. Any object with a smooth, non-porous surface is likely to retain latent prints if touched. Fingerprints
on rough surfaces are usually of little value. If the fingermark does not disclose ridge detail when viewed under
a reading glass, the chances are that its value in identification is nil when photographed. Where fingermarks are
found, it will be necessary for the investigator to compare them against the ones of persons having legitimate
access to the premises so that the traces might be eliminated as having evidentiary value if they prove to be from
these persons. Places to search for prints on an automobile are the rear view mirror, steering wheel hub, steering
column, windshield dashboard and the like.
Dusting of surface may be done with a fine brush or with an atomizer. The whit powders used are basically
finely powdered white lead, talc, or chalk. Another light powder is basically Chemists gray. A good black powder
is composed of lampblack, graphite, and powdered acacia. Dragons blood is good powder for white surface and
can be fixed on paper by heating. In developing latent prints, the accepted method is to use the powder sparingly
and brush lightly. Do not use powder if the fingermark is visible under oblique lighting. It can be photographed. A
good policy for the novice is to experiment with his own prints on a surface similar to the one he wishes to search
in order to determine the powder best suited to the surface. Fingerprints after dusting may be lifted by using fresh
cellulose tape or commercially prepared material especially designed to lift and transfer dusted latent fingerprints.
In addition to latent prints, the investigator must not overlook the possibility of two other types of finger-
print traces: molded impression and visible impression. Molded impressions are formed by the pressure of the fin-
ger upon comparatively soft, pliable, or plastic surfaces producing an actual mold of the fingerprint pattern. These
can be recorded by photograph without treating the surface, is usually most effective in revealing the impressions
clearly. Visible impressions are formed when the finger is covered with some substance which is transferred to the
surface contacted. Fingers smeared with blood, grease, dirt, paint, and the like will leave a visible impression. If
these impressions are clear and sharp, they are photographed under light without ant treatment. Ordinarily, prints
of this type are blurred or smeared and do not contain enough detail for identification by comparison. However,
they can not be overlooked or brushed aside without first being examined carefully.

1. What is the best title for this passage?


[A] Visible impressions.
[B] Moulded impressions.
[C] Fingerprints.
[D] Latent fingerprints.
2. How many fingermarks are mentioned in this passage?
[A] 2.
[B] 3.
[C] 4.
[D] 5.
3. Which type of fingerprints is most likely to retain?

102
[A] Latent fingerprints.
[B] Visible impressions.
[C] Moulded impressions.
[D] Clear fingerprints.
4. How many ways are there to develop fingerprints?
[A] 2.
[B] 3.
[C] 4.
[D] 5.

VOCABULARY
1. latent print
2. premise
3. non-porous
4. ridge
5. nil
6. premise
7. rear view mirror
8. steering wheel hub
9. steering column
10. windshield
11. dashboard
12. atomizer
13. talc
14. lampblack
15. acacia
16. dragons blood
17. sparingly
18. novice
19. cellulose tape
20. lift
21. molded impression
22. visible impression
23. pliable
24. smear

ANSWER
1. C 2. B 3. A 4. A

103
Passage 31: The Causes of European Separation in 16th Century

For a thousand years and more, the people of Europe had fought about many things, but they had been
united in believing one thing: that there existed a single Holy, Catholic and Apostolic Church of which the
Bishop of Rome, under the title of the Pope, was the visible and recognizable head in succession to St. Peter. But
in 1517 a German monk, Martin Luther, challenged certain Catholic teachings and renounced his obedience to the
papacy. Others had followed him, including Henry VIII.
Thus Europe was divided in every which way, the southern and eastern two-thirds still Catholic, the north-
ern and western one-third what was coming to be called Protestant, though English-ruled Ireland solidly Catholic
and the Spanish-ruled Netherlands, particularly the northern part approximating to modern Holland, grew increas-
ingly Protestant; while in virtually every country, whether officially Catholic or Protestant, those of the contrary
faith fiercely attempted to convert their neighbors and equally fiercely resisted their neighbors attempts to convert
them. For this there was no simple, friendly solution to be reached on the principle of live-and-let-live. Each party
believed that it had hold of the truth, the only truth that mattered, the one that led to eternal salvation, and its ad-
versaries clung to falsehood which must necessarily head to eternal damnation: not only for themselves but for all
who should permit them to survive and infect others with their errors. Toleration, even reasonable discussion, was
impossible. God and the devil could not mix. Just as Elizabeth was to ardent Catholics that Jezebel, so to earnest
Protestants the Pope was that wolfish bloodsucker, and their Catholic fellow-creatures mad dogs, toads and
other such vermin to be cleansed off the face of the earth.
These feelings, dangerous enough in themselves, were made more so by questions of geography and money.
The Catholic countries bordering on the Mediterranean were by far the richest. From the beginning of the Middle
Ages the Republic of Venice had controlled the trade routes to the East, bringing the wares carried out of Persia,
China and the Indies by camel to her depots in Syria and reloading them in her high, gorgeously painted vessels
for transshipment to Italy and beyond. Since the end of the fifteen century, first Portugal by sailing round Africa
to India, then Spain by the discovery of America, had likewise been in a position to bring for sale to Europe all the
rare and wonderful things for which Europe longedsilks and precious woods, sugar and spices, gold and silver,
works of exquisite art and strange animals from peacock to tigers. In 1494, two years after Columbuss first voyage
to America, Pope Alexander VI had divided the unexplored world beyond the seas between Spain and Portugal as
reward for their enterprise and to keep them from fighting. The other countries had respected this division so long
as they remained Catholic.

1. The best title for this passage is


[A] The History of Europe in 16th Century.
[B] The Religious History of Europe in 16th Century.
[C] The Causes of European Separation in 16th Century.
2. What does we learn from the passage?
[A] The Pope had the supreme power in religion before reform.
[B] The Pope had the greatest power in every thing outside religion.
[C] The Pope was the real king in Europe then.
[D] The Pope was the real ruler in Europe then.
3. What did the sentence The other countries had respected this division so long as they remained Catholic im-
ply?
[A] It implied this division could not be respected long.
[B] It implied this division would not face a challenge.

104
[C] It implied this division would be respected forever.
[D] It implied the power of the Pope would never decline.
4. Which of the following is not mentioned as a cause to deepen the dangerous feelings?
[A] Money.
[B] Geology.
[C] Religion.
[D] Geography.

VOCABULARY
1. Apostolic
2. in succession to
3. Martin Luther 14831546
4. teachings
5. renounce
6. papacy /
7. cling to
8. damnation
9. bloodsucker
10. depot
11. gorgeous
12. spice
13. enterprise

ANSWER
1. C 2. C 3. A 4. B

105
Passage 32: The Young Generation

Old people are always saying that the young are not what they were. The same comment is made from gen-
eration to generation and it is always true. It has never been truer than it is today. The young are better educated.
They have a lot more money to spend and enjoy more freedom. They grow up more quickly and are not so depen-
dent on their parents. They think more for themselves and do not blindly accept the ideals of their elders. Events
which the older generation remembers vividly are nothing more than past history. This is as it should be. Every
new generation is different from the one that preceded it. Today the difference is very marked indeed.
The old always assume that they know best for the simple reason that they have been around a bit longer.
They dont like to feel that their values are being questioned or threatened. And this is precisely what the young
are doing. They are question the assumptions of their elders and disturbing their complacency. Office hours, for
instance, are nothing more than enforced slavery. Wouldnt people work best if they were given complete freedom
and responsibility? And what about clothing? Who said that all the men in the world should wear drab grey suits
and convict haircuts? If we ruin our minds to more serious matters, who said that human differences can best
be solved through conventional politics or by violent means, who said that human difference can best be solved
through conventional politics or by violent means? Why have the older generation so often used violence to solve
their problems? Why are they so unhappy and guilt-ridden in their personal lives, so obsessed with mean ambitions
and the desire to amass more and more material possessions? Can anything be right with the rat-race? Havent the
old lost touch with all that is important in life?
These are not questions the older generation can shrug off lightly. Their record over the past forty years or
so hasnt been exactly spotless. Traditionally, the young have turned to their elders for guidance. Today, the situ-
ation might be reversed. The oldif they are prepared to admit itcould learn a thing or two from their children.
One of the biggest lessons they could learn is that enjoyment is not sinful. Enjoyment is a principle one could ap-
ply to all aspects of life. It is surely not wrong to enjoy your work and enjoy your leisure; to shed restricting inhibi-
tions. It is surely not wrong to live in the present rather than in the past or future. This emphasis on the present is
only to be expected because the young have grown up under the shadow of the bomb: the constant threat of com-
plete annihilation. This is their glorious heritage. Can we be surprised that they should so often question the sanity
of the generation that bequeathed it?

1. Which of the following features in the young is NOT mentioned?


[A] Better educated.
[B] More money and freedom.
[C] Independence.
[D] Hard work.
2. What so the young reject most?
[A] Values.
[B] The assumption of the elders.
[C] Conformity.
[D] Conventional ideas.
3. Why do the young stress on the present?
[A] They have grown up under the shadow of the bomb.
[B] They dislike the past.
[C] They think the present world is the best.
[D] They are afraid of destruction.

106
4. What can the old learn from the young generation?
[A] Enjoyment is not sinful.
[B] People should have more leisure time.
[C] Men might enjoy life.
[D] One should enjoy ones work.

VOCABULARY
1. reminder
2. complacency
3. take leave
I took leave to consider this matter settled.
4. conformity
5. guilt
6. ridden ride
7. guilt-ridden
8. amass
9. a rat-race
10. shrug off
11. spotless
12. shed
13. annihilate
14. bequeath
15. sanity

ANSWER
1. D 2. C 3. A 4. A

107
Passage 33: Importance of a Computer

As citizens of advanced but vulnerable economies, we must either relentlessly increase the quality of our
skills or see our standard of living erode. For the future, competition between nations will be increasingly based on
technological skill. Oil and natural resources will still be important, but they no longer will determine a nations
economic strength. This will now be a matter of the way people organize them selves and the nature and quality of
their work. Japan and the new Japans of East Asia are demonstrating this point in ways that are becoming pain-
fully obvious to the older industrial countries.
There is simply no way to rest on our past achievements. Todays competition renders obsolete huge chunks
of what we know and what forces us to innovate. For each individual. Several careers will be customary, and con-
tinuing education and retraining will be inescapable. To attain this extraordinary level of education, government,
business, schools, and even individuals will turn to technology for the answer.
In industry, processing the information and designing the changes necessary to keep up with the market
has meant the growing use of computers. The schools are now following close behind. Already some colleges in the
United States are requiting a computer for each student. It is estimated that 500,000 computers are already in use
in American high schools and elementary schools. Although there is an abysmal lack of educational software, the
number of computers in schools expands rapidly.
The computer is the Proteus of machines, as it takes on a thousand forms and serves a thousand functions.
But its truly revolutionary character can be seen in its interactive potential. With advanced computers, learning
can be individualized and self-paced. Teachers can become more productive and the entire learning environment
enriched.
It is striking how much current teaching is a product of pencil and paper technology. With the computers
capacity for simulation and diverse kinds of feedback, all sorts of new possibilities open up for the redesign of
curriculums. Seymour Papert, the inventor of the computer language LOGO, believes that concepts in physics and
advanced mathematics can be taught in the early grades with the use of computers. On every-day level, word-pro-
cessing significantly improves the capacity for written expression. In terms of drill and practice, self-paced comput-
er-assisted instruction enables the student to advance rapidlywithout being limited by the conflicting needs of the
entire class. In short, once we learn to use this new brain outside the brain, education will never be the same.
Industry, faced with the pressures of a rapidly shifting market, is already designing new methods to retrain
its workers, In the United States, a technological university has been set up to teach engineering courses by satel-
lite. And the advances in telecommunications and computational power will dramatically expand the opportunities
for national and international efforts in education and training.
Without romanticizing the machine, it is clear that computers uniquely change the potential for equipping
todays citizens for unprecedented tasks of the future. Particularly in Europe and the United States, innovation will
be the basis for continued prosperity. New competitors are emerging to challenge the old economic arrangements.
How successfully we respond will depend on how much we invest in people and how wisely we employ the learning
tools of the new technology.

1. What is the decisive factor in future competition between nations?


[A] Oil.
[B] Technological skill.
[C] Natural resources
[D] Education
2. The main idea of this passage is

108
[A] Knowledge of a Computer.
[B] Importance of a Computer.
[C] Function of Knowledge.
[D] Function of Technology.
3. Why does further study become indispensable?
[A] People want to so more jobs.
[B] People want to attain this extraordinary level of education.
[C] People would not rest on the past achievements.
[D] What we know becomes obsolete.
4. The word Proteus is closest in meaning to
[A] flexibility.
[B] diversity.
[C] variety.
[D] multiplicity.

VOCABULARY
1. relentlessly
2. obsolete
3. chunk
4. abysmal
5. Proteus flexible,
6. take on
7. LOGO=logotyre
8. rapidly shifting
9. romanticizing

ANSWER
1. B 2. B 3. D 4. A

109
Passage 34: The Relationship between Brain Process with Mental Experience

By 1950, the results of attempts to relate brain processes to mental experience appeared rather discour-
aging. Such variations in size, shape, chemistry, conduction speed, excitation threshold, and the like as had been
demonstrated in nerve cells remained negligible in significance for any possible correlation with the manifold
dimensions of mental experience.
Near the turn of the century, it had been suggested by Hering that different modes of sensation, such as
pain, taste and color, might be correlated with the discharge of specific kinds of nervous energy, However, sub-
sequently developed methods of recording and analyzing nerve potentials failed to reveal any such qualitative
diversity. It was possible to demonstrate by other methods refined structural differences among neuron types;
however, proof was lacking that the quality of the impulse or its conduction was influenced by these differences,
which seemed instead to influence the developmental patterning of the neural circuits. Although qualitative vari-
ance among nerve rigidly disproved, the doctrine was generally abandoned in favor of the opposing view, namely,
that nerve impulses are essentially homogeneous in quality and are transmitted as common currency throughout
the nervous system. According to this theory, it is not the quality of the sensory nerve impulses that determines the
diverse conscious sensations they produce, but, rather, the different areas of the brain into which they discharge,
and there is some evidence for this view. In one experiment, when an electric stimulus was applied to a given sen-
sory field of the cerebral cortex of a conscious human subject, it produced a sensation of the appropriate modality
for that particular locus, that is, a visual sensation from the visual cortex, an auditory sensation from the auditory
cortex, and so on. Other experiments revealed slight variations in the size, number, arrangement, and intercon-
nection of the nerve cells, but as for as psychoneural correlations were concerned, the obvious similarities of these
sensory fields to each other seemed much more remarkable than any of the minute differences.
However, cortical as diverse as those of red, black, green and white, or touch, cold, warmth, movement,
pain, posture and pressure apparently may arise through activation of the same cortical areas. What seemed to
remain was some kind of differential patterning effects in the brain excitation: it is the difference in the central
distribution of impulses that counts. In short, Brain theory suggested a correlation between mental experience and
the activity of relatively homogenous nerve-cell units conducting essentially homogeneous impulses through homo-
geneous cerebral tissue. To match the multiple dimensions of mental experience psychologists could only point to a
limitless variation in the spatiotemporal patterning of nerve impulses.

1. Up until 1950, efforts to establish that brain processes and mental experience are related would most likely have
been met with
[A] vexation.
[B] irritability.
[C] discouragement
[D] neutrality
2. The author mentions common currency primarily in order to emphasize the
[A] lack of differentiation among nerve impulses in human beings.
[B] similarities in the views of the scientists.
[C] similarity of sensations of human beings.
[D] continuous passage of nerve impulses through the nervous system.
3. Which of the following theories is reinforced by the depiction of the experiment in lines 1619?
[A] Cognitive experience manifested by sensory nerve impulses are influenced by the area of the brain stimu-
lated.

110
[B] Qualitative diversity in nerve potentials can now be studied more accurately.
[C] Sensory stimuli are heterogeneous and are greatly influenced by the nerve sensors they produce.
[D] Differentiation in neural modalities influences the length of nerve transmissions.
4. It can be inferred from the passage that which of the following exhibit the LEAST qualitative variation?
[A] Nerve cells.
[B] Nerve impulses.
[C] Cortical areas.
[D] Spatial patterns of nerve impulses.

VOCABULARY
1. mental experiences
2. discharge
3. negligible
4. manifold
5. neuron /
6. neural
7. qualitative diversity
8. disprove
9. homogeneous
10. sensory nerve
11. cortical
12. cerebral cortex
13. locus
14. psychoneural
15. heterogeneous
16. spatiotenporal

ANSWER
1. C 2. A 3. A 4. B

111
Passage 35: Exploration on the Origin of Continents

The origin of continental nuclei has long been a puzzle. Theories advanced so far have generally failed to
explain the first step in continent growth, or have been subject to serious objections. It is the purpose of this article
to examine the possible role of the impact of large meteorites or asteroids in the production of continental nuclei.
Unfortunately, the geological evolution of the Earths surface has had an obliterating effect on the original com-
position and structure of the continents to such an extent that further terrestrial investigations have small chance
of arriving at an unambiguous answer to the question of continental origin. Paradoxically, clues to the origin and
early history of the surface features of the Earth may be found on the Moon and planets, rather than on the Earth,
because some of these bodies appear to have had a much less active geological history. As a result, relatively prim-
itive surface features are preserved for study and analysis. In the case of both the Moon and Mars, it is generally
concluded from the appearance of their heavily cratered surfaces that they have been subjected to bombardment by
large meteoroids during their geological history. Likewise, it would appear a reasonable hypothesis that the Earth
has also been subjected to meteoroid bombardment in the past, and that very large bodies struck the Earth early in
its geological history.
The large crater on the Moon listed by Baldwin has a diameter of 285 km. However, if we accept the hy-
potheses of formation of some of the mare basins by impact, the maximum lunar impact crater diameter is probably
as large as 650km. Based on a lunar analogy, one might expect several impact craters of at least 500km diameter
to have been formed on Earth. By applying Baldwins equation, the depth of such a crater should be about 20km.
Baldwin admits that his equation gives excessive depths for large craters so that the actual depth should be some-
what smaller. Based on the measured depth of smaller lunar crater. Baldwins equation gives the depth of the zone
of brecciation for such a crater as about 75km. The plasticity of the Earths mantle at the depth makes it impossible
to speak of bracciation in the usual sense. However, local stresses may be temporarily sustained at that depth,
as shown by the existence of deep-focus earthquakes. Thus, short-term effects might be expected to a depth of more
than 50km in the mantle.
Even without knowing the precise effects, there is little doubt that the formation of a 500-km crater would
be a major geological event. Numerous authors have considered the geological implications of such an event. Donn
et al. have, for example, called on the impact of continent-size bodies of sialic composition to from the original
continents. Two major difficulties inherent in this concept are the lack of any known sialic meteorites, and the high
probability that the energy of impact would result in a wide dissemination of sialic material, rather than its
concentration at the point of impact.
Gilvarry, on the other hand, called on meteoroid impact to explain the production of ocean basins. The
major difficulties with this model are that the morphology of most of the ocean basins is not consistent with impact,
and that the origin and growth of continents is not adequately explained.
We agree with Donn at al. that the impact of large meteorites or asteroids may have caused continent for-
mation, but would rather think in terms of the localized addition of energy to the system, rather than in terms of the
addition of actual sialic material.

1. A mare basin is
[A] a formula for determining the relationship between the depth and width of craters.
[B] a valley that is filled in when a spatial body has impact with the moon or the earth.
[C] a planetoid (small planet) created when a meteorite, upon striking the moon, breaks off a part of the
moon.
[D] a dark spot on the moon, once supposed to be a sea, now a plain.

112
2. The writer does not believe that
[A] an asteroid is larger than a meteorite.
[B] material from space, upon hitting the earth, was eventually distributed.
[C] the earth, at one time, had craters.
[D] ocean were formerly craters.
3. The article is primarily concerned with
[A] the origin of continents.
[B] the relationship between astral phenomena and the moon.
[C] differences of opinion among authoritative geologists.
[D] the relationship between asteroids and meteorites.
4. Sialic material refers to
[A] the broken rock resulting from the impact of a meteorite against the earth.
[B] material that exists on planets other than the earth.
[C] a composite of rock typical of continental areas of the earth.
[D] material that is man-made to simulate materials that existed far back in geological history.

VOCABULARY
1. meteorit
2. asteroid
3. obliterate
4. crater
5. mare
6. impact
7. impact crater
8. brecciate
the zone of brecciation
9. mantle
10. stress
local stress
11. sialic
12. probability
13. disseminate
14. morphology
15. astral

ANSWER
1. D 2. D 3. A 4. C

113
:

Passage 1: Clinton Is Right

President Clintons decision on Apr.8 to send Chinese Premier Zhu Rongji packing without an agreement
on Chinas entry into the World Trade Organization seemed to be a massive miscalculation. The President took a
drubbing from much of the press, which had breathlessly reported that a deal was in the bag. The Cabinet and Whit
House still appeared divided, and business leaders were characterized as furious over the lost opportunity. Zhu
charged that Clinton lacked the courage to reach an accord. And when Clinton later telephoned the angry Zhu
to pledge a renewed effort at negotiations, the gesture was widely portrayed as a flip-flop.
In fact, Clinton made the right decision in holding out for a better WTO deal. A lot more horse trading is
needed before a final agreement can be reached. And without the Administrations goal of a bullet-proof agree-
ment that business lobbyists can enthusiastically sell to a Republican Congress, the whole process will end up in
partisan acrimony that could harm relations with China for years.
THE HARD PART. Many business lobbyists, while disappointed that the deal was not closed, agree that
better terms can still be had. And Treasury Secretary Robert E. Rubin, National Economic Council Director Gene
B. Sperling, Commerce Secretary William M. Daley, and top trade negotiator Charlene Barshefsky all advised
Clinton that while the Chinese had made a remarkable number of concessions, were not there yet, according to
senior officials.
Negotiating with Zhu over the remaining issues may be the easy part. Although Clinton can signal U.S. ap-
proval for Chinas entry into the WTO himself, he needs Congress to grant Beijing permanent most-favored-nation
status as part of a broad trade accord. And the temptation for meddling on Capital Hill may prove over-whelming.
Zhu had barely landed before Senate Majority Leader Trent Lott (R-Miss) declared himself skeptical that China
deserved entry into the WTO. And Senators Jesse A. Helms (R-N.C.) and Emest F. Hollings (D-S. C.) promised to
introduce a bill requiring congressional approval of any deal.
The hidden message from these three textile-state Southerners: Get more protection for the U. S. clothing
industry. Hoping to smooth the way, the Administration tried, but failed, to budge Zhu on textiles. Also left in the
lurch: Wall Street, Hollywood, and Detroit. Zhu refused to open up much of the lucrative Chinese securities market
and insisted on cultural restrictions on American movies and music. He also blocked efforts to allow U. S. auto
makers to provide fleet financing.
BIG JOB. Already, business lobbyists are blanketing Capitol Hill to presale any eventual agreement, but
what theyve heard so far isnt encouraging. Republicans, including Lott, say that the time just isnt right for the
deal. Translation: Were determined to make it look as if Clinton has capitulated to the Chinese and is ignoring
human, religious, and labor rights violations; the theft of nuclear-weapons technology; and the sale of missile parts
to Americas enemies. Beijings fierce critics within the Democratic Party, such as Senator Paul D. Wellstone of
Minnesota and House Minority leader Richard A. Gephardt of Missouri, wont help, either.
Just how tough the lobbying job on Capitol Hill will be become clear on Apr. 20, when Rubin lectured
19chief executives on the need to discipline their Republican allies. With business and the White House still trading
charges over who is responsible for the defeat of fast-track trade negotiating legislation in 1997, working together
wont be easy. And Republicanswith a winksay that theyll eventually embrace Chinas entry into the WTO as a
favor to Corporate America. Though not long before they torture Clinton. But Zhu is out on a limb, and if Congress
overdoes the criticism, he may be forced by domestic critics to renege. Business must make this much dear to both
its GOP allies and the Whit House: This historic deal is too important to risk losing to any more partisan squab-
bling.
114
1. The main idea of this passage is
[A] The Contradiction between the Democratic Party and the Republican Party.
[B] On Chinas entry into WTO.
[C] Clinton was right.
[D] Business Lobbyists Control Capitol Hill.
2. What does the sentence Also left in the lurch: Wall Street, Hollywood, Detroit convey?
[A] Premier Zhu rejected their requirements.
[B] The three places overdid criticism.
[C] They wanted more protection.
[D] They are in trouble.
3. What was the attitude of the Republican Party toward Chinas entry into the WTO?
[A] Contradictory.
[B] Appreciative.
[C] Disapproving.
[D] Detestful.
4. Who plays the leading part in the deal in America?
[A] White House .
[B] Republicans.
[C] The Democratic Party.
[D] Businessmen.
5. It can be inferred from the passage that
[A] America will make concessions.
[B] America will hold out for a better WTO
[C] Clinton has the right to signal U. S. approval for Chinas entry.
[D] Democratic party approve Chinas entry into the WTO.

VOCABULARY
1. drubbing
get/take a drabbing
2. flip-flop=great change suddenly
3. hold out
hold out for sth.
4. horse trading
5. bullet-proof
6. lobby
7. lobbyist
8. partisan
9. acrimony /
10. sell to
11. meddle
12. Capitol Hill
13. budge
14. lucrative
15. block
16. fleet
17. blanket

115
18. Capitulate
19. fast track
20. with a wink
21. out on a limb
22. renege
23. squabble

ANSWER
1. C 2. A 3. A 4. D 5. A

116
Passage 2: Europes Gypsies, Are They a Nation?

The striving of countries in Central Europe to enter the European Union may offer an unprecedented
chance to the continents Gypsies (or Roman) to be recognized as a nation, albeit one without a defined territory.
And if they were to achieve that they might even seek some kind of formal placeat least a total population out-
numbers that of many of the Unions present and future countries. Some experts put the figure at 4m-plus; some
proponents of Gypsy rights go as high as 15m.
Unlike Jews, Gypsies have had no known ancestral land to hark back to. Though their language is related
to Hindi, their territorial origins are misty. Romanian peasants held them to be born on the moon. Other Euro-
peans (wrongly) thought them migrant Egyptians, hence the derivative Gypsy. Most probably they were itinerant
metal workers and entertainers who drifted west from India in the 7th century.
However, since communism in Central Europe collapsed a decade ago, the notion of Romanestan as a
landless nation founded on Gypsy culture has gained ground. The International Romany Union, which says it
stands for 10m Gypsies in more than 30 countries, is fostering the idea of self-rallying. It is trying to promote
a standard and written form of the language; it waves a Gypsy flag (green with a wheel) when it lobbies in such
places as the United Bations; and in July it held a congress in Prague, The Czech capital. Where President Vaclav
Havel said that Gypsies in his own country and elsewhere should have a better deal.
At the congress a Slovak-born lawyer, Emil Scuka, was elected president of the International Tomany
Union. Later this month a group of elected Gypsy politicians, including members of parliament, mayors and local
councilors from all over Europe (OSCE), to discuss how to persuade more Gypsies to get involved in politics.
The International Romany Union is probably the most representative of the outfits that speak for Gypsies,
but that is not saying a lot. Of the several hundred delegates who gathered at its congress, few were democratically
elected; oddly, none came from Hungary, whose Gypsies are perhaps the worlds best organized, with some 450
Gypsy bodies advising local councils there. The union did, however, announce its ambition to set up a parliament,
but how it would actually be elected was left undecided.
So far, the European Commission is wary of encouraging Gypsies to present themselves as a nation. The
might, it is feared, open a Pandoras box already containing Basques, Corsicans and other awkward peoples.
Besides, acknowledging Gypsies as a nation might backfire, just when several countries, particularly Hungary,
Slovakia and the Czech Republic, are beginning to treat them better, in order to qualify for EU membership. The
EUs whole premise is to overcome differences, not to highlight them, says a nervous Eurocrat.
But the idea that the Gypsies should win some kind of special recognition as Europes largest continent
wide minority, and one with a terrible history of persecution, is catching on . Gypsies have suffered many pogroms
over the centuries. In Romania, the country that still has the largest number of them (more than 1m), in the 19th
century they were actually enslaved. Hitler tried to wipe them out, along with the Jews.
Gypsies deserve some space within European structures, says Jan Marinus Wiersma, a Dutchman in
the European Parliament who suggests that one of the current commissioners should be responsible for Gypsy af-
fairs. Some prominent Gypsies say they should be more directly represented, perhaps with a quota in the European
Parliament. That, they argue, might give them a boost. There are moves afoot to help them to get money for, among
other things, a Gypsy university.
One big snag is that Europes Gypsies are, in fact, extremely heterogeneous. They belong to many different,
and often antagonistic, clans and tribes, with no common language or religion, Their self-proclaimed leaders have
often proved quarrelsome and corrupt. Still, says, Dimitrina Petrova, head of the European Roma Rights Center in
Budapest, Gypsies shared experience of suffering entitles them to talk of one nation; their potential unity, she says,
stems from being regarded as sub-human by most majorities in Europe.

117
And they have begun to be a bit more pragmatic. In Slovakia and Bulgaria, for instance, Gypsy political
parties are trying to form electoral blocks that could win seats in parliament. In Macedonia, a Gypsy party already
has someand even runs a municipality. Nicholas Gheorge, an expert on Gypsy affairs at the OSCE, reckons that,
spread over Central Europe, there are now about 20 Gypsy MPS and mayors, 400-odd local councilors, and a
growing number of businessmen and intellectuals.
That is far from saying that they have the people or the cash to forge a nation. But, with the Gypsy question
on the EUs agenda in Central Europe, they are making ground.

1. The Best Title of this passage is


[A] Gypsies Want to Form a Nation.
[B] Are They a Nation.
[C] EU Is Afraid of Their Growth.
[D] They Are a Tribe
2. Where are the most probable Gypsy territory origins?
[A] Most probably they drifted west from India in the 7th century.
[B] They are scattered everywhere in the world.
[C] Probably, they stemmed from Central Europe.
[D] They probably came from the International Romany Union.
3. What does the International Romany lobby for?
[A] It lobbies for a demand to be accepted by such international organizations as EU and UN.
[B] It lobbies for a post in any international Romany Union.
[C] It lobbies for the right as a nation.
[D] It lobbies for a place in such international organizations as the EU or UN.
4. Why is the Europe Commission wary of encouraging Gypsies to present themselves as a nation?
[A] It may open a Pandoras Box.
[B] Encouragement may lead to some unexpected results.
[C] It fears that the Basgnes, Corsicans and other nations seeking separation may raise the same demand.
[D] Gyspsies demand may highlight the difference in the EU.
5. The big problem lies in the fact that
[A] Gypsies belong to different and antagonistic clans and tribes without a common language or religion.
[B] Their leaders prove corrupt.
[C] Their potential unity stems from being regarded as sub-human.
[D] They are a bit more pragmatic.

VOCABULARY
1. albeit
2. outnumber
3. ethnic
4. Hindi
5. misty
6. derivative
7. itinerant
8. Romanesten
Romanes
Stan
9. outfit

118
10. local
11. wary
12. backfire
13. highlight
14. persecution
15. catch on =to become popular
16. pogrom
17. commissioner
18. quota
19. snag
20. heterogeneous
21. antagonistic
22. clan
23. tribe
24. pragmatic
25. municipality
26. Rom

ANSWER
1. B 2. A 3. D 4. C 5. A

119
Passage 3: Method of Scientific Inquiry

Why the inductive and mathematical sciences, after their first rapid development at the culmination of
Greek civilization, advanced so slowly for two thousand yearsand why in the following two hundred years a
knowledge of natural and mathematical science has accumulated, which so vastly exceeds all that was previously
known that these sciences may be justly regarded as the products of our own timesare questions which have
interested the modern philosopher not less than the objects with which these sciences are more immediately conver-
sant. Was it the employment of a new method of research, or in the exercise of greater virtue in the use of the old
methods, that this singular modern phenomenon had its origin? Was the long period one of arrested development,
and is the modern era one of normal growth? Or should we ascribe the characteristics of both periods to so-called
historical accidentsto the influence of conjunctions in circumstances of which no explanation is possible, save in
the omnipotence and wisdom of a guiding Providence?
The explanation which has become commonplace, that the ancients employed deduction chiefly in their
scientific inquiries, while the moderns employ induction, proves to be too narrow, and fails upon close examina-
tion to point with sufficient distinctness the contrast that is evident between ancient and modern scientific doctrines
and inquiries. For all knowledge is founded on observation, and proceeds from this by analysis, by synthesis and
analysis, by induction and deduction, and if possible by verification, or by new appeals to observation under the
guidance of deductionby steps which are indeed correlative parts of one method; and the ancient sciences afford
examples of every one of these methods, or parts of one method, which have been generalized from the examples of
science.
A failure to employ or to employ adequately any one of these partial methods, an imperfection in the arts
and resources of observation and experiment, carelessness in observation, neglect of relevant facts, by appeal to
experiment and observationthese are the faults which cause all failures to ascertain truth, whether among the
ancients or the moderns; but this statement does not explain why the modern is possessed of a greater virtue, and
by what means he attained his superiority. Much less does it explain the sudden growth of science in recent times.
The attempt to discover the explanation of this phenomenon in the antithesis of facts and theories or facts
and ideasin the neglect among the ancients of the former, and their too exclusive attention to the latter
proves also to be too narrow, as well as open to the charge of vagueness. For in the first place, the antithesis is
not complete. Facts and theories are not coordinate species. Theories, if true, are factsa particular class of facts
indeed, generally complex, and if a logical connection subsists between their constituents, have all the positive at-
tributes of theories.
Nevertheless, this distinction, however inadequate it may be to explain the source of true method in science,
is well founded, and connotes an important character in true method. A fact is a proposition of simple. A theory, on
the other hand, if true has all the characteristics of a fact, except that its verification is possible only by indirect, re-
mote, and difficult means. To convert theories into facts is to add simple verification, and the theory thus acquires
the full characteristics of a fact.

1. The title that best expresses the ideas of this passage is


[A] Philosophy of mathematics.
[B] The Recent Growth in Science.
[C] The Verification of Facts.
[D] Methods of Scientific Inquiry.
2. According to the author, one possible reason for the growth of science during the days of the ancient Greeks and
in modern times is

120
[A] the similarity between the two periods.
[B] that it was an act of God.
[C] that both tried to develop the inductive method.
[D] due to the decline of the deductive method.
3. The difference between fact and theory
[A] is that the latter needs confirmation.
[B] rests on the simplicity of the former.
[C] is the difference between the modern scientists and the ancient Greeks.
[D] helps us to understand the deductive method.
4. According to the author, mathematics is
[A] an inductive science.
[B] in need of simple verification.
[C] a deductive science.
[D] based on fact and theory.
5. The statement Theories are facts may be called.
[A] a metaphor.
[B] a paradox.
[C] an appraisal of the inductive and deductive methods.
[D] a pun.

VOCABULARY
1. inductive
2. deductive
3. culmination /
4. conversant (with)
5. exercise
singular
6. conjunction
7. omnipotence /
8. Providence
9. commonplace
10. inquiry
11. doctrine
12. correlative
13. antithesis
14. coordinate
15. subsist
16. attribute
17. connote

ANSWER
1. D 2. B 3. A 4. C 5. B

121
Passage 4: It Is Bush

On the 36th day after they had voted, Americans finally learned Wednesday who would be their next presi-
dent: Governor George W. Bush of Texas.
Vice President Al Gore, his last realistic avenue for legal challenge closed by a U. S. Supreme Court deci-
sion late Tuesday, planned to end the contest formally in a televised evening speech of perhaps 10 minutes, advisers
said.
They said that Senator Joseph Lieberman, his vice presidential running mate, would first make brief com-
ments. The men would speak from a ceremonial chamber of the Old Executive office Building, to the west of the
White House.
The dozens of political workers and lawyers who had helped lead Mr. Gores unprecedented fight to claw
a come-from-behind electoral victory in the pivotal state of Florida were thanked Wednesday and asked to stand
down.
The vice president has directed the recount committee to suspend activities, William Daley, the Gore
campaign chairman, said in a written statement.
Mr. Gore authorized that statement after meeting with his wife, Tipper, and with top advisers including Mr.
Daley.
He was expected to telephone Mr. Bush during the day. The Bush campaign kept a low profile and moved
gingerly, as if to leave space for Mr. Gore to contemplate his next steps.
Yet, at the end of a trying and tumultuous process that had focused world attention on sleepless vote coun-
ters across Florida, and on courtrooms form Miami to Tallahassee to Atlanta to Washington the Texas governor
was set to become the 43d U. S. president.
The news of Mr. Gores plans followed the longest and most rancorous dispute over a U. S. presidential
election in more than a century, one certain to leave scars in a badly divided country.
It was a bitter ending for Mr. Gore, who had outpolled Mr. Bush nationwide by some 300000 votes, but,
without Florida, fell short in the Electoral College by 271votes to 267the narrowest Electoral College victory
since the turbulent election of 1876.
Mr. Gore was said to be distressed by what he and many Democratic activists felt was a partisan decision
from the nations highest court.
The 5-to 4 decision of the Supreme Court held, in essence, that while a vote recount in Florida could be
conducted in legal and constitutional fashion, as Mr. Gore had sought, this could not be done by the Dec. 12 dead-
line for states to select their presidential electors.
James Baker 3rd, the former secretary of state who represented Mr. Bush in the Florida dispute, issued a
short statement after the U. S. high court ruling, saying that the governor was very pleased and gratified.
Mr. Bush was planning a nationwide speech aimed at trying to begin to heal the countrys deep, aching and
varied divisions. He then was expected to meet with congressional leaders, including Democrats. Dick Cheney, Mr.
Bushs ruing mate, was meeting with congressmen Wednesday in Washington.
When Mr. Bush, who is 54, is sworn into office on Jan.20, he will be only the second son of a president to
follow his father to the White House, after John Adams and John Quincy Adams in the early 19th century.
Mr. Gore, in his speech, was expected to thank his supporters, defend his hive-week battle as an effort to
ensure, as a matter of principle, that every vote be counted, and call for the nation to join behind the new president.
He was described by an aide as resolved and resigned.
While some constitutional experts had said they believed states could present electors as late as Dec. 18,
the U. S. high court made clear that it saw no such leeway.

122
The U.S. high court sent back for revision to the Florida court its order allowing recounts but made
clear that for all practical purposes the election was over.
In its unsigned main opinion, the court declared, The recount process, in its features here described, is
inconsistent with the minimum procedures necessary to protect the fundamental right of each voter.
That decision, by a court fractured along philosophical lines, left one liberal justice charging that the high
courts proceedings bore a political taint.
Justice John Paul Stevens wrote in an angry dissent: Although we may never know with complete certain-
ty the identity of the winner of this years presidential election, the identity of the loser is perfectly clear. It is the
nations confidence in the judge as an impartial guardian of the law.
But at the end of five seemingly endless weeks, during which the physical, legal and constitutional ma-
chines of the U. S. election were pressed and sorely tested in ways unseen in more than a century, the system finally
produced a result, and one most Americans appeared to be willing at lease provisionally to support.
The Bush team welcomed the news with an outward show of restraint and aplomb. The governors hopes
had risen and fallen so many times since Election night, and the legal warriors of each side suffered through so
many dramatic reversals, that there was little energy left for celebration.

1. The main idea of this passage is


[A] Bushs victory in presidential election bore a political taint.
[B] The process of the American presidential election.
[C] The Supreme Court plays a very important part in the presidential election.
[D] Gore is distressed.
2. What does the sentence as if to leave space for Mr. Gore to contemplate his next step mean
[A] Bush hopes Gore to join his administration.
[B] Bush hopes Gore to concede defeat and to support him.
[C] Bush hopes Gore to congraduate him.
[D] Bush hopes Gore go on fighting with him.
3. Why couldnt Mr. Gore win the presidential election after he outpolled Mr. Bush in the popular vote? Because
[A] the American president is decided by the supreme courts decision.
[B] people cant directly elect their president.
[C] the American president is elected by a slate of presidential electors.
[D] the people of each state support Mr. Bush.
4. What was the result of the 54 decision of the supreme court?
[A] It was in fact for the vote recount.
[B] It had nothing to do with the presidential election.
[C] It decided the fate of the winner.
[D] It was in essence against the vote recount.
5. What did the turbulent election of 1876 imply?
[A] The process of presidential election of 2000 was the same as that.
[B] There were great similarities between the two presidential elections (2000 and 1876).
[C] It was compared to presidential election of 2000.
[D] It was given an example.

VOCABULARY
1. avenue //
2. running mate

123
3. pivotal
4. gingerly=carefully
5. tumultuous
6. rancorous
7. elector
8. elector college
9. leeway
10. for all practical purpose
11. fracture
12. taint
13. dissent
14. provisionally
15. aplomb
16. restraint

ANSWER
1. A 2. B 3. C 4. D 5. B

124
Passage 5: Womens Positions in the 17th Century

Social circumstances in Early Modern England mostly served to repress womens voices. Patriarchal
culture and institutions constructed them as chaste, silent, obedient, and subordinate. At the beginning of the 17th
century, the ideology of patriarchy, political absolutism, and gender hierarchy were reaffirmed powerfully by King
James in The Trew Law of Free Monarchie and the Basilikon Doron; by that ideology the absolute power of God
the supreme patriarch was seen to be imaged in the absolute monarch of the state and in the husband and father of
a family. Accordingly, a womans subjection, first to her father and then to her husband, imaged the subjection of
English people to their monarch, and of all Christians to God. Also, the period saw an outpouring of repressive or
overtly misogynist sermons, tracts, and plays, detailing womens physical and mental defects, spiritual evils, rebel-
liousness, shrewish ness, and natural inferiority to men.
Yet some social and cultural conditions served to empower women. During the Elizabethan era (1558
1603) the culture was dominated by a powerful Queen, who provided an impressive female example though she left
scant cultural space for other women. Elizabethan women writers began to produce original texts but were occu-
pied chiefly with translation. In the 17th century, however, various circumstances enabled women to write original
texts in some numbers. For one thing, some counterweight to patriarchy was provided by female communities
mothers and daughters, extended kinship networks, close female friends, the separate court of Queen Anne (King
James consort) and her often oppositional masques and political activities. For another, most of these women
had a reasonably good education (modern languages, history, literature, religion, music, occasionally Latin) and
some apparently found in romances and histories more expansive terms for imagining womens lives. Also, repre-
sentation of vigorous and rebellious female characters in literature and especially on the stage no doubt helped to
undermine any monolithic social construct of womens mature and role.
Most important, perhaps, was the radical potential inherent in the Protestant insistence on every Chris-
tians immediate relationship with God and primary responsibility to follow his or her individual conscience. There
is plenty of support in St Pauls epistles and elsewhere in the Bible for patriarchy and a wifes subjection to her
husband, but some texts (notably Galatians 3:28) inscribe a very different politics, promoting womens spiritual
equality: There is neither Jew nor Greek, there is neither bond nor free, there is neither male nor female: for ye
are all one in Jesus Christ. Such texts encouraged some women to claim the support of God the supreme patriarch
against the various earthly patriarchs who claimed to stand toward them in his stead.
There is also the gap or slippage between ideology and common experience. English women throughout the
17th century exercised a good deal of accrual power: as managers of estates in their husbands absences at court
or on military and diplomatic missions; as members of guilds; as wives and mothers who apex during the English
Civil War and Interregnum (1640-60) as the execution of the King and the attendant disruption of social hierar-
chies led many women to seize new rolesas preachers, as prophetesses, as deputies for exiled royalist husbands,
as writers of religious and political tracts.

1. What is the best title for this passage?


[A] Womens Position in the 17th Century.
[B] Womens Subjection to Patriarchy.
[C] Social Circumstances in the 17th Century.
[D] Womens objection in the 17th Century.
2. What did the Queen Elizabeth do for the women in culture?
[A] She set an impressive female example to follow.
[B] She dominated the culture.

125
[C] She did little.
[D] She allowed women to translate something.
3. Which of the following is Not mention as a reason to enable women to original texts?
[A]Female communities provided some counterweight to patriarchy.
[B] Queen Annes political activities.
[C] Most women had a good education.
[D] Queen Elizabeths political activities.
4. What did the religion so for the women?
[A] It did nothing.
[B] It too asked women to be obedient except some texts.
[C] It supported women.
[D] It appealed to the God.

VOCABULARY
1. repress
2. patriarchy
3. chaste
4. hierarchy
5. monarch
6. image
7. overtly
8. outpour
9. sermon
10. tract
11. misogynist
12. shrewish
13. counterweight
14. consort
15. masque
16. monolithic
17. epistle
18. Galatians
19. inscribe

ANSWER
1. A 2. C 3. D 4. B

126
Passage 6: The Present Is the Most Important

Shams and delusions are esteemed for soundest truths, while reality is fabulous. If men would steadily
observe realities only, and not allow themselves to be deluded, life, to compare it with such things as we know,
would be like a fairy tale and the Arabian Nights Entertainments. If we respected only what is inevitable and has a
right to be , music and poetry would resound along the streets. When we are unhurried and wise, we perceive that
only great and worthy things have any permanent and absolute existence, --that petty fears and petty pleasure are
but the shadow of reality. This is always exhilarating and sublime. By closing the eyes and slumbering, by consent-
ing to be deceived by shows, men establish and confirm their daily life of routine and habit everywhere, which still
is built on purely illusory foundation. Children, who play life, discern its true law and relations more clearly than
men, who fail to live worthily, but who think that they are wiser by experience, that is, by failure. I have read in a
Hindoo book, that there was a kings son, who, being expelled in infancy from his native city, was brought up by a
forester, and, growing up to maturity in that state, imagined himself to belong to the barbarous race with which be
lived.
One of his fathers ministers having discovered him, revealed to him what he was, and the misconception
of his character was removed, and he knew himself to be a prince. So soul, from the circumstances in which it is
placed, mistakes its own character, until the truth is revealed to it by some holy teacher, and then it knows itself to
be Brahme. We think that that is which appears to be. If a man should give us an account of the realities he be-
held, we should not recognize the place in his description. Look at a meeting-house, or a court-house, or a jail, or
a shop. Or a dwelling-house, and say what that thing really is before a true gaze, and they would all go to pieces in
your account of them. Men esteem truth remote, in the outskirts of the system, behind the farthest star, before Adam
and after the last man. In eternity there is indeed something true and sublime. But all these times and places and
occasions are now and here. God himself culminates in the present moment, and will never be more divine in the
lapse of all ages. And we are enabled to apprehend at all what is sublime and noble only by the perpetual instilling
and drenching of the reality that surrounds us. The universe constantly and obediently answers to our conceptions;
whether we travel fast or slow, the track is laid for us. Let us spend our lives in conceiving then. The poet or the
artist never yet had as fair and noble a design but some of his posterity at least could accomplish it.

1. The writers attitude toward the arts is one of


[A] admiration.
[B] indifference.
[C] suspicion.
[D] repulsion
2. The author believes that a child.
[A] should practice what the Hindoos preach.
[B] frequently faces vital problems better than grownups do.
[C] hardly ever knows his true origin.
[D] is incapable of appreciating the arts.
3. The author is primarily concerned with urging the reader to
[A] look to the future for enlightenment.
[B] appraise the present for its true value.
[C] honor the wisdom of the past ages.
[D] spend more time in leisure activities.
4. The passage is primarily concerned with problem of

127
[A] history and economics.
[B] society and population.
[C] biology and physics.
[D] theology and philosophy.

VOCABULARY
1. sham
2. delusion
3. fabulous
4. exhilarating
5. sublime
6. slumber
7. Hindoo
8. Brahma
9. come, fall, go to pieces
10. culminate
11. lapse /
12. apprehend
13. instill
14. drench
15. posterity
16. look to

ANSWER
1. A 2. B 3. B 4. D

128
Passage 7: Forecasting of Statistics

Nearly two thousand years have passed since a census decreed by Caesar Augustus become part of the
greatest story ever told. Many things have changed in the intervening years. The hotel industry worries more about
overbuilding than overcrowding, and if they had to meet an unexpected influx, few inns would have a manager to
accommodate the weary guests. Now it is the census taker that does the traveling in the fond hope that a highly
mobile population will stay long enough to get a good sampling. Methods of gathering, recording, and evaluating
information have presumably been improved a great deal. And where then it was the modest purpose of Rome to
obtain a simple head count as an adequate basis for levying taxes, now batteries of complicated statistical se-
ries furnished by governmental agencies and private organizations are eagerly scanned and interpreted by sages
and seers to get a clue to future events. The Bible does not tell us how the Roman census takers made out, and as
regards our more immediate concern, the reliability of present day economic forecasting, there are considerable
differences of opinion. They were aired at the celebration of the 125th anniversary of the American Statistical Asso-
ciation. There was the thought that business forecasting might well be on its way from an art to a science, and some
speakers talked about newfangled computers and high-falutin mathematical system in terms of excitement and en-
dearment which we, at least in our younger years when these things mattered, would have associated more readily
with the description of a fair maiden. But others pointed to the deplorable record of highly esteemed forecasts and
forecasters with a batting average below that of the Mets, and the President-elect of the Association cautioned that
high powered statistical methods are usually in order where the facts are crude and inadequate, the exact con-
trary of what crude and inadequate statisticians assume. We left his birthday party somewhere between hope and
despair and with the conviction, not really newly acquired, that proper statistical methods applied to ascertainable
facts have their merits in economic forecasting as long as neither forecaster nor public is deluded into mistaking
the delineation of probabilities and trends for a prediction of certainties of mathematical exactitude.

1. Taxation in Roman days apparently was based on


[A] wealth.
[B] mobility.
[C] population.
[D] census takers.
2. The American Statistical Association
[A] is converting statistical study from an art to a science.
[B] has an excellent record in business forecasting.
[C] is neither hopeful nor pessimistic.
[D] speaks with mathematical exactitude.
3. The message the author wishes the reader to get is
[A] statisticians have not advanced since the days of the Roman.
[B] statistics is not as yet a science.
[C] statisticians love their machine.
[D] computer is hopeful.
4. The greatest story ever told referred to in the passage is the story of
[A] Christmas.
[B] The Mets.
[C] Moses.
[D] Roman Census Takers.

129
VOCABULARY
1. census
2. decreed
3. influx
4. census taker
5. in the intervening years
6. sampling
7. presumable
8. batteries
9. sage
10. seer
11. newfangled
12. high-falutin
13. deplorable
14. batting average
15. ascertainable /
16. delineation
17. exactitude

ANSWER
1. C 2. A 3. B 4. A

130
Passage 8: Wakefield Masters Realism

Moreover, insofar as any interpretation of its author can be made from the five or six plays attributed to
him, the Wake field Master is uniformly considered to be a man of sharp contemporary observation. He was, for-
mally, perhaps clerically educated, as his Latin and music, his Biblical and patristic lore indicate. He is, still, cele-
brated mainly for his quick sympathy for the oppressed and forgotten man, his sharp eye for character, a ready ear
for colloquial vernacular turns of speech and a humor alternately rude and boisterous, coarse and happy. Hence
despite his conscious artistry as manifest in his feeling for intricate metrical and stanza forms, he is looked upon as
a kind of medieval Steinbeck, indignantly angry at, uncompromisingly and even brutally realistic in presenting the
plight of the agricultural poor.
Thus taking the play and the author together, it is mow fairly conventional to regard the former as a kind
of ultimate point in the secularization of the medieval drama. Hence much emphasis on it as depicting realistically
humble manners and pastoral life in the bleak hills of the West Riding of Yorkshire on a typically cold bight of De-
cember 24th. After what are often regarded as almost documentaries given in the three successive monologues
of the three shepherds, critics go on to affirm that the realism is then intensified into a burlesque mock-treatment of
the Nativity. Finally as a sort of epilogue or after-thought in deference to the Biblical origins of the materials, the
play slides back into an atavistic mood of early innocent reverence. Actually, as we shall see, the final scene is not
only the culminating scene but perhaps the raison detre of introductory realism.
There is much on the surface of the present play to support the conventional view of its mood of secular re-
alism. All the same, the realism of the Wakefield Master is of a paradoxical turn. His wide knowledge of people,
as well as books indicates no cloistered contemplative but one in close relation to his times. Still, that life was after
all a predominantly religious one, a time which never neglected the belief that man was a rebellious and sinful
creature in need of redemption, So deeply (one can hardly say naively of so sophisticated a writer) and implicitly
religious is the Master that he is less able (or less willing) to present actual history realistically than is the author
of the Brome Abraham and Isaac. His historical sense is even less realistic than that of Chaucer who just a few
years before had done for his own time costume romances, such as The Knights Tale, Troilus and Cressida, etc.
Moreover Chaucer had the excuse of highly romantic materials for taking liberties with history.

1. Which of the following statements about the Wakefield Master is NOT True?
[A] He was Chaucers contemporary.
[B] He is remembered as the author of five or six realistic plays.
[C] He write like John Steinbeck.
[D] HE was an accomplished artist.
2. By patristic, the author means
[A] realistic.
[B] patriotic
[C] superstitious.
[D] pertaining to the Christian Fathers.
3. The statement about the secularization of the medieval drama refers to the
[A] introduction of mundane matters in religious plays.
[B] presentation of erudite material.
[C] use of contemporary introduction of religious themes in the early days.
4. In subsequent paragraphs, we may expect the writer of this passage to
[A] justify his comparison with Steinbeck.

131
[B] present a point of view which attack the thought of the second paragraph.
[C] point out the anachronisms in the play.
[D] discuss the works of Chaucer.

VOCABULARY
1. clerically educated
2. lore
3. patristic
4. vernacular
5. boisterous
6. metrical
7. stanza
8. medieval
9. plight
10. secularization
11. pastoral
12. bleak
13. documentary
14. monologue
15. burlesque
16. Nativity
17. epilogue
18. deference
19. atavistic
20. slide back to
21. raison detre
22. all the same
23. paradoxical turn
24. cloistered
25. contemplative
26. the contemplative life
27. redemption
28. mundane
29. erudite
30. anachronism

ANSWER
1. C 2. D 3. A 4. B

132
Passage 9: The Continuity of the Religious Struggle in Britain

Though England was on the whole prosperous and hopeful, though by comparison with her neighbors she
enjoyed internal peace, she could not evade the fact that the world of which she formed a part was torn by hatred
and strife as fierce as any in human history. Men were still for from recognizing that two religions could exist side
by side in the same society; they believed that the toleration of another religion different from their own. And hence
necessarily false, must inevitably destroy such a society and bring the souls of all its members into danger of hell.
So the struggle went on with increasing fury within each nation to impose a single creed upon every subject, and
within the general society of Christendom to impose it upon every nation. In England the Reformers, or Protestants,
aided by the power of the Crown, had at this stage triumphed, but over Europe as a whole Rome was beginning
to recover some of the ground it had lost after Martin Luthers revolt in the earlier part of the century. It did this
in two ways, by the activities of its missionaries, as in parts of Germany, or by the military might of the Catholic
Powers, as in the Low Countries, where the Dutch provinces were sometimes near their last extremity under the
pressure of Spanish arms. Against England, the most important of all the Protestant nations to reconquer, military
might was not yet possible because the Catholic Powers were too occupied and divided: and so, in the 1570s Rome
bent her efforts, as she had done a thousand years before in the days of Saint Augustine, to win England back by
means of her missionaries.
These were young Englishmen who had either never given up the old faith, or having done so, had returned
to it and felt called to become priests. There being, of course, no Catholic seminaries left in England, they went
abroad, at first quite easily, later with difficulty and danger, to study in the English colleges at Douai or Rome: the
former established for the training of ordinary or secular clergy, the other for the member of the Society of Jesus,
commonly known as Jesuits, a new Order established by St, Ignatius Loyola same thirty years before. The secu-
lars came first; they achieved a success which even the most eager could hardly have expected. Cool-minded and
well-informed men, like Cecil, had long surmised that the conversion of the English people to Protestantism was for
from complete; manyCecil thought even the majorityhad conformed out of fear, self-interest orpossibly the
commonest reason of allsheer bewilderment at the rapid changes in doctrine and forms of worship imposed on
them in so short a time. Thus it happened that the missionaries found a welcome, not only with the families who had
secretly offered them hospitality if they came, but with many others whom their first hosts invited to meet them or
passed them on to. They would land at the ports in disguise, as merchants, courtiers or what not, professing some
plausible business in the country, and make by devious may for their first house of refuge. There they would admin-
ister the Sacraments and preach to the house holds and to such of the neighbors as their hosts trusted and presently
go on to some other locality to which they were directed or from which they received a call.

1. The main idea of this passage is


[A] The continuity of the religious struggle in Britain in new ways.
[B] The conversion of religion in Britain.
[C] The victory of the New religion in Britain.
[D] England became prosperous.
2. What was Martin Luthers religions?
[A] Buddhism.
[B] Protestantism.
[C] Catholicism.
[D] Orthodox.
3. Through what way did the Rome recover some of the lost land?

133
[A] Civil and military ways.
[B] Propaganda and attack.
[C] Persuasion and criticism.
[D] Religious and military ways.
4. What did the second paragraph mainly describe?
[A] The activities of missionaries in Britain.
[B] The conversion of English people to Protestantism was far from complete.
[C] The young in Britain began to convert to Catholicism
[D] Most families offered hospitality to missionaries.

VOCABULARY
1. evade
2. creed
3. the Crown /
4. low Countries
5. last extremity
6. bend ones effort
7. seminary /
8. surmise
9. doctrine
10. plausible /
11. courtier
12. devious
13. Sacrament /
14. secular
15. the society of Jesus
16. Douai
17. Jesuit

ANSWER
1. A 2. B 3. D 4. A

134
Passage 10: Photography and Art

The earliest controversies about the relationship between photography and art centered on whether pho-
tographs fidelity to appearances and dependence on a machine allowed it to be a fine art as distinct from merely
a practical art. Throughout the nineteenth century, the defence of photography was identical with the struggle
to establish it as a fine art. Against the charge that photography was a soulless, mechanical copying of reality,
photographers asserted that it was instead a privileged way of seeing, a revolt against commonplace vision, and no
less worthy an art than painting.
Ironically, now that photography is securely established as a fine art, many photographers find it preten-
tious or irrelevant to label it as such. Serious photographers variously claim to be finding, recording, impartially
observing, witnessing events, exploring themselvesanything but making works of art. They are no longer willing
to debate whether photography is or is not a fine art, except to proclaim that their own work is not involved with
art. It shows the extent to which they simply take for granted the concept of art imposed by the triumph of Modern-
ism: the better the art, the more subversive it is of the traditional aims of art.
Photographers disclaimers of any interest in making art tell us more about the harried status of the con-
temporary notion of art than about whether photography is or is not art. For example, those photographers who
suppose that, by taking pictures, they are getting away from the pretensions of art as exemplified by painting re-
mind us of those Abstract Expressionist painters who imagined they were getting away from the intellectual austeri-
ty of classical Modernist painting by concentrating on the physical act of painting. Much of photographys prestige
today derives from the convergence of its aims with those of recent art, particularly with the dismissal of abstract
art implicit in the phenomenon of Pop painting during the 1960s. Appreciating photographs is a relief to sensibili-
ties tired of the mental exertions demanded by abstract art. Classical Modernist paintingthat is, abstract art as
developed in different ways by Picasso, Kandinsky, and Matissepresupposes highly developed skills of looking
and a familiarity with other paintings and the history of art. Photography, like Pop painting, reassures viewers that
art is not hard; photography seems to be more about its subjects than about art.
Photography, however, has developed all the anxieties and self-consciousness of a classic Modernist art.
Many professionals privately have begun to worry that the promotion of photography as an activity subversive of
the traditional pretensions of art has gone so far that the public will forget that photography is a distinctive and
exalted activityin short, an art.

1. What is the author mainly concerned with? The author is concerned with
[A] defining the Modernist attitude toward art.
[B] explaining how photography emerged as a fine art.
[C] explaining the attitude of serious contemporary photographers toward photography as art and placing
those attitudes in their historical context.
[D] defining the various approaches that serious contemporary photographers take toward their art and as-
sessing the value of each of those approaches.
2. Which of the following adjectives best describes the concept of art imposed by the triumph of Modernism as
the author represents it in lines 1213?
[A] Objective
[B] Mechanical.
[C] Superficial.
[D] Paradoxical.
3. Why does the author introduce Abstract Expressionist painter?

135
[A] He wants to provide an example of artists who, like serious contemporary photographers, disavowed tra-
ditionally accepted aims of modern art.
[B] He wants to set forth an analogy between the Abstract Expressionist painters and classical Modernist
painters.
[C] He wants to provide a contrast to Pop artist and others.
[D] He wants to provide an explanation of why serious photography, like other contemporary visual forms, is
not and should not pretend to be an art.
4. How did the nineteenth-century defenders of photography stress the photography?
[A] They stressed photography was a means of making people happy.
[B] It was art for recording the world.
[C] It was a device for observing the world impartially.
[D] It was an art comparable to painting.

VOCABULARY
1. fine arts
2. assert
3. privileged
4. pretentious
5. irrelevant
6. subversive
7. disclaimer
8. harry
9. austerity
10. convergence
11. implicit
12. distinctive
13. exalted

ANSWER
1. C 2. D 3. A 4. D

136

Das könnte Ihnen auch gefallen